├── .gitignore ├── figure ├── ODE1_2_1.PNG ├── ODE1_2_2.PNG └── ODE1_2_3.PNG ├── README.md ├── main.tex ├── settings.tex ├── preface.tex ├── ref.bib ├── chapter09.tex ├── chapter01.tex ├── chapter08.tex ├── chapter05.tex ├── chapter04.tex ├── chapter03.tex ├── chapter07.tex ├── chapter02.tex └── chapter06.tex /.gitignore: -------------------------------------------------------------------------------- 1 | *.aux 2 | *.pdf 3 | *.log 4 | *.toc 5 | *.out 6 | *.bbl 7 | *.blg -------------------------------------------------------------------------------- /figure/ODE1_2_1.PNG: -------------------------------------------------------------------------------- https://raw.githubusercontent.com/SwitWu/ODE-DingTongren-Solutions/HEAD/figure/ODE1_2_1.PNG -------------------------------------------------------------------------------- /figure/ODE1_2_2.PNG: -------------------------------------------------------------------------------- https://raw.githubusercontent.com/SwitWu/ODE-DingTongren-Solutions/HEAD/figure/ODE1_2_2.PNG -------------------------------------------------------------------------------- /figure/ODE1_2_3.PNG: -------------------------------------------------------------------------------- https://raw.githubusercontent.com/SwitWu/ODE-DingTongren-Solutions/HEAD/figure/ODE1_2_3.PNG -------------------------------------------------------------------------------- /README.md: -------------------------------------------------------------------------------- 1 | # Introduction 2 | 3 | 常微分方程教程(丁同仁、李承治著)课后习题解答,文档由 $\rm\LaTeX$ 制作,目前完成了前 7 章的内容。欢迎 Fork 以及 Pull requests。 -------------------------------------------------------------------------------- /main.tex: -------------------------------------------------------------------------------- 1 | \documentclass[withinsec,chinese,tikzcover]{mathexercise} 2 | 3 | \input settings 4 | 5 | \begin{document} 6 | 7 | \maketitle 8 | \tableofcontents 9 | 10 | \input{preface.tex} 11 | \input{chapter01.tex} 12 | \input{chapter02.tex} 13 | \input{chapter03.tex} 14 | \input{chapter04.tex} 15 | \input{chapter05.tex} 16 | \input{chapter06.tex} 17 | \input{chapter07.tex} 18 | \input{chapter08.tex} 19 | 20 | \nocite{*} 21 | 22 | \bibliography{ref} 23 | \addcontentsline{toc}{chapter}{参考文献} 24 | 25 | \end{document} -------------------------------------------------------------------------------- /settings.tex: -------------------------------------------------------------------------------- 1 | \usepackage{graphicx} 2 | \usepackage{wrapfig} 3 | \usepackage{fancyhdr} 4 | \usepackage{bm} 5 | \usepackage{siunitx} 6 | 7 | \newcommand\bmitPhi{\bm{\varPhi}} 8 | \newcommand\bmitPsi{\bm{\varPsi}} 9 | \newcommand\rmJ{\mathrm{J}} 10 | \renewcommand{\contentsname}{目录} 11 | 12 | \newtheorem{conclusion}{结论} 13 | 14 | 15 | \let\leq=\leqslant 16 | \let\geq=\geqslant 17 | 18 | \lineskiplimit=2.5pt 19 | \lineskip=2.5pt plus .5pt 20 | 21 | \bibliographystyle{plain} 22 | 23 | \textbook{常微分方程教程} 24 | \textbookauthor{丁同仁 李承治} 25 | \textbookversion{第二版} 26 | \title{ODE Solutions} 27 | \author{正寅} -------------------------------------------------------------------------------- /preface.tex: -------------------------------------------------------------------------------- 1 | \chapter*{前言} 2 | 3 | 4 | 5 | 这份习题解答的开始制作日期我也记得不太真切了,大概是 2020 年的时候吧, 6 | 当时用 \TeX{} 把手写的答案全部敲了一遍,工作量着实不小。 7 | 8 | 2022 年 6 月份,因一次偶然的机会,我又重新翻阅了一遍这份习题解答,发现里面有比较多的排版问题, 9 | 所以本人花了几天时间将源代码修改了一番。但无论怎样,错误和不规范的地方总是不可避免的, 10 | 如果您在参考这份习题解答的时候发现了任何问题,希望您能够及时向我反馈, 11 | 让我们一起将这份文档变得更好。 12 | 13 | 而且,这份习题解答目前只完成了前面 7 章的习题,后续章节可能也会进行完善, 14 | 当然,也欢迎热心的朋友通过 pull request 来进行助力。 15 | 16 | \begin{itemize} 17 | \item 仓库: \url{https://github.com/SwitWu/ODE-DingTongren-Solutions} 18 | \item 邮箱: \href{mailto:sywumath@gmail.com}{\ttfamily sywumath@gmail.com} 19 | \end{itemize} 20 | 21 | \begin{flushright} 22 | 2022 年 6 月 15 日 \\ 23 | 武汉珞珈山 24 | \end{flushright} -------------------------------------------------------------------------------- /ref.bib: -------------------------------------------------------------------------------- 1 | %% This BibTeX bibliography file was created using BibDesk. 2 | %% https://bibdesk.sourceforge.io/ 3 | 4 | %% Created for wsy at 2022-06-14 22:50:07 +0800 5 | 6 | 7 | %% Saved with string encoding Unicode (UTF-8) 8 | 9 | 10 | 11 | @book{T-Ding, 12 | author = {丁同仁 and 李承治}, 13 | date-added = {2022-06-14 22:48:04 +0800}, 14 | date-modified = {2022-06-14 22:50:00 +0800}, 15 | edition = {2}, 16 | publisher = {高等教育出版社}, 17 | title = {常微分方程教程}, 18 | year = {2004}} 19 | 20 | @book{R-Yuan, 21 | author = {袁荣}, 22 | date-added = {2022-06-14 22:33:49 +0800}, 23 | date-modified = {2022-06-14 22:39:38 +0800}, 24 | publisher = {高等教育出版社}, 25 | title = {常微分方程}, 26 | year = {2012},} 27 | 28 | @book{Ma-Zhou-Li, 29 | title={常微分方程定性与稳定性方法}, 30 | author={马知恩 and 周义仓 and 李承治}, 31 | volume={85}, 32 | year={2001}, 33 | publisher={科学出版社} 34 | } 35 | -------------------------------------------------------------------------------- /chapter09.tex: -------------------------------------------------------------------------------- 1 | \chapter{边值问题} 2 | 3 | \section{施图姆比较定理} 4 | 5 | \begin{exercise} 6 | 如果在定理 2 中假设 7 | \[R(x) > Q(x)\quad (x\in J),\] 8 | 则定理的结论可以加强到: $x_1 < x_0 < x_2$. 9 | \end{exercise} 10 | 11 | \begin{proof} 12 | 假设 13 | \[\psi(x) > 0, \quad (x_1 < x < x_2).\] 14 | 中间的论述过程与定理 9.2 的完全类似,最后得到 15 | \[\e^{\int_{x_1}^{x_2} p(x) \diff x} v(x_2) > v(x_1).\] 16 | 但是 17 | \[v(x_1) = \psi(x_1)\phi'(x_1) \geq 0, 18 | \quad v(x_2) = \psi(x_2)\phi'(x_2)\leq 0.\] 19 | 矛盾. 20 | \end{proof} 21 | 22 | 23 | \begin{exercise} 24 | 如果微分方程 25 | \[y'' + Q(x)y = 0\] 26 | 中的系数函数 $Q(x)$ 满足不等式 27 | \[Q(x) \leq M\quad (a\leq x<\infty),\] 28 | 其中常数 $M>0$. 则它的任何非零解 $y = \varphi(x)$ 的相邻零点的间距不小于 $\pi/\sqrt{M}$. 29 | \end{exercise} 30 | 31 | 32 | \begin{exercise} 33 | 利用定理 9.2 证明: 贝塞尔函数 $\mathrm{J}_n(x)$ 和诺伊曼函数 $\mathrm{Y}_n(x)$ 34 | 都有无穷多个零点, 而且它们各自相邻零点的间距当 $x\to\infty$ 时趋于 $\pi$. 35 | \end{exercise} 36 | 37 | 38 | \begin{exercise} 39 | 设微分方程 40 | \[\frac{\diff^2 x}{\diff t^2} + P(t)x = 0,\] 41 | 其中 $P(t)$ 是 $t$ 的连续函数, 而且满足 42 | \[n^2 < P(t) < (n+1)^2,\] 43 | 这里 $n$ 是一个非负的整数. 44 | 则上述方程的任何非零解都不是 $2\pi$ 周期的. 45 | \end{exercise} 46 | 47 | 48 | \begin{exercise} 49 | 利用定理 9.2 证明: 齐次线性微分方程 (9.1) 的任何两个线性无关的解的零点是互相交错的. 50 | \end{exercise} -------------------------------------------------------------------------------- /chapter01.tex: -------------------------------------------------------------------------------- 1 | \chapter{基本概念} 2 | 3 | 4 | 5 | \section{微分方程及其解的定义} 6 | 7 | 8 | 9 | \begin{exercise} 10 | 验证下列函数是右侧相应微分方程的解或通解: 11 | \begin{enumerate}[(1)] 12 | \item $y=C_1\e^{2x}+C_2\e^{-2x}:y''-4y=0$; 13 | \item $\displaystyle y=\frac{\sin x}{x}:xy'+y=\cos x$; 14 | \item $y=x\left(\int x^{-1}\e^x\diff x+C\right):xy'-y=x\e^x$; 15 | \item $y=\begin{cases}-\frac{1}{4}(x-C_1)^2,&-\infty0$ 是一个常数); 39 | \item $\frac{\diff y}{\diff x}=1+y^2$, $y(x_0)=y_0$. 40 | \end{enumerate} 41 | \end{exercise} 42 | 43 | \begin{solution} 44 | (1) $y(x)=\frac{1}{24}x^4+\frac{a_2}{2}x^2+a_1x+a_0$; 45 | 46 | (2) $y(x)=\int_0^xf(t)\diff t$; 47 | 48 | (3) $R(t)=\e^{-at}$; 49 | 50 | (4) $y(x)=\tan(x+\arctan y_0-x_0)$. 51 | \end{solution} 52 | 53 | 54 | 55 | \begin{exercise} 56 | 求出: 57 | \begin{enumerate}[(1)] 58 | \item 曲线族 $y=Cx+x^2$ 所满足的微分方程; 59 | \item 曲线族 $y=C_1\e^x+C_2x\e^x$ 所满足的微分方程; 60 | \item 平面上以原点为中心的一切圆所满足的微分方程; 61 | \item 平面上一切圆所满足的微分方程. 62 | \end{enumerate} 63 | \end{exercise} 64 | 65 | \begin{solution} 66 | (1) 求导得 $y'=C+2x$, 联立方程消去 $C$ 得 $y+x^2-xy'=0$. 67 | 68 | (2) 求两次导得 69 | $\begin{cases}y=C_1\e^x+C_2x\e^x\\y'=C_1\e^x+C_2(x+1)\e^x\\y''=C_1\e^x+C_2(x+2)\e^x\end{cases}$, 70 | 由前两个方程解得 $C_1=\frac{(x+1)y-xy'}{\e^x}$, $C_2=\frac{y'-y}{\e^x}$, 71 | 代入第三个方程得 $y''-2y'+y=0$. 72 | 73 | (3) 平面上以原点为中心的一切圆的参数方程为 $x^2+y^2=R^2$ ($R$ 为参数), 求导得 $x+yy'=0$. 74 | 75 | (4) 平面上一切圆的参数方程为 $(x-a)^2+(y-b)^2=R^2$ ($a,b,R$ 为参数), 76 | 关于 $x$ 求三次导并消去参数即得 $3y'(y'')^2-\bigl[1+(y')^2\bigr]y'''=0$. 77 | \end{solution} 78 | 79 | 80 | 81 | \begin{exercise} 82 | 证明: 设 $y=g(x,C_1,C_2,\cdots,C_n)$ 是一个充分光滑的函数族, 其中 $x$ 是自变量, 83 | 而 $C_1$, $C_2$, $\cdots$, $C_n$ 是 $n$ 个独立的参数(任意常数), 则存在一个形如 (1.1) 的 $n$ 阶微分方程, 84 | 使得它的通解恰好是上述函数族. 85 | \end{exercise} 86 | 87 | \begin{proof} 88 | 已知 89 | \begin{equation} 90 | \begin{cases} 91 | y=g(x,C_1,C_2,\cdots,C_n)\\ 92 | y'=g^{(1)}(x,C_1,C_2,\cdots,C_n)\\\cdots\\ 93 | y^{(n-1)}=g^{(n-1)}(x,C_1,C_2,\cdots,C_n)\\ 94 | y^{(n)}=g^{(n)}(x,C_1,C_2,\cdots,C_n) 95 | \end{cases}\tag{$\star$} 96 | \end{equation} 97 | 因为 $C_1,C_2,\cdots,C_n$ 独立, 所以 Jacobi 行列式 98 | \[\frac{D[g,g^{(1)},\cdots,g^{(n-1)}]}{D[C_1,C_2,\cdots,C_n]}=\begin{vmatrix} 99 | \frac{\partial g}{\partial C_1}&\frac{\partial g}{\partial C_2}&\cdots&\frac{\partial g}{\partial C_n} 100 | \\ 101 | \frac{\partial g^{(1)}}{\partial C_1}&\frac{\partial g^{(1)}}{\partial C_2}&\cdots&\frac{\partial g^{(1)}}{\partial C_n}\\ 102 | \vdots&\vdots&&\vdots\\ 103 | \frac{\partial g^{(n-1)}}{\partial C_1}&\frac{\partial g^{(n-1)}}{\partial C_2}&\cdots&\frac{\partial g^{(n-1)}}{\partial C_n}\end{vmatrix}\neq 0.\] 104 | 由隐函数存在定理知可由方程组 $(\star)$ 的前 $n$ 个方程解出 105 | \[C_i=C_i(x,y,\cdots,y^{(n-1)})(i=1,2,\cdots,n).\] 106 | 将之代入方程组 $(\star)$ 最后一个方程中得 107 | \[y^{(n)}=g^{(n)}(x,C_1(x,y,\cdots,y^{(n-1)}),\cdots,C_n(x,y,\cdots,y^{(n-1)})).\] 108 | 上式即为所求的 $n$ 阶微分方程. 109 | \end{proof} 110 | 111 | 112 | 113 | \section{微分方程及其解的几何解释} 114 | 115 | 116 | 117 | \begin{exercise} 118 | 作出如下微分方程的线素场: 119 | \begin{enumerate}[(1)] 120 | \item $\displaystyle y'=\frac{xy}{|xy|}$; 121 | \item $y'=(y-1)^2$; 122 | \item $y'=x^2+y^2$. 123 | \end{enumerate} 124 | \end{exercise} 125 | 126 | \begin{solution} 127 | (1)奇异点集合为 $\{(x,y)\mid x=0\text{\ 或\ }y=0\}$, 线素场如图 (Matlab 制图). 128 | \begin{figure}[htb] 129 | \centering 130 | \includegraphics[width=4cm]{figure/ODE1_2_1.png} 131 | \caption{(1)题图} 132 | \end{figure} 133 | 134 | (2)等斜线为$(y-1)^2=k\Rightarrow y=1\pm\sqrt{k}$, 线素场如图. 135 | \begin{figure}[htb] 136 | \centering 137 | \includegraphics[width=4cm]{figure/ODE1_2_2.png} 138 | \caption{(2)题图} 139 | \end{figure} 140 | 141 | (3)等斜线为$x^2+y^2=k$, 线素场如图. 142 | \begin{figure}[htb] 143 | \centering 144 | \includegraphics[width=4cm]{figure/ODE1_2_3.png} 145 | \caption{(3)题图} 146 | \end{figure} 147 | \end{solution} 148 | 149 | 150 | 151 | \begin{exercise} 152 | 利用线素场研究下列微分方程的积分曲线族: 153 | \begin{enumerate}[(1)] 154 | \item $y' = 1+xy$; 155 | \item $y' = x^2-y^2$. 156 | \end{enumerate} 157 | \end{exercise} 158 | 159 | 160 | 161 | \begin{exercise} 162 | 根据磁场的物理直观, 试作微分方程 (2.8) 的线素场及其积分曲线族的草图. 163 | \end{exercise} -------------------------------------------------------------------------------- /chapter08.tex: -------------------------------------------------------------------------------- 1 | \chapter{定性理论与分支理论初步} 2 | 3 | 4 | 5 | \setcounter{section}{1} 6 | \section{解的李雅普诺夫稳定性} 7 | 8 | 定理 8.2, 8.3 和 8.4 的证明见 \cite{R-Yuan}. 9 | 10 | \begin{exercise} 11 | 证明: 线性方程零解的渐进稳定性等价于它的全局稳定性. 12 | \end{exercise} 13 | 14 | \begin{proof} 15 | 只需要证明渐进稳定性蕴含全局渐进稳定性. 16 | 设 $X(t)$ 为基解矩阵, 则过 $(t_0, x_0)$ 的解可以表为 17 | \[x(t; t_0,x_0) = X(t) X^{-1}(t_0)x_0.\] 18 | 因为零解吸引, 所以存在 $\delta_0>0$, 使得当 $|\tilde{x}_0| < \delta_0$ 时, 有 19 | \[x(t; x_0, \tilde{x}_0) \to 0,\quad t\to\infty.\] 20 | 对任意 $x_0$, 存在常数 $c\neq 0$ 使得 $|c x_0| < \delta_0$, 故 21 | \[x(t; t_0, cx_0) = X(t)X^{-1}(x_0)(cx_0) = c X(t)X^{-1}(t_0)x_0 \to 0,\quad t\to\infty.\] 22 | 于是 $X(t)X^{-1}(t_0)x_0\to 0$ ($t\to\infty$), 这说明零解是全局渐进稳定的. 23 | \end{proof} 24 | 25 | 26 | \begin{exercise} 27 | 设 $x$ 和 $t$ 都是标量, 试求出方程 28 | \[\frac{\diff x}{\diff t} = a(t)x\] 29 | 的零解为稳定或渐进稳定的充要条件. 30 | \end{exercise} 31 | 32 | \begin{solution} 33 | 方程的解为 34 | \[x(t) = C \e^{\int_0^t a(s) \diff s}.\] 35 | 于是零解稳定的充要条件为 36 | \[\int_0^{\infty} a(s) \diff s < \infty.\] 37 | 零解渐进稳定的充要条件为 38 | \[\int_0^{\infty} a(s) \diff s = -\infty. \qedhere\] 39 | \end{solution} 40 | 41 | 42 | \begin{exercise} 43 | 对于极坐标下的方程 44 | \[\dot{\theta} = 1,\quad \dot{\theta} = \begin{cases} 45 | r^2\sin\frac{1}{r}, & \text{当\ } r>0, \\ 46 | 0, & \text{当\ } r=0, 47 | \end{cases}\] 48 | 试作出原点附近的相图, 并研究平衡点 $r=0$ 的稳定性质. 49 | \end{exercise} 50 | 51 | 52 | 53 | \begin{exercise} 54 | 设二阶常系数线性方程 55 | \[\frac{\diff\bm{x}}{\diff t} = \bm{A}\bm{x},\] 56 | 其中 $\bm{A}$ 是一个 $2\times 2$ 的常矩阵. 记 57 | \[\begin{cases} 58 | p = -\tr\bm{A}, \\ 59 | q = \det\bm{A}. 60 | \end{cases}\] 61 | 再设 $p^2+q^2\neq 0$, 试证: 62 | \begin{enumerate}[(1)] 63 | \item 当 $p>0$ 且 $q>0$ 时, 零解是渐进稳定的; 64 | \item 当 $p>0$ 且 $q=0$ 或 $p=0$ 且 $q>0$ 时, 零解是稳定的, 但不是渐进稳定的; 65 | \item 在其他情形下, 零解都是不稳定的. 66 | \end{enumerate} 67 | \end{exercise} 68 | 69 | \begin{proof} 70 | 特征多项式为 $\chi_A(\lambda) = \lambda^2 + p\lambda +q$. 于是特征根 $\lambda_1$, 71 | $\lambda_2$ 满足 72 | \[\lambda_1 + \lambda_2 = -p,\qquad \lambda_1\lambda_2 = q.\] 73 | 74 | (1) 当 $p>0$ 且 $q>0$ 时, $\lambda_1$ 和 $\lambda_2$ 的实部都为负, 因此零解是渐进稳定的. 75 | 76 | (2) 当 $p>0$ 且 $q=0$ 或 $p=0$ 且 $q>0$ 时, $\lambda_1$ 和 $\lambda_2$ 一个为零, 一个为负, 77 | 因此零解是稳定的但不是渐进稳定的. 78 | 79 | (3) 其它情形下, 零解不是稳定的. 80 | \end{proof} 81 | 82 | 83 | 84 | \begin{exercise} 85 | 讨论二维的微分方程 86 | \[\dot{x} = y-xf(x,y),\quad \dot{y} = -x-yf(x,y)\] 87 | 零解的稳定性, 其中函数 $f(x,y)$ 在 $(0,0)$ 附近是连续可微的. 88 | \end{exercise} 89 | 90 | \begin{solution} 91 | 取 $V(x,y) = \frac12 (x^2+y^2)$, 则 92 | \[\frac{\diff V}{\diff t}(x(t), y(t)) = -(x^2+y^2) f(x,y).\] 93 | 由定理 8.4 知, 若存在 $\varepsilon>0$, 使得当 $0< x^2 + y^2 < \varepsilon$ 时, 94 | $f(x,y) > 0$ ($f(x,y)\geq 0$, 或 $f(x,y)<0$), 95 | 则零解是渐进稳定的 (稳定的, 或不稳定的). 96 | \end{solution} 97 | 98 | 99 | 100 | \begin{exercise} 101 | 设 $x\in\mathbb{R}^1$, 函数 $g(x)$ 连续, 且 $xg(x)>0$ 当 $x\neq 0$. 试证方程 102 | \[\ddot{x}+g(x) = 0\] 103 | 的零解是稳定的, 但不是渐进稳定的. 104 | \end{exercise} 105 | 106 | \begin{proof} 107 | 令 $y = \dot{x}$, 则 $\ddot{x} + g(x) = 0$ 等价于 108 | \[\begin{cases} 109 | \dot{x} = y, \\ 110 | \dot{y} = -g(x). 111 | \end{cases}\] 112 | 取 $V(x,y) = \frac{1}{2}y^2 + \int_0^x g(s) \diff s$, 则由条件可知 113 | $V(x,y)$ 满足\textbf{条件 I}, 又因为 114 | \[\frac{\diff V}{\diff t} = 0,\] 115 | 因此零解是稳定的, 但不是渐进稳定的. 116 | \end{proof} 117 | 118 | 119 | 120 | \begin{exercise} 121 | 研究二维微分方程 122 | \[\dot{x} = y,\quad \dot{y} = -1+x^2\] 123 | 的两个平衡点的稳定性. 124 | \end{exercise} 125 | 126 | \begin{solution} 127 | 方程有两个平衡点 $(1,0)$ 和 $(-1,0)$. 128 | 129 | (1) 考虑 $(1,0)$ 点. 令 $u = x-1$, $v = y$, 则方程变为 130 | \[\frac{\diff u}{\diff t} = v,\qquad \frac{\diff v}{\diff t} = u^2+2u.\] 131 | 线性部分的特征函数为 $\lambda^2-2=0$, 特征根是 $\lambda_{1,2} = \pm\sqrt{2}$, 132 | 它有一个正实部的根, 所以 $(1,0)$ 是不稳定的. 133 | 134 | (2) 考虑 $(-1,0)$ 点. 令 $u = x+1$, $v=y$, 则方程变为 135 | \[\frac{\diff u}{\diff t} = v,\qquad \frac{\diff v}{\diff t} = -2u + u^2.\] 136 | 线性部分的特征根为纯虚根 $\pm\sqrt{2}\mathrm{i}$. 137 | 取 $V = u^2 + \frac{1}{2}v^2 -\frac{1}{2}u^3$, 则 $V$ 在 $(0,0)$ 138 | 附近是定正的, 且 139 | \[\frac{\diff V}{\diff t} = (2u-u^2) \dot{u} + v\dot{v} = 0,\] 140 | 因此 $(-1,0)$ 是稳定的. 141 | \end{solution} 142 | 143 | 144 | 145 | \begin{exercise} 146 | 讨论下列微分方程零解的稳定性: 147 | \begin{enumerate}[(1)] 148 | \item $\dot{x} = -y-xy^2$, $\dot{y} = x-x^4y$; 149 | \item $\dot{x} = -y^3-x^5$, $\dot{y} = x^3-y^5$; 150 | \item $\dot{x} = -x+2x(x+y)^2$, $\dot{y} = -y^3+2y^3(x+y)^2$; 151 | \item $\dot{x} = 2x^2y+y^3$, $\dot{y} = -xy^2 + 2x^5$. 152 | \end{enumerate} 153 | \end{exercise} 154 | 155 | \begin{solution} 156 | (1) 取 $V(x,y) = \frac12 (x^2+y^2)$, 则 157 | \[\frac{\diff V}{\diff t} = -x^2 y^2 - x^4y^2 \leq 0.\] 158 | 因此零解是稳定的. 159 | 160 | (2) 取 $V(x,y) = \frac{1}{4}(x^4+y^4)$, 则 161 | \[\frac{\diff V}{\diff t} = -(x^8+y^8) \leq 0.\] 162 | 因此零解是稳定的. 163 | 164 | (3) 取 $V(x,y) = \frac14 x^4 + \frac12 y^2$, 则在 $(0,0)$ 的小邻域中有 165 | \[\frac{\diff V}{\diff t} = (x^4+y^4) \bigl(2(x+y)^2 - 1\bigr) \leq 0.\] 166 | 因此零解是稳定的. 167 | 168 | (4) 取 $V(x,y) = xy$, 则 169 | \[\frac{\diff V}{\diff t} = x^2y^2 + y^4 + 2x^6.\] 170 | 因此 $\frac{\diff V}{\diff t}$ 是正定的, 但 $V$ 本身不是半负定的, 171 | 由 \cite[定理 3.3]{Ma-Zhou-Li} 知零解是不稳定的. 172 | \end{solution} 173 | 174 | 175 | \section{平面上的动力系统, 奇点与极限环} 176 | 177 | \begin{exercise} 178 | 设线性系统 (8.22) 以 $(0,0)$ 为高阶奇点, 试作出其相图. 179 | \end{exercise} 180 | 181 | 182 | \begin{exercise} 183 | 判断下列方程的奇点 $(0,0)$ 的类型, 并作出该奇点附近的相图: 184 | \begin{enumerate}[(1)] 185 | \item $\dot{x} = 4y-x$, $\dot{y} = -9x+y$; 186 | \item $\dot{x} = 2x+y+xy^2$, $\dot{y} = x+2y+x^2+y^2$; 187 | \item $\dot{x} = 2x+4y+\sin y$, $\dot{y} = x+y+\e^y-1$; 188 | \item $\dot{x} = x+2y$, $\dot{y} = 5y-2x+x^3$; 189 | \item $\dot{x} = x(1-y)$, $\dot{y} = y(1-x)$. 190 | \end{enumerate} 191 | \end{exercise} 192 | 193 | 194 | \begin{exercise} 195 | 设函数 $P(x,y)$ 和 $Q(x,y)$ 在单连通区域 $D$ 内连续可微, 且 196 | \[\frac{\partial P}{\partial x} + \frac{\partial Q}{\partial y} \neq 0, 197 | \qquad (x,y)\in D.\] 198 | 试证系统 199 | \[\dot{x} = P(x,y),\quad \dot{y} = Q(x,y)\] 200 | 在 $D$ 内不存在闭轨线. 201 | \end{exercise} -------------------------------------------------------------------------------- /chapter05.tex: -------------------------------------------------------------------------------- 1 | \setcounter{chapter}{4} 2 | \chapter{高阶微分方程} 3 | 4 | 5 | 6 | \section{几个例子} 7 | 8 | 9 | 10 | \begin{exercise} 11 | 利用线性单摆方程测量你所在地的重力常数 $g$. 12 | \end{exercise} 13 | 14 | \begin{solution} 15 | 利用单摆周期 $T=2\pi\sqrt{\frac{l}{g}}$ 即得 $g=\frac{4\pi^2}{T^2}l$, 16 | 测量摆长以及单摆完成一个周期运动的时间即可得出所在地的重力常数 $g$. 17 | \end{solution} 18 | 19 | 20 | 21 | \begin{exercise} 22 | 如果在非线性单摆方程中取 $\sin x$ 的三次近似, 即 23 | \[\sin x\sim x-\frac{x^3}{6},\] 24 | 则有单摆的三次近似方程 25 | \[\frac{\diff^2x}{\diff t^2}+a^2\left(x-\frac{x^3}{6}\right)=0.\] 26 | 由此证明单摆振动是不等时的, 而且它的相图说明可以发生进动. 27 | \end{exercise} 28 | 29 | \begin{proof} 30 | 将方程变形然后两边同时乘以 $\frac{\diff x}{\diff t}$ 即得 31 | \[\frac{\diff x}{\diff t}\frac{\diff^2x}{\diff t^2} 32 | = a^2\left(\frac{x^3}{6}-x\right)\frac{\diff x}{\diff t}.\] 33 | 积分得 34 | \[\frac{1}{2}\left(\frac{\diff x}{\diff t}\right)^2 35 | = a\left(\frac{x^4}{24}-\frac{x^2}{2}\right)+\frac{C}{2}.\] 36 | 设单摆的振幅为 $A$, 则 37 | \[\frac{\diff x}{\diff t}=\pm\sqrt{a^2\left(\frac{x^4}{12}-x^2\right)+C} 38 | = \pm\sqrt{a^2\left(\frac{x^4}{12}-x^2\right)-a^2\left(\frac{A^4}{12}-A^2\right)}.\] 39 | 故周期 $T$ 满足 40 | \[\frac{T}{4}=\int_0^A\frac{\diff x}{\sqrt{a^2\left(\frac{x^4}{12}-x^2-\frac{A^4}{12}+A^2\right)}}.\] 41 | 因此 42 | \[T=\frac{4}{a}\int_0^1\frac{A\diff u}{\sqrt{\frac{A^4}{12}(u^4-1)-A^2(u^2-1)}} 43 | = \frac{4}{a}\int_0^1\frac{\diff u}{\sqrt{(u^2-1)\left(\frac{A^2}{12}(u^2+1)-1\right)}}.\] 44 | 由于 $T$ 与 $A$ 有关, 故单摆的振动是不等时的. 45 | \end{proof} 46 | 47 | 48 | 49 | \begin{exercise} 50 | 在悬链线问题中当 $L=\sqrt{(x_2-x_1)^2+(y_2-y_1)^2}$ 时如何处理? 51 | \end{exercise} 52 | 53 | \begin{solution} 54 | 此时悬链线即为直线段, 其方程为 $y=\frac{y_2-y_1}{x_2-x_1}(x-x_1)+y_1$ $(x_1\leq x\leq x_2)$. 55 | \end{solution} 56 | 57 | 58 | 59 | \begin{exercise} 60 | 微分方程 (5.20) 表示二体问题的运动方程. 在上面求解过程中, 试适当选择积分常数, 61 | 使运动 $(x(t),y(t),z(t))$ 的轨道在一条直线上并且趋向 $O$ 点 (即二体发生碰撞); 或者使轨道是一圆周. 62 | \end{exercise} 63 | 64 | \begin{solution} 65 | (i) 运动 $(x(t),y(t),z(t))$ 的轨道在一条直线上并且趋向 $O$ 点 (即二体发生碰撞)时, 66 | $\frac{\diff\theta}{\diff t}=0$, 由 (5.28) 知 $C_3=0$, 再由 (5.27) 知 67 | $\frac{\diff r}{\diff t}=-\sqrt{\frac{2\mu}{r}+C_4}$. 68 | 69 | (ii) 运动 $(x(t),y(t),z(t))$ 的轨道是一圆周时, $\frac{\diff r}{\diff t}=0$, 70 | 故 $r=r_0$, 由 (5.27) 知 $C_4=\left(r_0\frac{\diff\theta}{\diff t}\right)^2-\frac{2\mu}{r_0}$, 71 | 又由 (5.28) 知 $r_0^2\frac{\diff\theta}{\diff t}=-C_3$, 72 | 故 $C_4=\frac{C_3^2}{r_0^2}-\frac{2\mu}{r_0}$. 73 | \end{solution} 74 | 75 | 76 | 77 | \section{$n$ 维线性空间中的微分方程} 78 | 79 | 80 | 81 | \begin{exercise} 82 | 把单摆方程 (5.7), 悬链线方程 (5.15) 和二体运动方程 (5.20) 分别写成标准微分方程组. 83 | \end{exercise} 84 | 85 | \begin{solution} 86 | (i) 单摆方程为 87 | \[\frac{\diff^2x}{\diff t^2}+a^2\sin x=0.\] 88 | 令 $x_1=x,x_2=\frac{\diff x}{\diff t}$, 则标准微分方程组为 89 | \[\begin{cases} 90 | \frac{\diff x_1}{\diff t}=x_2, \\ 91 | \frac{\diff x_2}{\diff t}=-a^2\sin x_1. 92 | \end{cases}\] 93 | (ii) 悬链线方程为 94 | \[y''=a\sqrt{1+(y')^2}.\] 95 | 令 $y_1=y,y_2=\frac{\diff y}{\diff x}$, 则标准微分方程组为 96 | \[\begin{cases} 97 | \frac{\diff y_1}{\diff t}=y_2, \\ 98 | \frac{\diff y_2}{\diff t}=a\sqrt{1+y_2^2}. 99 | \end{cases}\] 100 | (iii) 二体运动方程为 101 | \[\begin{cases} 102 | \ddot{x} = -\frac{Gm_sx}{\bigl(\sqrt{x^2+y^2+z^2}\bigr)^3}, \\ 103 | \ddot{y} = -\frac{Gm_sy}{\bigl(\sqrt{x^2+y^2+z^2}\bigr)^3}, \\ 104 | \ddot{z} = -\frac{Gm_sz}{\bigl(\sqrt{x^2+y^2+z^2}\bigr)^3}. 105 | \end{cases}\] 106 | 令$s_1=x,s_2=\frac{\diff x}{\diff t},s_3=y,s_4=\frac{\diff y}{\diff t},s_5=z,s_6=\frac{\diff z}{\diff t}$, 则标准微分方程组为 107 | \[\begin{cases} 108 | \frac{\diff s_1}{\diff t} = s_2, \\ 109 | \frac{\diff s_2}{\diff t} = -\frac{Gm_ss_1}{(\sqrt{s_1^2+s_3^2+s_5^2})^3}, \\ 110 | \frac{\diff s_3}{\diff t} = s_4,\\ 111 | \frac{\diff s_4}{\diff t} = -\frac{Gm_ss_3}{(\sqrt{s_1^2+s_3^2+s_5^2})^3}, \\ 112 | \frac{\diff s_5}{\diff t} = s_6,\\ 113 | \frac{\diff s_6}{\diff t} = -\frac{Gm_ss_5}{(\sqrt{s_1^2+s_3^2+s_5^2})^3}. 114 | \end{cases}\] 115 | \end{solution} 116 | 117 | 118 | 119 | \begin{exercise} 120 | 对 $n$ 维向量形式的微分方程, 叙述相应的皮卡存在和唯一性定理以及佩亚诺存在定理, 并写出证明的主要步骤. 121 | \end{exercise} 122 | 123 | \begin{solution} 124 | (Picard存在和唯一性定理)设初值问题 125 | \[(E):\frac{\diff\bm{y}}{\diff x}=\bm{f}(x,\bm{y}),\bm{y}(x_0)=\bm{y}_0.\] 126 | 其中函数 $\bm{f}(x,\bm{y})$ 在矩阵区域 127 | \[R:|x-x_0|\leq a,\|\bm{y}-\bm{y}_0\|\leq b\] 128 | 内连续, 而且对 $\bm{y}$ 满足 Lipschitz 条件, 则初值问题 $(E)$ 在区间 $I=[x_0-h,x_0+h]$ 129 | 上有且只有一个解 $\bm{y}=\bm{\phi}(x)$, 130 | 其中$\displaystyle h=\min\left\{a,\frac{b}{M}\right\}$, 131 | $M=\max\limits_{(x,\bm{y})\in R}\|\bm{f}(x,\bm{y})\|$. 132 | 133 | Proof: 134 | (一)初值问题($E$)等价于积分方程 135 | \[\bm{y}=\bm{y}_0+\int_{x_0}^x\bm{f}(x,\bm{y})\diff x.\] 136 | 137 | (二)用逐次迭代法构造皮卡序列 138 | \[\bm{y}_{n+1}(x)=\bm{y}_0+\int_{x_0}^x\bm{f}(x,\bm{y}_n(x))\diff x(x\in I)\quad (n=0,1,2,\cdots),\] 139 | 其中 $\bm{y}_0(x)=\bm{y}_0$, 因为 $\bm{f}(x,\bm{y}_0(x))$ 在 $I$ 上连续, 故 140 | \[\bm{y}_1(x)=\bm{y}_0+\int_{x_0}^x\bm{f}(x,\bm{y}_0(x))\diff x\] 141 | 在 $I$ 上连续可微, 而且满足不等式 142 | \[\|\bm{y}_1(x)-\bm{y}_0\|\leq\left|\int_{x_0}^x\|\bm{f}(x,\bm{y}_0(x))\|\right|\leq M|x-x_0|.\] 143 | 这就是说在 $I$ 上 $\|\bm{y}_1(x)-\bm{y}_0\|\leq Mh\leq b$. 144 | 因此, $\bm{f}(x,\bm{y}_1(x))$ 在 $I$ 上是连续的, 故 145 | \[\bm{y}_2(x)=\bm{y}_0+\int_{x_0}^x\bm{f}(x,\bm{y}_1(x))\diff x\] 146 | 在 $I$ 上连续可微, 而且满足不等式 147 | \[\|\bm{y}_2(x)-\bm{y}_0\|\leq\left|\int_{x_0}^x\|\bm{f}(x,\bm{y}_1(x))\|\right|\leq M|x-x_0|.\] 148 | 这就是说在 $I$ 上 $\|\bm{y}_2(x)-\bm{y}_0\|\leq Mh\leq b$. 149 | 由归纳法可证: Picard序列 $\{\bm{y}_n(x)\}$ 在区间 $I$ 上连续可微并且满足不等式 150 | \[\|\bm{y}_n(x)-\bm{y}_0\|\leq M|x-x_0|\quad(n=0,1,2,\cdots).\] 151 | 152 | (三) Picard序列 $\{\bm{y}_n(x)\}$ 在区间 $I$上一致收敛到积分方程的解. 153 | $\{\bm{y}_n(x)\}$ 的收敛性等价于级数 154 | \[\sum_{n=1}^{\infty}[\bm{y}_{n}(x)-\bm{y}_{n-1}(x)]\] 155 | 的收敛性, 利用归纳法证明不等式 156 | \[\|\bm{y}_n(x)-\bm{y}_{n-1}(x)\|\leq\frac{M}{L}\frac{(L|x-x_0|^n)}{n!}\quad (n=1,2,\cdots).\] 157 | 上述不等式意味着级数 $\sum\limits_{n=1}^{\infty}[\bm{y}_{n}(x)-\bm{y}_{n-1}(x)]$ 一致收敛, 158 | 故 Picard 序列 $\{\bm{y}_n(x)\}$ 一致收敛, 因此极限函数 159 | \[\bm{\phi}(x)=\lim_{n\to\infty}\bm{y}_n(x)\] 160 | 在 $I$ 上连续, 在关系式 161 | \[\bm{y}_{n+1}(x)=\bm{y}_0+\int_{x_0}^x\bm{f}(x,\bm{y}_n(x))\diff x\] 162 | 两侧取极限 $n\to\infty$, 得 163 | \[\bm{\phi}(x)=\bm{y}_0+\int_{x_0}^x\bm{f}(x,\bm{\phi}(x))\diff x.\] 164 | 故 $\bm{y}=\bm{\phi}(x)$ 是积分方程的连续解. 165 | 166 | (四)解的唯一性 167 | 168 | 设积分方程有两个解 $\bm{y}=\bm{\phi}(x)$ 和 $\bm{y}=\bm{\psi}(x)$. 169 | 设两个解的共同存在区间为 $J=[x_0-d,x_0+d]$, 则 170 | \[\bm{\phi}(x)-\bm{\psi}(x)=\int_{x_0}^x\left(\bm{f}(x,\bm{\phi}(x)) 171 | -\bm{f}(x,\bm{\psi}(x))\right)\diff x\quad(x\in J).\] 172 | 故利用李氏条件有 173 | \begin{equation} 174 | \|\bm{\phi}(x)-\bm{\psi}(x)\| 175 | \leq L\left|\int_{x_0}^x\|\bm{\phi}(x)-\bm{\psi}(x)\|\diff x\right|.\tag{$\star$} 176 | \end{equation} 177 | 注意在区间 $J$ 上, $\|\bm{\phi}(x)-\bm{\psi}(x)\|$ 是连续有界的, 故可取其一个上界 $K$, 则 178 | \[\|\bm{\phi}(x)-\bm{\psi}(x)\|\leq LK|x-x_0|.\] 179 | 将其代入 $(\star)$ 右端, 有 180 | \[\|\bm{\phi}(x)-\bm{\psi}(x)\|\leq K\frac{(L|x-x_0|)^2}{2}.\] 181 | 利用归纳法可得 182 | \[\|\bm{\phi}(x)-\bm{\psi}(x)\|\leq K\frac{(L|x-x_0|)^n}{n!}\quad (x\in J).\] 183 | 令 $n\to\infty$, 即得 184 | \[\bm{\phi}(x)=\bm{\psi}(x)\quad (x\in J).\] 185 | 故积分方程的解是唯一的. 186 | 187 | (佩亚诺存在定理)定理的叙述和证明可参考教材. 188 | \end{solution} 189 | 190 | 191 | 192 | \begin{exercise} 193 | 对 $n$ 阶线性微分方程组的初值问题, 试叙述并证明解的存在和唯一性定理. 194 | \end{exercise} 195 | 196 | \begin{solution} 197 | 设 $\bm{A}(x)$ 和 $\bm{f}(x)$ 在区间 $a0\] 267 | 对一切的 $x$ 和 $\eta$ 都成立. 268 | \end{exercise} 269 | 270 | \begin{proof} 271 | $z=\frac{\partial y}{\partial\eta}$ 满足的初值问题为 272 | \[\frac{\partial z}{\partial x} = x\cos(x\varphi)z,\quad z(0)=1.\] 273 | 直接解得 274 | \[\frac{\partial y}{\partial\eta} = z = \e^{\int_{0}^x x\cos(x\varphi)\diff x}.\] 275 | 因此 $\frac{\partial y}{\partial\eta}(x,\eta)>0$ 恒成立. 276 | \end{proof} -------------------------------------------------------------------------------- /chapter04.tex: -------------------------------------------------------------------------------- 1 | \chapter{奇解} 2 | 3 | 4 | 5 | \section{一阶隐式微分方程} 6 | 7 | 8 | 9 | \subsection{证明与总结} 10 | 11 | 12 | 13 | 考虑一阶隐式微分方程: 14 | \begin{equation} 15 | F\left(x,y,\frac{\diff y}{\diff x}\right)=0. \tag{$\star$} 16 | \end{equation} 17 | 18 | \textbullet 若可由 $(\star)$ 式导出 $y=f(x,p)$, 则 19 | \[p=f'_x(x,p)+f'_p(x,p)\frac{\diff y}{\diff x}.\] 20 | 21 | 若解出 $p=u(x,C)$, 则通解为 $y=f(x,u(x,C))$; 22 | 若解出 $x=v(p,C)$, 则通解为 23 | \[\begin{cases} 24 | x=v(p,C), \\ y=f(v(p,C),p), 25 | \end{cases}\] 26 | 其中 $p$ 视作参变量. 27 | 28 | \textbullet 若可由 $(\star)$ 式导出 $F(y,p)=0$, 即不显含自变量 $x$, 29 | 设 $y=g(t),p=h(t)$, 则 30 | \[\diff x=\frac{1}{p}\diff y=\frac{g'(t)}{h(t)}\diff t\Rightarrow x=\int\frac{g'(t)}{h(t)}\diff t.\] 31 | 故通解为 $x=\int\frac{g'(t)}{h(t)}\diff t$, $y=g(t)$. 32 | 33 | \textbullet 若可由 $(\star)$ 式导出 $F(x,p)=0$, 即不显含未知函数 $y$, 此时处理方式同上. 34 | 35 | \textbullet 设 $x=f(u,v),y=g(u,v),p=h(u,v)$, 由 $\diff y=p\diff x$ 若能解出 $v=Q(u,C)$, 36 | 则通解为 $x=f(u,Q(u,C)),y=g(u,Q(u,C))$. 37 | 38 | 39 | 40 | \subsection{习题} 41 | 42 | 43 | 44 | \begin{exercise} 45 | 求解下列微分方程: 46 | \begin{enumerate}[(1)] 47 | \item $\displaystyle 2y=p^2+4px+2x^2\left(p=\frac{\diff y}{\diff x}\right)$; 48 | \item $\displaystyle y=px\ln x+(xp)^2$; 49 | \item $\displaystyle 2xp=2\tan y+p^3\cos^2y$. 50 | \end{enumerate} 51 | \end{exercise} 52 | 53 | \begin{proof} 54 | (1) 将原方程求导得 55 | \[2p=2p\frac{\diff p}{\diff x}+4p+4x\frac{\diff p}{\diff x}+4x.\] 56 | 即 57 | \[(p+2x)\biggl(\frac{\diff p}{\diff x}+1\biggr)=0.\] 58 | 59 | 当 $p+2x=0$ 时, 代入原方程得特解 $y=-x^2$; 60 | 61 | 当 $\frac{\diff p}{\diff x}=-1$ 时, $p=-x+C$, 62 | 代入原方程得通解 $y=-\frac{1}{2}x^2+Cx+\frac{1}{2}C^2$. 63 | 64 | (2) 将原方程求导得 65 | \[p=x\ln x\frac{\diff p}{\diff x}+p(\ln x+1)+2xp\left(p+x\frac{\diff p}{\diff x}\right).\] 66 | 即 67 | \[\left(x\frac{\diff p}{\diff x}+p\right)(\ln x+2xp)=0.\] 68 | 69 | 当 $x\frac{\diff p}{\diff x}+p=0$ 时, $p=\frac{C}{x}$, 代入原方程得通解 $y=C\ln x+C^2$; 70 | 71 | 当 $\ln x+2xp=0$ 时, $p=-\frac{\ln x}{2x}$, 代入原方程得特解 $y=-\frac{1}{4}(\ln x)^2$. 72 | 73 | (3) 当 $p=0$ 时得特解 $y=k\pi$ ($k\in\mathbb{Z}$); 74 | 75 | 当 $p\neq 0$ 时, 将原方程变形为 76 | \[x=\frac{\tan y}{p}+\frac{1}{2}p^2\cos^2y.\] 77 | 在上式中关于 $y$ 求导得 78 | \[\frac{1}{p}=\frac{1}{p^2}\left(\sec^2y\cdot p-\tan y\cdot\frac{\diff p}{\diff y}\right)+p\frac{\diff p}{\diff y}\cos^2y-p^2\sin y\cos y.\] 79 | 即 80 | \[\left(\frac{\diff p}{\diff y}-p\tan y\right)\left(p^3\cos^2y-\tan y\right)=0.\] 81 | 82 | 当 $\frac{\diff p}{\diff y}-p\tan y=0$ 时, 83 | 解得 $p=\frac{1}{C\cos y}$ $(C\neq 0)$, 代入原方程得通解 $x=C\sin y+\frac{1}{2C^2}$; 84 | 85 | 当 $p^3\cos^2y-\tan y=0$ 时, $p=\frac{\tan^{\frac{1}{3}}y}{\cos^{\frac{2}{3}}y}$, 86 | 代入原方程得特解 $x=\frac{3}{2}\sin^{\frac{2}{3}}y$. 87 | \end{proof} 88 | 89 | 90 | 91 | \begin{exercise} 92 | 用参数法求解下列微分方程: 93 | \begin{enumerate}[(1)] 94 | \item $\displaystyle 2y^2+5\left(\frac{\diff y}{\diff x}\right)^2=4$; 95 | \item $\displaystyle x^2-3\left(\frac{\diff y}{\diff x}\right)^2=1$; 96 | \item $\displaystyle\left(\frac{\diff y}{\diff x}\right)^2+y-x^2=0$; 97 | \item $\displaystyle x^3+\left(\frac{\diff y}{\diff x}\right)^3=4x\frac{\diff y}{\diff x}$. 98 | \end{enumerate} 99 | \end{exercise} 100 | 101 | \begin{proof} 102 | (1) 令 $y=\sqrt{2}\sin t,\frac{\diff y}{\diff x}=\frac{2}{\sqrt{5}}\cos t$, 则 103 | 104 | 当 $\diff y=0$ 时, 得特解 $y=\pm\sqrt{2}$; 105 | 106 | 当 $\diff y\neq 0$时, $\diff x=\frac{\diff x}{\diff y}\diff y=\frac{\sqrt{5}}{2\cos t}\sqrt{2}\cos t\diff t=\sqrt{\frac{5}{2}}\diff t\Rightarrow x=\sqrt{\frac{5}{2}}t+C$, 107 | 故通解为 $y=\sqrt{2}\sin\left(\sqrt{\frac{2}{5}}(x-C)\right)$. 108 | 109 | (2) 令 $x=\sec t,\frac{\diff y}{\diff x}=\frac{1}{\sqrt{3}}\tan t$, 则 110 | \[\diff y=\frac{\diff y}{\diff x}\diff x=\frac{1}{\sqrt{3}}\tan^2t\sec t\diff t.\] 111 | 积分得 112 | \[y=\frac{1}{2\sqrt{3}}(\sec t\tan t-\ln|\sec t+\tan t|)+C.\] 113 | 故通解为 114 | \[\begin{cases} 115 | x = \sec t, \\ 116 | y = \frac{1}{2\sqrt{3}}(\sec t\tan t-\ln|\sec t+\tan t|)+C. 117 | \end{cases}\] 118 | 119 | 本题也可以利用恒等式: $\cosh^2t-\sinh^2t=1$, 120 | 得到另外一种通解表达式: $x=\cosh t,y=\frac{1}{4\sqrt{3}}(\sinh2t-2t)+C$. 121 | 122 | (3) 将原方程求导得 123 | \[2p\frac{\diff p}{\diff x}+p-2x=0\Rightarrow \frac{\diff p}{\diff x}=\frac{x}{p}-\frac{1}{2}.\] 124 | 令 $u=\frac{p}{x}\neq 0$, 则 125 | \[\frac{\diff p}{\diff x}=u+x\frac{\diff u}{\diff x}=\frac{1}{u}-\frac{1}{2}\Rightarrow x\frac{\diff u}{\diff x}+\frac{2u^2+u-2}{2u}=0.\] 126 | 127 | \textbullet 当 $2u^2+u-2=0$ 即 $u=\frac{-1\pm\sqrt{17}}{4}$ 时, 128 | 若 $u=\frac{-1+\sqrt{17}}{4}$, 129 | 则 $p=\frac{\diff y}{\diff x}=\frac{-1+\sqrt{17}}{4}x\Rightarrow y=\frac{-1+\sqrt{17}}{8}x^2+C$, 130 | 将 $p=\frac{-1+\sqrt{17}}{4}x$ 直接代入原方程得 $y=\frac{-1+\sqrt{17}}{8}x^2$, 131 | 故原方程有特解 $y=\frac{-1+\sqrt{17}}{8}x^2$, 同理可得到另外一个特解 $y=\frac{-1-\sqrt{17}}{8}x^2$; 132 | 133 | \textbullet 当 $2u^2+u-2\neq 0$ 时, $\frac{2u}{2u^2+u-2}\diff u+\frac{1}{x}\diff x=0$, 134 | 下面对该变量分离的方程进行不定积分: 135 | \begin{align*} 136 | & \int\frac{2u}{2u^2+u-2}\diff u+\int\frac{1}{x}\diff x=0 \\ 137 | \Longrightarrow{} & \frac{1}{2}\ln|2u^2+u-2|-\frac{1}{2}\int\frac{1}{2\left(x+\frac{1}{4}\right)^2-\frac{17}{8}}\diff u+\ln|x|=C \\ 138 | \Longrightarrow{} & \frac{1}{2}\ln|2u^2+u-2|-\frac{1}{4}\int\frac{1}{\left(x+\frac{1}{4}\right)^2-\frac{17}{16}}\diff u+\ln|x|=C \\ 139 | \Longrightarrow{} & \frac{1}{2}\ln|2u^2+u-2|+\frac{1}{2\sqrt{17}}\ln\left|\frac{u+\frac{1+\sqrt{17}}{4}}{u+\frac{1-\sqrt{17}}{4}}\right|+\ln|x|=C \\ 140 | \Longrightarrow{} & \frac{\sqrt{17}}{4}\ln|2p^2-2x^2+px|+\frac{1}{4}\ln\left|\frac{p+\frac{1+\sqrt{17}}{4}x}{p+\frac{1-\sqrt{17}}{4}x}\right|=C_1 \\ 141 | \Longrightarrow{} & (2(p-\alpha x)(p-\beta x))^{\frac{\sqrt{17}}{4}}\left(\frac{p-\beta x}{p-\alpha x}\right)^{\frac{1}{4}}=C_2\neq 0 \\ 142 | \Longrightarrow{} & (p-\alpha x)^{\alpha}=C_3(p-\beta x)^{\beta},C_3\neq 0, 143 | \end{align*} 144 | 其中 $\alpha=\frac{\sqrt{17}-1}{4}$, $\beta=\frac{-\sqrt{17}-1}{4}$. 145 | 146 | 综上所述, 原方程有通解 $(p-\alpha x)^{\alpha}=C(p-\beta x)^{\beta}$ $(C\neq 0)$ 147 | 以及两个特解 $y_1=\frac{1}{2}\alpha x^2,y_2=\frac{1}{2}\beta x^2$. 148 | 149 | (4) 令 $p=tx$, 则 $x^3(1+t^3)=4tx^2\Rightarrow x=\frac{4t}{1+t^3},p=\frac{4t^2}{1+t^3}$, 故 150 | \[\diff y=p\diff x=\frac{4t^2}{1+t^3}\frac{4(1+t^3)-4t\cdot3t^2}{(1+t^3)^2}\diff t 151 | = \frac{4t^2(4-8t^2)}{(1+t^3)^3}\diff t.\] 152 | 积分得 153 | \[y=\int\frac{4t^2(4-8t^2)}{(1+t^3)^3}\diff t=\frac{32}{3}\frac{1}{1+t^3}-\frac{8}{(1+t^3)^2}+C.\] 154 | 故通解为 155 | \[\begin{cases} 156 | x=\frac{4t}{1+t^3}, \\ 157 | y=\frac{32}{3}\frac{1}{1+t^3}-\frac{8}{(1+t^3)^2}+C. 158 | \end{cases}\qedhere\] 159 | \end{proof} 160 | 161 | 162 | 163 | \section{奇解} 164 | 165 | 166 | 167 | \begin{exercise} 168 | 利用 $p$-判别式求下列微分方程的奇解: 169 | \begin{enumerate}[(1)] 170 | \item $\displaystyle y=x\frac{\diff y}{\diff x}+\left(\frac{\diff y}{\diff x}\right)^2$; 171 | \item $\displaystyle y=2x\frac{\diff y}{\diff x}+\left(\frac{\diff y}{\diff x}\right)^2$; 172 | \item $\displaystyle (y-1)^2\left(\frac{\diff y}{\diff x}\right)^2=\frac{4}{9}y$; 173 | \end{enumerate} 174 | \end{exercise} 175 | 176 | \begin{solution} 177 | (1) 令 $F(x,y,p)=y-xp-p^2$, 则 $p$-判别式为 178 | $y-xp-p^2=0,-x-2p=0\Rightarrow y=-\frac{1}{4}x^2$, 经验证 $y=-\frac{1}{4}x^2$ 是原方程的解, 又 179 | \[F_y'\left(x,-\frac{1}{4}x^2,-\frac{1}{2}x\right)=1,\: 180 | F_{pp}''\left(x,-\frac{1}{4}x^2,-\frac{1}{2}x\right)=-2,\: 181 | F_p'\left(x,-\frac{1}{4}x^2,-\frac{1}{2}x\right)=0.\] 182 | 故 $y=-\frac{1}{4}x^2$ 是奇解. 183 | 184 | (2) 令 $F(x,y,p)=y-2xp-p^2$, 则$p$-判别式为$y-2xp-p^2=0,-2x-2p=0\Rightarrow y=-x^2$, 185 | 但是 $y=-x^2$ 不是原方程的解更不是奇解. 186 | 187 | (3) 令 $F(x,y,p)=(y-1)^2p^2-\frac{4}{9}y$, 188 | 则 $p$-判别式为 $(y-1)^2p^2-\frac{4}{9}y=0,2(y-1)^2p=0\Rightarrow y=0$, 189 | 经验证 $y=0$ 是原方程的解, 又 190 | \[F_y'(x,0,0)=-\frac{4}{9},\: F_{pp}''(x,0,0)=2,\: F_p'(x,0,0)=0,\] 191 | 故 $y=0$ 是原方程的奇解. 192 | \end{solution} 193 | 194 | 195 | 196 | \begin{exercise} 197 | 举例说明, 在定理 4.2 的条件 (4.28) 中的两个不等式是缺一不可的. 198 | \end{exercise} 199 | 200 | \begin{solution} 201 | 分别考虑方程 $\displaystyle\left(\frac{\diff y}{\diff x}\right)^2-y^2=0$ 202 | 与 $\displaystyle\sin\left(y\frac{\diff y}{\diff x}\right)=y$. 203 | \end{solution} 204 | 205 | 206 | 207 | \begin{exercise} 208 | 研究下面的例子, 说明定理 4.2 的条件 (4.29) 是不可缺少的: 209 | \[y=2x+y'-\frac{1}{3}(y')^3.\] 210 | \end{exercise} 211 | 212 | \begin{solution} 213 | $p$-判别式为: $y=2x+p-\frac{1}{3}p^3$, $0 = 1-p^2$, 解得 $y = 2x\pm\frac{2}{3}$, 214 | 经检验 $y=2x+\frac{2}{3}$ 不是原方程的解. 而 $y=2x-\frac{2}{3}$ 是原方程的解, 但不是奇解. 215 | 216 | 下面证明 $y = 2x - \frac23$ 不是奇解. 217 | 218 | 令 $F(x,y,p) = y-2x+\frac{1}{3}p^3-p$, 则 219 | \[F_y'\left(x,2x-\frac{2}{3},2\right)=1,\: 220 | F_{pp}''\left(x,2x-\frac{2}{3},2\right)=4,\: 221 | F_p'\left(x,2x-\frac{2}{3},2\right)=3\neq 0.\] 222 | 故条件 $F_p'(x,\varphi(x),\varphi'(x))=0$ 不可缺少. 223 | \end{solution} 224 | 225 | 226 | 227 | \section{包络} 228 | 229 | 230 | 231 | \begin{exercise} 232 | 试求克莱罗方程的通解及其包络. 233 | \end{exercise} 234 | 235 | \begin{solution} 236 | 克莱罗方程为: $y=xp+f(p)\left(p=\frac{\diff y}{\diff x}\right)$, 其中 $f''(p)\neq0$. 237 | \[y=xp+f(p)\Rightarrow(x+f'(p))\frac{\diff p}{\diff x}=0.\] 238 | 由 $\frac{\diff p}{\diff x}=0 $ 即 $p=C$ 得通解 $y=Cx+f(C)$, $C$ 判别式为 239 | \[\begin{cases} 240 | y=Cx+f(C), \\ 241 | x+f'(C)=0 242 | \end{cases}\Rightarrow 243 | \begin{cases} 244 | x=-f'(C)=\varphi(C), \\ 245 | y=-Cf'(C)+f(C)=\psi(C). 246 | \end{cases}(*)\] 247 | 令 $V(x,y,C)=Cx+f(C)-y$, 则 $V_x'(\varphi(C),\psi(C),C)=C,V_y'(\varphi(C),\psi(C),C)=-1$, 248 | 故 $(V_x',V_y')\neq(0,0)$, 又$(\varphi'(C),\psi'(C))=(-f''(C),-Cf''(C))\neq(0,0)$, 249 | 故 $(*)$ 是曲线族 $y=Cx+f(C)$ 的一支包络. 250 | \end{solution} 251 | 252 | 253 | 254 | \begin{exercise} 255 | 试求一微分方程, 使它有奇解为$y=\sin x$. 256 | \end{exercise} 257 | 258 | \begin{solution} 259 | 考虑克莱罗方程 $y=xp+f(p)$, 将 $y=\sin x$ 代入得 260 | \[\sin x=x\cos x+f(\cos x).\] 261 | 令 $\cos x=p$, 得 262 | \[\sqrt{1-p^2}=p\arccos p+f(p).\] 263 | 故 264 | \[f(p)=-p\arccos p+\sqrt{1-p^2}.\] 265 | 容易验证 $y=\sin x$ 是方程 $y=xp-p\arccos p+\sqrt{1-p^2}$ 的奇解. 266 | \end{solution} -------------------------------------------------------------------------------- /chapter03.tex: -------------------------------------------------------------------------------- 1 | \chapter{存在和唯一性定理} 2 | 3 | 4 | 5 | \section{皮卡存在和唯一性定理} 6 | 7 | 8 | 9 | \begin{exercise} 10 | 利用右端函数的性质讨论下列微分方程满足初值条件 $y(0)=0$ 的解的唯一性问题: 11 | \begin{enumerate}[(1)] 12 | \item $\displaystyle\frac{\diff y}{\diff x}=|y|^{\alpha}$ (常数 $\alpha>0$); 13 | \item $\displaystyle\frac{\diff y}{\diff x}= 14 | \begin{cases} 15 | 0, & \text{当\ } y=0, \\ 16 | y\ln|y|, & \text{当\ } y\neq 0. 17 | \end{cases}$ 18 | \end{enumerate} 19 | \end{exercise} 20 | 21 | \begin{solution} 22 | (1) 显然 $y=0$ 是满足初值条件的解, 当 $0<\alpha<1$ 时, 由下式 23 | \[\int_0^y\frac{\diff y}{|y|^{\alpha}}=x\] 24 | 确定的函数 $\varPhi(x,y)$ 是满足初值条件的解, 且当 $x\neq 0$ 时, $y\neq 0$, 故此时解不唯一. 25 | 26 | 当 $\alpha\geq 1$ 时, 满足初值条件的解是唯一的, 用反证法, 假设还存在另一解 $y=y(x),y(0)=0$, 27 | 则必存在 $x_0$ 和 $\epsilon$ 使得$y(x_0)=0$, 且当 $x_0x_0$ 使得 $y_1(x_1)\neq y_2(x_1)$, 不妨设 $y_1(x_1)>y_2(x_1)$, 记 77 | \[\bar{x}=\sup\{x_0\leq x0$, 又 80 | \[r'(x)=y_1'(x)-y_2'(x)=f(x,y_1(x))-f(x,y_2(x))<0,\bar{x}0$, 使得 104 | \[|f_n(x)|\leq M,\quad\forall x\in(a,b),\forall n=1,2,\cdots\] 105 | \item (等度连续) 对任意 $\epsilon>0$, 存在 $\delta=\delta(\epsilon)>0$, 106 | 使得 $\forall x_1,x_2\in(a,b)$, 当 $|x_1-x_2|<\delta$ 时,有 107 | \[|f_n(x_1)-f_n(x_2)|<\epsilon,\quad\forall n=1,2,\cdots\] 108 | \end{enumerate} 109 | 由 Cauchy 收敛原理知 $\lim_{x\to a+}f_n(x)$ 存在等价于 110 | \[\forall\epsilon>0,\exists\delta>0, 111 | \forall x_1,x_2\in\{x\mid 0n. 136 | \end{cases}\] 137 | 容易验证 $\{f_n(x)\}$ 是一致有界且等度连续的, 138 | 下面证明 $\{f_n(x)\}$ 没有一致收敛的子序列 $\{f_{n_k}(x)\}$, 139 | 首先若 $\{f_{n_k}(x)\}$ 一致收敛, 则必收敛到 $0$,但是 140 | \[\lim_{k\to\infty}d(f_{n_k},0)=\lim_{k\to\infty}\sup_{x\in[0,+\infty)}|f_{n_k}(x)-0|=1\neq 0.\] 141 | 矛盾, 故 $\{f_n(x)\}$ 没有一致收敛的子序列. 142 | \end{solution} 143 | 144 | 145 | 146 | \begin{exercise} 147 | 我们知道: 皮卡序列满足 Ascoli 引理的条件.试问: 能用皮卡序列来证明佩亚诺的存在定理吗?说明理由. 148 | \end{exercise} 149 | 150 | \begin{solution} 151 | 不能. 因为在定义皮卡序列的积分式中: 152 | \[y_{n+1}(x)=y_0+\int_{x_0}^xf(x,y_n(x))\diff x.\] 153 | $y_{n+1}(x)$ 通过 $y_n(x)$ 表示出来, 一旦限制在子序列上, 这种表示法就失效了. 154 | \end{solution} 155 | 156 | 157 | 158 | \begin{exercise} 159 | 对于与初值问题 $(E)$ 等价的积分方程 160 | \[y(x)=y_0+\int_{x_0}^xf(x,y(x))\diff x.\] 161 | 在区间 $I=[x_0,x_0+h]$ 上 (其中正数 $h$ 的意义同定理 3.3) 162 | 构造序列 $y_n(x)$ 如下: 163 | 任给正整数 $n$, 令 $x_k=x_0+kd_n$, 其中 $d_n=h/n$, $k=0,1,\cdots,n$. 则分点 164 | \[x_0,x_1,x_2,\cdots,x_n(=x_0+h)\] 165 | 把区间 $I$ 分成 $n$ 等份. 我们从 $[x_0,x_1]$ 到 $[x_1,x_2]$, 166 | 再从 $[x_1,x_2]$ 到 $[x_2,x_3],\cdots$, 167 | 最后从 $[x_{n-2},x_{n-1}]$ 到 $[x_{n-1},x_0+h]$ 用递推法定义下面的函数: 168 | \[y_n(x)=\begin{cases} 169 | y_0, & \text{当\ } x\in[x_0,x_1]; \\ 170 | y_0 + \int_{x_0}^{x-d_n}f(s,y_n(s))\diff s, & \text{当\ } x\in[x_1,x_0+h]. 171 | \end{cases}\] 172 | 称序列 173 | \[y_1(x),y_2(x),\cdots,y_n(x),\cdots\quad (x\in I)\] 174 | 为 Tonelli 序列, 试用 Tonelli 序列和 Ascoli 引理证明佩亚诺存在定理. 175 | \end{exercise} 176 | 177 | \begin{proof} 178 | 由定义式可知 $y_n(x)$ 是通过如下递推得到的: 179 | 180 | 当 $x\in[x_0,x_1]$ 时, 181 | \[y_n(x)=y_0.\] 182 | 183 | 当 $x\in[x_1,x_2]$ 时, 184 | \[y_n(x)=y_0+\int_{x_0}^{x-d_n}f(s,y_n(s))\diff s=y_0+\int_{x_0}^{x-d_n}f(s,y_0)\diff s.\] 185 | 186 | 当 $x\in[x_2,x_3]$ 时, 187 | \[y_n(x)=y_0+\int_{x_0}^{x-d_n}f(s,y_n(s))\diff s=y_0+\int_{x_0}^{x_1}f(s,y_0)\diff s+\int_{x_1}^{x-d_n}f(s,y_n(s))\diff s,\] 188 | 其中上式中最右侧积分式里的 $y_n(s)$ 为当 $x\in[x_1,x_2]$ 时已经得到的 $y_n(x)$. 189 | 190 | 这样不断地递推下去, 即可得到 $y_n(x)$ 在区间 $[x_0,x_0+h]$ 上面的表达式, 191 | 容易验证 $\{y_n(x)\}$ 在 $[x_0,x_0+h]$ 上连续, 且由 $y_n(x)-y_0=0$, $x\in[x_0,x_1]$ 和 192 | \[|y_n(x)-y_0|=\biggl|\int_{x_0}^{x-d_n}f(s,y_n(s))\diff s\biggr|\leq M(x-d_n-x_0), 193 | \quad x_1\leq x\leq x_0+h\] 194 | 知 $\{(x,y_n(x))\mid x\in[x_0,x_0+h]\}\subset R$, 从而 $(y_n(x))_{n\geq 1}$ 一致有界. 195 | 196 | 对于 $\forall x,\tilde{x}\in[x_0,x_0+h]$, 不妨设 $x<\tilde{x}$, 197 | 则 $\max\{\tilde{x}-d_n,x_0\}\geq\max\{x-d_n,x_0\}$, 由定义知 198 | \[y_n(x)=y_0+\int_{x_0}^{\max\{x-d_n,x_0\}}f(s,y_n(s))\diff s.\] 199 | \[y_n(\tilde{x})=y_0+\int_{x_0}^{\max\{\tilde{x}-d_n,x_0\}}f(s,y_n(s))\diff s.\] 200 | 故 201 | \[\begin{split} 202 | |y_n(\tilde{x})-y_n(x)| 203 | & = \left|\int_{\max\{x-d_n,x_0\}}^{\max\{\tilde{x}-d_n,x_0\}}f(s,y_n(s))\diff s\right|\\ 204 | & \leq M(\max\{\tilde{x}-d_n,x_0\}-\max\{x-d_n,x_0\})\leq M(\tilde{x}-x). 205 | \end{split}\] 206 | 因此 $\{y_n(x)\}$ 在区间 $[x_0,x_0+h]$ 上等度连续, 207 | 由 Ascoli 定理知 $(y_n(x))_{n\geq 1}$ 有一致收敛的子序列 208 | $\bigl(y_{n_k}(x)\bigr)_{k\geq 1}$, 记 $y_{n_k}(x)\rightrightarrows\phi(x)$, 在下面定义式中 209 | \[y_{n_k}(x)=y_0+\int_{x_0}^{\max\left\{x-d_{n_k},x_0\right\}}f(s,y_{n_k}(s))\diff s,\] 210 | 取极限 $k\to\infty$, 并注意到 $\max\left\{x-d_{n_k},x_0\right\}\to x$, 即得 211 | \[\phi(x)=y_0+\int_{x_0}^xf(s,\phi(s))\diff s.\] 212 | 故 $\phi(x)$ 是初值问题 $(E)$ 的一个解. 213 | \end{proof} 214 | 215 | 216 | 217 | \section{解的延伸} 218 | 219 | 220 | 221 | \begin{exercise} 222 | 利用定理 3.5 证明: 线性微分方程 223 | \[\frac{\diff y}{\diff x}=a(x)y+b(x)\quad (x\in I)\] 224 | 的每一个解 $y=y(x)$ 的(最大)存在区间为 $I$, 这里假设 $a(x)$ 和 $b(x)$ 在区间 $I$ 上是连续的. 225 | \end{exercise} 226 | 227 | \begin{proof} 228 | 显然 229 | \[|a(x)y+b(x)|\leq|a(x)||y|+|b(x)|.\] 230 | 令 $A(x)=|a(x)|\geq 0$, $B(x)=|b(x)|\geq 0$, 231 | 则 $A(x)$ 和 $B(x)$ 都是 $I$ 上的连续函数, 由定理 3.5 知每一个解的最大存在区间为 $I$. 232 | \end{proof} 233 | 234 | 235 | 236 | \begin{exercise} 237 | 讨论下列微分方程解的存在区间: 238 | \begin{enumerate}[(1)] 239 | \item $\displaystyle\frac{\diff y}{\diff x}=\frac{1}{x^2+y^2}$; 240 | \item $\displaystyle\frac{\diff y}{\diff x}=y(y-1)$; 241 | \item $\displaystyle\frac{\diff y}{\diff x}=y\sin(xy)$; 242 | \item $\displaystyle\frac{\diff y}{\diff x}=1+y^2$. 243 | \end{enumerate} 244 | \end{exercise} 245 | 246 | \begin{solution} 247 | (1) 因为 $\frac{1}{x^2+y^2}$ 在区域 $\mathbb{R}^2\backslash\setminus\{0\}$ 上连续, 248 | 故由解的延伸定理知任意积分曲线必延伸到 $\mathbb{R}^2\setminus\{0\}$ 的边界. 249 | 设 $y=y(x)$, $x\in J$ 为一个饱和解, 则 250 | \[\frac{\diff y(x)}{\diff x}=\frac{1}{x^2+y^2(x)}>0.\] 251 | 故存在反函数 $x=x(y)$, 且 $x(y)$ 满足 252 | \[\frac{\diff x(y)}{\diff y}=x^2(y)+y^2.\] 253 | 由教材例 1 知 $x=x(y)$ 的存在区间有限, 不妨记为 $(c,d)$, 254 | 则当 $y\to c+$ 或 $y\to d-$时, $x(y)\to\infty$, 也就说明 $y(x)$ 有界, 255 | 故其存在区间为 $(-\infty,+\infty)$ 或 $(-\infty,0)$ 或 $(0,+\infty)$. 256 | 257 | 注: 对照教材例 1 可以证明一般结论: 微分方程 258 | \[\frac{\diff y}{\diff x}=af(x)+by^2\quad (f(x)\uparrow>0,ab>0)\] 259 | 任一解的存在区间都是有界的. 260 | 261 | (2)这是变量分离的方程, 容易解得方程的通解为 262 | $\displaystyle y(x)=\frac{1}{1-C\e^x}$ $(C\in\mathbb{R})$, 另外有特解 $y=0$. 263 | 264 | 当 $C<0$ 时, $00$ 时, 分母有零点 $x_c=-\ln C$, 当 $x\in(-\infty,x_c)$ 时, $y(x)>0$ 单调增, 270 | $x\to-\infty$ 时, $y(x)\to1$, $x\to x_c$ 时, $y(x)\to+\infty$; 271 | 当 $x\in(x_c,+\infty)$ 时, $y(x)<0$ 单调增, $x\to x_c$时, 272 | $y(x)\to-\infty$, $x\to+\infty$ 时, $y(x)\to0$. 273 | 274 | 综上, 解的存在区间为 $(-\infty,+\infty)$ 或 $(-\infty,-\ln C)$ 或 $(-\ln C,+\infty)$. 275 | 276 | (3) 因为 $y\sin(xy)$ 在 $\mathbb{R}^2$ 上连续, 277 | 且 $|y\sin(xy)|\leq|y|$, 故由定理 3.5 知解的存在区间为 $(-\infty,+\infty)$. 278 | 279 | (4) 原方程的解为 $x=\arctan y+C$, 故解的存在区间为 $(C-\frac{\pi}{2},C+\frac{\pi}{2})$. 280 | \end{solution} 281 | 282 | 283 | 284 | \begin{exercise} 285 | 考虑对称形式的微分方程 $x\diff x+y\diff y=0$, 286 | 它的定义域为 $G=\{(x,y):x^2+y^2>0\}$. 则单位圆 $(x^2+y^2=1)$ 是一条积分曲线, 287 | 它在区域 $G$ 的内部; 它并没有延伸到 $G$的边界, 这一点是否与解的延伸定理相矛盾?为什么? 288 | \end{exercise} 289 | 290 | \begin{solution} 291 | $\frac{\diff y}{\diff x}=-\frac{x}{y}$, 因为 $-\frac{x}{y}$ 在区域 $G$ 上不连续,故不能运用延伸定理. 292 | \end{solution} 293 | 294 | 295 | 296 | \begin{exercise} 297 | 设初值问题 298 | \[(E):\quad \frac{\diff y}{\diff x}=(y^2-2y-3)\e^{(x+y)^2},\quad y(x_0)=y_0\] 299 | 的解的最大存在区间为: $a0$, 故解 $y=y(x)$ 严格单调增, 309 | 但不能与 $y=-1$ 相交, 故必有 $b=+\infty$. 310 | 311 | (2) $-13$ 时, $\frac{\diff y}{\diff x}>0$, 故解 $y=y(x)$ 严格单调增, 315 | 但不能与 $y=3$ 相交, 故必有 $a=-\infty$. 316 | 317 | (4) $y_0 = -1$ 或 $y_0 = 3$ 时, 显然 $a=-\infty$, $b=+\infty$. 318 | \end{proof} 319 | 320 | 321 | 322 | \begin{exercise} 323 | 设初值问题 324 | \[(E):\quad \frac{\diff y}{\diff x}=(x^2-y^2)f(x,y),\quad y(x_0)=y_0,\] 325 | 其中函数 $f(x,y)$ 在全平面连续且满足 $yf(x,y)>0$, 当 $y\neq 0$. 326 | 则对于任意的 $(x_0,y_0)$, 327 | 当 $x_0<0$ 和 $|y_0|$ 适当小时 $(E)$ 的解可延拓到 $-\infty0$, 当 $y\neq 0$ 可知 $f(x,0)=0$, 333 | 故 $y=0$ 为方程的特解. 当 $x_0<0$ 且 $|y_0|<|x_0|$ 时 (以 $y_0>0$ 为例), 334 | 解 $y=y(x)$ 在区域 $\{(x,y)\mid 0\leq y<-x,x<0\}$ 单调增加, 故可向左延伸至 $-\infty$, 335 | 向右穿过 $y=-x$ 后单调减, 必与 $y=x$ 相交, 336 | 穿过直线 $y=x$ 后单调增加且不能再次穿过 $y=x$, 故可向右延拓至 $+\infty$. 337 | \end{proof} 338 | 339 | 340 | 341 | \section{比较定理及其应用} 342 | 343 | 344 | 345 | \begin{exercise} 346 | 设初值问题 $(E)$, 矩形区域 $R$, 和正数 $h$ 的意义同定理 3.1. 347 | 试证在 $(E)$ 的最小解 $y=W(x)$ 和最大解 $y=Z(x)$ 之间充满了 $(E)$ 的其它解, 348 | 即任取一点 $(x_1,y_1)$, 其中 349 | \[|x_1-x_0|\leq h,\quad W(x_1)\leq y_1\leq Z(x_1),\] 350 | 则 $(E)$ 在 $|x-x_0|\leq h$ 上至少有一个解 $y=u(x)$ 满足: $u(x_1)=y_1$. 351 | \end{exercise} 352 | 353 | \begin{proof} 354 | 为证明简单以 $x_1\in(x_0,x_0+h]$ 为例, 由解的延伸定理知初值问题: 355 | \[(E_1):\quad \frac{\diff y}{\diff x}=f(x,y),\quad y(x_1)=y_1\] 356 | 的解 $y=\phi(x)$ 必与 $y=W(x)$ 或 $y=Z(x)$ 相交, 357 | 不妨设与 $y=W(x)$ 相交于点 $(\xi,W(\xi))$ 且两曲线在交点处相切, 令 358 | \[u(x)=\begin{cases} 359 | W(x), & x_0-h\leq x\leq\xi,\\ 360 | \phi(x), & \xi\varPhi_2(x)>\cdots>\varPhi_n(x)>\cdots>\phi(x),\quad x_0\leq x\leq c.\] 390 | 显然 $\{\varPhi_n(x)\}$ 在 $[x_0,c]$ 上一致有界, 且由 391 | \[|\varPhi_n(x_1)-\varPhi_n(x_2)|\leq 392 | \left|\int_{x_1}^{x_2}\left(F(x,\varPhi_n(x))+\frac{1}{n}\right)\diff x\right|\leq(M+1)|x_1-x_2|\] 393 | 知 $\{\varPhi_n(x)\}$ 等度连续, 故其有一致收敛的子列, 不妨设其一致收敛: 394 | \[\lim_{n\to\infty}\varPhi_n(x)=\varPhi_*(x).\] 395 | 在下式 396 | \[\varPhi_n(x)=y_0+\int_{x_0}^x\left(F(x,\varPhi_n(x))+\frac{1}{n}\right)\diff x\] 397 | 中令 $n\to\infty$, 得 398 | \[\varPhi_*(x)=y_0+\int_{x_0}^xF(x,\varPhi_*(x))\diff x.\] 399 | 故 $\varPhi_*(x)$ 是 $(E_2)$ 的解, 因此 400 | \[\phi(x)\leq\lim_{n\to\infty}\varPhi_n(x)=\varPhi_*(x)\leq\varPhi(x),\quad x_0\leq x\leq c.\] 401 | 同理可证 $\phi(x)\geq\varPhi(x),a\frac{\pi}{4}$. 443 | 444 | 取正数 $\lambda$ 使得 $0\ll\lambda < 1$, 初值问题 445 | \[(E_4)\colon\qquad \frac{\diff y}{\diff x} = \lambda^2 + (y+1)^2,\quad y(0) = 0\] 446 | 的解得右侧最大存在区间记为 $0\leq x<\tilde{C}(\lambda)$, 447 | 其中 $\tilde{C} = \frac{\pi}{2\lambda} - \frac{1}{\lambda} \arctan\frac{1}{\lambda}$. 448 | 计算得 $\tilde{C}(1)=1$ 且 449 | \[\frac{\diff\tilde{C}}{\diff\lambda}\bigg|_{\lambda=1} < 0,\] 450 | 因此当 $0\ll\lambda < 1$ 时 $\tilde{C}>\frac{\pi}{4}$. 451 | 对 $(*)$ 和 $(E_4)$ 运用比较定理即得 $\beta>\frac{\pi}{4}$. 452 | \end{proof} -------------------------------------------------------------------------------- /chapter07.tex: -------------------------------------------------------------------------------- 1 | \chapter{幂级数解法} 2 | 3 | 4 | \section{柯西定理} 5 | 6 | \begin{figure}[htb] 7 | \caption{柯西定理图示} 8 | \begin{tikzpicture} 9 | %\draw[help lines] (0,-2)grid(13,2); 10 | \draw (0,0)coordinate(a) 11 | (0,2)coordinate(b) 12 | (9.5,0.3)coordinate(c) 13 | (9.5,1.5)coordinate(d) 14 | (2.4,0.3)coordinate(e) 15 | (2.4,1.5)coordinate(f) 16 | (2.4,-0.4)coordinate(g) 17 | (3,-1)coordinate(h) 18 | (5.3,-0.2)coordinate(i) 19 | (7,-1)coordinate(j); 20 | \draw (a)node[anchor=west]{$(\widehat{E}):\frac{\diff y}{\diff x}=F(x,y),y(x_0)=y_0\xrightarrow{\scriptsize{形式解}}y=y_0+\sum_{n=1}^{\infty}\widehat{C}_n(x-x_0)^n$}; 21 | \draw (b)node[anchor=west]{$(E):\frac{\diff y}{\diff x}=f(x,y),y(x_0)=y_0\xrightarrow{\scriptsize{形式解}}y=y_0+\sum_{n=1}^{\infty}C_n(x-x_0)^n$}; 22 | \draw[<->,thick](c)--(d)node[pos=0.5,anchor=west]{$|C_n|\leq\widehat{C}_n$}; 23 | \draw[<->,thick](e)--(f)node[anchor=west,pos=0.5]{在$R_0$内$F(x,y)$是$f(x,y)$的优函数}; 24 | \draw [->,out=270,in=180,thick](g)to(h)node[anchor=west]{$\frac{M}{\left(1-\frac{x-x_0}{a}\right)\left(1-\frac{y-y_0}{b}\right)}$}; 25 | \draw [->,out=-10,in=180,thick](i)to node[anchor=south,pos=0.5,sloped]{\scriptsize{真实解}}(j); 26 | \draw (j)node[anchor=west]{$y=y_0+b-b\sqrt{1+\frac{2aM}{b}\ln\left(1-\frac{x-x_0}{a}\right)}(|x-x_0|<\rho)$}; 27 | \end{tikzpicture} 28 | \end{figure} 29 | 30 | 31 | 32 | \begin{exercise} 33 | 陈述并详细证明解析微分方程组的柯西定理. 34 | \end{exercise} 35 | 36 | 37 | \begin{solution} 38 | 柯西定理:考虑微分方程组的初值问题 39 | \[(E):\begin{cases} 40 | \frac{\diff y_1}{\diff x}=f_1(x,y_1,\cdots,y_n),y_1(0)=0\\ 41 | \cdots\\ 42 | \frac{\diff y_n}{\diff x}=f_n(x,y_1,\cdots,y_n),y_n(0)=0 43 | \end{cases}\] 44 | 其中函数 $f_k(k=1,2,\cdots,n)$ 在区域 45 | $R:|x|\leq\alpha,|y_1|\leq\beta,\cdots,|y_n|\leq\beta$ 内可以展成收敛的幂级数 46 | \[f_k(x,y_1,\cdots,y_n)=\sum_{i,j_1,\cdots,j_n=0}^{\infty}a_{i,j_1,\cdots,j_n}^{(k)}x^iy_1^{j_1}\cdots y_n^{j_n}.\] 47 | 则初值问题 $(E)$ 在邻域 $|x|<\rho$ 内有唯一的解析解 $y_k=y_k(x)$, 48 | 其中 $\displaystyle\rho=a\left(1-\e^{\frac{-b}{(n+1)aM}}\right),a<\alpha,b<\beta$. 49 | 50 | \begin{proof} 51 | 因为 $f_k(x,y_1,\cdots,y_n)$ 在区域 $R$ 上可以展成收敛的幂级数 52 | \[f_k(x,y_1,\cdots,y_n)=\sum_{i,j_1,\cdots,j_n=0}^{\infty}a_{i,j_1,\cdots,j_n}^{(k)}x^iy_1^{j_1}\cdots y_n^{j_n}.\] 53 | 故对任意的正数 $a<\alpha,b<\beta$, 正项级数 54 | \[\sum_{i,j_1,\cdots,j_n=0}^{\infty}a_{i,j_1,\cdots,j_n}^{(k)}a^ib^{j_1+\cdots+j_n}\] 55 | 收敛, 故其通项有界, 即存在 $M>0$ 使得 56 | \[\left|a_{i,j_1,\cdots,j_n}^{(k)}\right|a^ib^{j_1+\cdots+j_n}\leq M\Rightarrow\left|a_{i,j_1,\cdots,j_n}^{(k)}\right|\leq\frac{M}{a^ib^{j_1+\cdots+j_n}}.\cdots(*)\] 57 | 考虑下述函数在区域 $R_0:|x|,thick] (-4,0)--(4,0)node[anchor=west]{$x$}; 154 | \draw[->,thick] (0,-2.1)--(0,2.1)node[anchor=south]{$y$}; 155 | \draw (0,0)node[anchor=north west]{$O$}; 156 | \draw[color=red,smooth] plot(\x,{sin(\x r)}); 157 | \draw[color=blue,smooth]plot(\x,{sin(\x r)-1}); 158 | \draw[color=magenta,smooth]plot(\x,{sin(\x r)+1}); 159 | \end{tikzpicture} 160 | \caption{$y=\sin x+C$} 161 | \label{fig2.1} 162 | \end{figure} 163 | 164 | (2) $y=0$ 为特解, 当 $y\neq 0$ 时, 积分得 $y=C\e^{ax}$ $(C\neq 0)$, 165 | 积分曲线族如图~\ref{fig2.2} (以 $a>0$ 为例). 166 | \begin{figure} 167 | \centering 168 | \begin{tikzpicture}[domain=-2:1] 169 | \draw[->,thick] (-2.1,0)--(1.1,0)node[anchor=west]{$x$}; 170 | \draw[->,thick] (0,-0.2)--(0,5.5)node[anchor=south]{$y$}; 171 | \draw (0,0)node[anchor=north west]{$O$}; 172 | \draw[color=red,smooth]plot(\x,{0.5*exp(\x)}); 173 | \draw[color=blue,smooth]plot(\x,{exp(\x)}); 174 | \draw[color=magenta,smooth]plot(\x,{2*exp(\x)}); 175 | \end{tikzpicture} 176 | \caption{$y=C\e^{ax}$} 177 | \label{fig2.2} 178 | \end{figure} 179 | 180 | (3) $y=\pm 1$为特解, 当 $y\neq\pm 1$ 时, $\frac{\diff y}{1-y^2}=\diff x$, 181 | 积分得 $y=\frac{C\e^{2x}-1}{C\e^{2x}+1}(C\neq0)$, 182 | 当$C>0$时, 函数图像位于直线 $y=1$ 和 $y=-1$ 之间且单调递增; 183 | 当$C<0$时, 存在间断点 $x_0=\frac{1}{2}\ln\left(-\frac{1}{C}\right)$, 184 | $y=y(x)$ 在 $(-\infty,x_0)$ 上单调递减且当 $x\to x_0-$ 时 $y\to-\infty$, 185 | 在 $(x_0,\infty)$ 上单调递减且当 $x\to x_0+$ 时 $y\to+\infty$, 积分曲线族如图~\ref{fig2.3}. 186 | \begin{figure} 187 | \centering 188 | \begin{tikzpicture}[domain=-3.5:3.5] 189 | \draw[->,thick] (-4,0)--(4,0)node[anchor=west]{$x$}; 190 | \draw[->,thick] (0,-5)--(0,6)node[anchor=south]{$y$}; 191 | \draw(0,0)node[anchor=south east]{$O$}; 192 | \draw[color=yellow,smooth]plot(\x,1); 193 | \draw[color=yellow,smooth]plot(\x,-1); 194 | \draw(-2,1)node[anchor=south]{$y=1$}; 195 | \draw(2,-1)node[anchor=south]{$y=-1$}; 196 | \draw[dashed](0.3466,-5)--(0.3466,6); 197 | \draw(0.3466,-2)node[anchor=west]{$x=\frac{1}{2}\ln\left(-\frac{1}{C}\right)(C<0)$}; 198 | \draw[color=red,smooth]plot(\x,{(exp(2*\x)-1)/(exp(2*\x)+1)}); 199 | \draw[color=red,smooth]plot(\x,{(2*exp(2*\x)-1)/(2*exp(2*\x)+1)}); 200 | \draw[color=red,smooth]plot(\x,{(0.5*exp(2*\x)-1)/(0.5*exp(2*\x)+1)}); 201 | \draw[color=red,smooth]plot(\x,{(exp(2*\x)-1)/(exp(2*\x)+1)}); 202 | \draw[color=blue,smooth,domain=-3.5:0.15]plot(\x,{(-0.5*exp(2*\x)-1)/(-0.5*exp(2*\x)+1)}); 203 | \draw[color=blue,smooth,domain=0.55:3.5]plot(\x,{(-0.5*exp(2*\x)-1)/(-0.5*exp(2*\x)+1)}); 204 | \end{tikzpicture} 205 | \caption{$y=\pm 1$和$y=\frac{C\e^{2x}-1}{C\e^{2x}+1}$} 206 | \label{fig2.3} 207 | \end{figure} 208 | 209 | (4) 下述三种情形积分曲线族都易作出(略去). 210 | \begin{enumerate}[(i)] 211 | \item $n=\frac{1}{3}$ 时, 通解为 $\frac{3}{2}y^{\frac{2}{3}}=x+C$ $(x\geq-C)$, 特解为 $y=0$; 212 | \item $n=1$时, 通解为 $y=C\e^x$ $(C\in\mathbb{R})$; 213 | \item $n=2$时, 通解为 $y=\frac{1}{-x+C}$ $(C\in\mathbb{R})$, 特解为$y=0$. 214 | \end{enumerate} 215 | \end{solution} 216 | 217 | 218 | 219 | \begin{exercise} 220 | 跟踪: 设某 $A$ 从 $Oxy$ 平面上的原点出发, 沿 $x$ 轴正方向前进; 同时某 $B$ 从点 $(0,b)$ 开始跟踪 $A$, 221 | 即 $B$ 的运动方向永远指向 $A$ 并与 $A$ 保持等距 $b$. 试求 $B$ 的光滑运动轨迹. 222 | \end{exercise} 223 | 224 | \begin{solution} 225 | 设 $B$ 的运动轨迹方程为 $y=y(x)$, 记某时刻 $B$ 的位置为 $(x,y(x))$, 226 | 则此时 $A$ 相应的位置为 $\left(x-\frac{y(x)}{y'(x)},0\right)$, 由于 $A$ 与 $B$ 保持等距, 故 227 | \[\left(\frac{y}{y'}\right)^2+y^2=b^2\Rightarrow\diff x=-\frac{\sqrt{b^2-y^2}}{y}\diff y.\] 228 | 积分得 229 | \[\begin{split} 230 | x 231 | & = -\int\frac{\sqrt{b^2-y^2}}{y}\diff y(y=b\cos\theta) \\ 232 | & = -\int\frac{b\sin\theta}{b\cos\theta}(-b\sin\theta)\diff\theta \\ 233 | & = b\int(\sec\theta-\cos\theta)\diff\theta \\ 234 | & = b\ln|\sec\theta+\tan\theta|-b\sin\theta+C \\ 235 | & = b\ln\frac{b+\sqrt{b^2-y^2}}{y}-\sqrt{b^2-y^2}+C. 236 | \end{split}\] 237 | 由初值条件 $y(0)=b$ 得 $C=0$, 故 $B$ 的光滑运动轨迹方程为 238 | $x=b\ln\frac{b+\sqrt{b^2-y^2}}{y}-\sqrt{b^2-y^2}$. 239 | \end{solution} 240 | 241 | 242 | 243 | \begin{exercise} 244 | 设微分方程 245 | \[\frac{\diff y}{\diff x}=f(y),\] 246 | 其中 $f(y)$ 在 $y=a$ 的某邻域 (例如区间 $|y-a|\leq\varepsilon$) 内连续, 而且$f(y)=0$当且仅当$y=a$. 247 | 证明: 在直线$y=a$上的每一点, 上述方程的解是局部唯一的, 当且仅当瑕积分 248 | \[\left|\int_a^{a\pm\varepsilon}\frac{\diff y}{f(y)}\right|=\infty.\] 249 | \end{exercise} 250 | 251 | \begin{solution} 252 | ($\Leftarrow$) 显然, $y=a$ 是方程的一个解, 用反证法, 设 $y=y(x)$ 是方程的另一个解, 253 | 它与直线 $y=a$ 相交. 不妨设 $(x_0,a)$ 是它们的一个交点, 254 | 且存在区间 $I=(x_0,x_0+\delta)$ 或 $I=(x_0-\delta,x_0)$, 255 | 使得当 $x\in I$ 时, $y(x)\neq a$, 从而 256 | \[\frac{\diff y(x)}{f(y(x))}=\diff x,\quad x\in I.\] 257 | 积分得 258 | \[\int_a^y\frac{\diff y}{f(y)}=\int_{x_0}^x\frac{\diff y(x)}{f(y(x))} 259 | = \int_{x_0}^x\diff x = x-x_0<\infty.\] 260 | 矛盾. 261 | 262 | $(\Rightarrow)$ 用反证法, 设 $\left|\int_a^{a\pm\varepsilon}\frac{\diff y}{f(y)}\right|<+\infty$, 263 | 则由 264 | \[\int_a^y\frac{\diff y}{f(y)}=x-x_0\] 265 | 定义的函数是方程的解, 且通过点 $(x_0,a)$, 而 $y=a$ 也是过点 $(x_0,a)$ 的解, 矛盾. 266 | \end{solution} 267 | 268 | 269 | 270 | \begin{exercise} 271 | 利用上题结果, 作出下列微分方程积分曲线族的草图: 272 | \[(1)\ \frac{\diff y}{\diff x}=\sqrt{|y|};\quad 273 | (2)\ \frac{\diff y}{\diff x}= 274 | \begin{cases} 275 | y\ln|y|, & y\neq 0, \\ 276 | 0, & y=0. 277 | \end{cases}\] 278 | \end{exercise} 279 | 280 | \begin{solution} 281 | (1) 因为 $\int_0^{\pm\varepsilon}\frac{\diff y}{\sqrt{|y|}}$ 收敛, 故解不是局部唯一的, 微分方程的通解为 282 | \[y= 283 | \begin{cases} 284 | \frac{1}{4}(x+C)^2, & x\geq -C, \\ 285 | -\frac{1}{4}(x+C)^2, & x\leq -C. 286 | \end{cases}\] 287 | 另外特解为 $y=0$, 积分曲线族容易作出 (考虑 $x$ 轴外的点, 此例可以作为局部唯一不代表全局唯一的典型例子). 288 | 289 | (2) 因为 $\int_0^{\pm\varepsilon}\frac{\diff y}{y\ln|y|}$ 发散, 所以解是局部唯一的, 微分方程的通解为 290 | \[y=\pm\e^{C\e^x}\quad(C\in\mathbb{R}).\] 291 | 另外特解为 $y=0$, 积分曲线族容易作出. 292 | \end{solution} 293 | 294 | 295 | 296 | \section{一阶线性方程} 297 | 298 | 299 | 300 | \begin{exercise} 301 | 求解微分方程: 302 | \begin{enumerate}[(1)] 303 | \item $\displaystyle\frac{\diff y}{\diff x}+2y=x\e^{-x}$; 304 | \item $\displaystyle\frac{\diff y}{\diff x}+y\tan x=\sin(2x)$; 305 | \item $\displaystyle x\frac{\diff y}{\diff x}+2y=\sin x$, $y(\pi)=\frac{1}{\pi}$; 306 | \item $\displaystyle\frac{\diff y}{\diff x}-\frac{1}{1-x^2}y=1+x$, $y(0)=1$. 307 | \end{enumerate} 308 | \end{exercise} 309 | 310 | \begin{solution} 311 | (1) $p(x)=2$, $q(x)=x\e^{-x}$, 故 312 | \[y = \e^{-\int2\diff x}\left(C+\int x\e^{-x}\e^{\int2\diff x}\diff x\right) 313 | = C\e^{-2x}+(x-1)\e^{-x}.\] 314 | 315 | (2) $p(x)=\tan x$, $q(x)=\sin(2x)$, 故 316 | \[\begin{split} 317 | y & = \e^{-\int\tan x\diff x}\left(C+\int\sin(2x)\e^{\int\tan x\diff x}\diff x\right)\\ 318 | & = |\cos x|\left(C+\int\frac{\sin(2x)}{|\cos x|}\diff x\right)=C|\cos x|-2\cos^2x. 319 | \end{split}\] 320 | 321 | (3) $p(x)=\frac{2}{x}$, $q(x)=\frac{\sin x}{x}$, 故 322 | \[y = \e^{-\int\frac{2}{x}\diff x}\left(C+\int\frac{\sin x}{x} 323 | \e^{\int\frac{2}{x}\diff x}\diff x\right) = \frac{1}{x^2}(C+\sin x-x\cos x).\] 324 | 代入初值条件得 $C=0$, 故原方程的解为 325 | \[y=\frac{\sin x}{x^2}-\frac{\cos x}{x}.\] 326 | 327 | (4) $p(x) = -\frac{1}{1-x^2}$, $q(x)=1+x$, 故 328 | \begin{align*} 329 | y 330 | & = \e^{\int\frac{1}{1-x^2}\diff x}\left(C+\int(1+x)\e^{\int\frac{1}{x^2-1}\diff x}\diff x\right) \\ 331 | & = \left|\frac{x+1}{x-1}\right|^{\frac{1}{2}}\left(C+\int(1+x)\left|\frac{x-1}{x+1}\right|^{\frac{1}{2}}\diff x\right)\\ 332 | & = \begin{cases} 333 | \sqrt{\frac{x+1}{x-1}}\left(C+\int\sqrt{x^2-1}\diff x\right), & |x|>1, \\ 334 | \sqrt{\frac{x+1}{1-x}}\left(C+\int\sqrt{1-x^2}\diff x\right), & |x|<1, 335 | \end{cases} \\ 336 | & = \begin{cases} 337 | \sqrt{\frac{x+1}{x-1}}\left(C+\frac{1}{2}x\sqrt{x^2-1}-\frac{1}{2}\ln|x+\sqrt{x^2-1}|\right), 338 | & |x|>1, \\ 339 | \sqrt{\frac{x+1}{1-x}}\left(C+\frac{1}{2}\arcsin x+\frac{1}{2}x\sqrt{1-x^2}\right), 340 | & |x|<1. 341 | \end{cases}\qedhere 342 | \end{align*} 343 | \end{solution} 344 | 345 | 346 | 347 | \begin{exercise} 348 | 把下列微分方程化为线性微分方程: 349 | \begin{enumerate}[(1)] 350 | \item $\displaystyle\frac{\diff y}{\diff x}=\frac{x^2+y^2}{2y}$; 351 | \item $\displaystyle\frac{\diff y}{\diff x}=\frac{y}{x+y^2}$; 352 | \item $\displaystyle 3xy^2\frac{\diff y}{\diff x}+y^3+x^3=0$; 353 | \item $\displaystyle\frac{\diff y}{\diff x}=\frac{1}{\cos y}+x\tan y$. 354 | \end{enumerate} 355 | \end{exercise} 356 | 357 | \begin{solution} 358 | (1) 令 $u=y^2$, 则 $\frac{\diff u}{\diff x}=2y\frac{\diff y}{\diff x}=x^2+u$. 359 | 360 | (2) 将 $x$ 看作 $y$ 的函数, 即 $\frac{\diff x}{\diff y}=\frac{x}{y}+y$. 361 | 362 | (3) 令 $u=y^3$, 则 $\frac{\diff u}{\diff x}=3y^2\frac{\diff y}{\diff x}=-\frac{u}{x}-x^2$. 363 | 364 | (4) 原方程变形为 $\cos y\frac{\diff y}{\diff x}=1+x\sin y$, 365 | 令 $u=\sin y$, 即得 $\frac{\diff u}{\diff x}=1+xu$. 366 | \end{solution} 367 | 368 | 369 | 370 | \begin{exercise} 371 | 设 $y=\varphi(x)$ 满足微分不等式 372 | \[y'+a(x)y\leq 0\quad (x\geq 0).\] 373 | 求证: 374 | \[\varphi(x)\leq\varphi(0)\e^{-\int_0^x a(s)\diff s}\quad (x\geq 0).\] 375 | \end{exercise} 376 | 377 | \begin{proof} 378 | 在不等式两边同时乘以 $\e^{\int_0^xa(s)\diff s}$, 得 379 | \[\e^{\int_0^xa(s)\diff s}\frac{\diff y}{\diff x}+a(x)y\e^{\int_0^xa(s)\diff s}\leq 0,\] 380 | 即 381 | \[\frac{\diff\left(\varphi(x)\e^{\int_0^xa(s)\diff s}\right)}{\diff x}\leq 0.\] 382 | 将上式从 $0$ 到 $x$ 积分得 383 | \[\varphi(x)\e^{\int_0^xa(s)\diff s}-\varphi(0)\leq 0 384 | \Rightarrow\varphi(x)\leq\varphi(0)\e^{-\int_0^xa(s)\diff s}.\qedhere\] 385 | \end{proof} 386 | 387 | 388 | 389 | \begin{exercise} 390 | 用常数变易法求解非齐次线性方程 $\frac{\diff y}{\diff x}+p(x)y=q(x)$, 即: 391 | 假设方程有形如 $y=C\e^{-\int p(x)\diff x}$ 的解, 但其中的常数 $C$ 变易为 $x$ 的一个待定函数 $C(x)$. 392 | 然后将这种形式的解代入原方程, 再去确定 $C(x)$. 393 | \end{exercise} 394 | 395 | \begin{solution} 396 | 因为 397 | \[y=C(x)\e^{-\int p(x)\diff x},\] 398 | 所以 399 | \[\frac{\diff y}{\diff x}=C'(x)\e^{-\int p(x)\diff x}+C(x)(-p(x))\e^{-\int p(x)\diff x}.\] 400 | 即 401 | \[\frac{\diff y}{\diff x}+p(x)y=C'(x)\e^{-\int p(x)\diff x}.\] 402 | 故有 403 | \[C'(x)\e^{-\int p(x)\diff x} = q(x).\] 404 | 解之得 405 | \[C(x)=\int q(x)\e^{\int p(x)\diff x}\diff x+C.\] 406 | 代回即得原方程的解为 407 | \[y=\e^{-\int p(x)\diff x}\left(C+\int q(x)\e^{\int p(x)\diff x}\diff x\right).\qedhere\] 408 | \end{solution} 409 | 410 | 411 | 412 | \begin{exercise} 413 | 考虑方程 414 | \[\frac{\diff y}{\diff x}+p(x)y=q(x),\] 415 | 其中 $p(x)$ 和 $q(x)$ 都是以 $\omega>0$ 为周期的连续函数. 试证: 416 | 417 | (1) 若 $q(x)\equiv 0$, 则方程的任一非零解以 $\omega$ 为周期, 当且仅当函数 $p(x)$ 的平均值 418 | \[\bar{p}=\frac{1}{\omega}\int_0^{\omega}p(x)\diff x=0.\] 419 | 420 | (2) 若 $q(x)$ 不恒为零, 则方程有唯一的 $\omega$ 周期解, 当且仅当 $\bar{p}\neq0$. 试求出此解. 421 | \end{exercise} 422 | 423 | \begin{proof} 424 | (1)若$q(x)\equiv0$, 则方程的通解为 425 | \[y=C\e^{-\int_0^xp(s)\diff s}.\] 426 | 从而 427 | \[y(x)=y(x+\omega)\Leftrightarrow\int_0^xp(s)\diff s 428 | = \int_0^{x+\omega}p(s)\diff s 429 | \Leftrightarrow\int_0^{\omega}p(x)\diff x=0\Leftrightarrow\bar{p} = 0.\] 430 | 431 | (2) 若 $q(x)$ 不恒为零, 则方程的通解为 432 | \[y=\e^{-\int_0^xp(s)\diff s}\left(C+\int_0^x q(s)\e^{\int_0^s p(t)\diff t}\diff s\right).\] 433 | 下面求常数 $C$ 使得 $y(x)$ 为 $\omega$ 周期解, 即 434 | \[y(x)=y(x+\omega),\forall x\in\mathbb{R}.\] 435 | 可以断言若 $y(x)$ 是原方程的解且满足 $y(0)=y(\omega)$, 则 $y(x)$ 是原方程的 $\omega$ 周期解, 436 | 事实上, 若 $y(x)$ 是原方程的解, 则 $y(x+\omega)$ 也是原方程的解, 437 | 令 $u(x)=y(x+\omega)-y(x)$, 则 $u(x)$ 是相应齐次线性方程的解, 又因为 $u(0)=0$, 故 $u(x)\equiv0 $. 438 | 439 | 现将 $y(0)=y(\omega)$ 代入通解表达式得 440 | \[C = \e^{-\int_0^{\omega}p(s)\diff s}\left(C+\int_0^{\omega}q(s) 441 | \e^{\int_0^sp(t)\diff t}\diff s\right),\] 442 | 解得 443 | \[C = \frac{1}{\e^{\int_0^{\omega}p(s)\diff s}-1}\int_0^{\omega}q(s)\e^{\int_0^sp(t)\diff t}\diff s.\] 444 | 故方程有唯一的 $\omega$ 周期解当且仅当 $\int_0^{\omega}p(s)\diff s\neq0\Leftrightarrow\bar{p}\neq 0$, 445 | 下面求 $y(x)$ 的表达式: 446 | 447 | 因为 448 | \[\frac{\diff y(x)}{\diff x}+p(x)y(x)=q(x).\] 449 | 在等式两边同时乘以 $\e^{\int_0^x p(t)\diff t}$, 得 450 | \[\e^{\int_0^xp(t)\diff t}\frac{\diff y(x)}{\diff x}+\e^{\int_0^xp(t)\diff t}p(x)y(x) 451 | = \e^{\int_0^xp(t)\diff t}q(x).\] 452 | 即 453 | \[\frac{\diff}{\diff x}\left(y(x)\e^{\int_0^xp(t)\diff t}\right)=\e^{\int_0^xp(t)\diff t}q(x).\] 454 | 将上式从 $x$ 到 $x+\omega$ 积分, 利用 $y(x)$ 及 $p(x)$ 的周期性得 455 | \[\begin{split} 456 | y(x+\omega)\e^{\int_0^{x+\omega}p(t)\diff t}-y(x)\e^{\int_0^xp(t)\diff t} 457 | & = y(x)\e^{\int_0^xp(t)\diff t}\left(\e^{\int_x^{x+\omega}p(t)\diff t}-1\right) \\ 458 | & = y(x)\e^{\int_0^xp(t)\diff t}\left(\e^{\int_0^{\omega}p(t)\diff t}-1\right) \\ 459 | & = \int_x^{x+\omega}\e^{\int_0^sp(t)\diff t}q(s)\diff s 460 | \end{split}\] 461 | 从而 462 | \[y(x)=\frac{1}{\e^{\int_0^{\omega}p(t)\diff t}-1} 463 | \int_x^{x+\omega}\e^{\int_x^sp(t)\diff t}q(s)\diff s.\qedhere\] 464 | \end{proof} 465 | 466 | 467 | 468 | \begin{exercise} 469 | 设连续函数 $f(x)$ 在区间 $-\infty0$, 使得 $|f(x)|\leq M(\forall x\in\mathbb{R})$,故 485 | \[|y(x)|\leq M\int_{-\infty}^x\e^{s-x}\diff s = M.\] 486 | 说明 $y(x)$ 的确是有界解. 当 $f(x)$ 以 $\omega$ 为周期时, 有 487 | \begin{align*} 488 | y(x+\omega) 489 | & = \int_{-\infty}^{x+\omega}f(s)\e^{s-(x+\omega)}\diff s\quad(\text{Let }t=s-\omega) \\ 490 | & = \int_{-\infty}^xf(t)\e^{t-x}\diff t \\ 491 | & = y(x), 492 | \end{align*} 493 | 所以 $y(x)$ 也是以 $\omega$ 为周期的周期函数. 494 | \end{proof} 495 | 496 | 497 | 498 | \begin{exercise} 499 | 令集合 $H^0=\{f(x)\mid f\text{\ 是以\ }2\pi\text{\ 为周期的连续函数}\}$, 500 | 易知 $H^0$ 关于实数域构成一个线性空间. 对于任意 $f\in H^0$, 定义它的模 501 | \[\|f\| = \max_{0\leq x\leq 2\pi}|f(x)|.\] 502 | 证明 $H^0$ 是 Banach 空间, 利用下式 503 | \[y(x)=\frac{1}{\e^{2a\pi}-1}\int_x^{x+2\pi}\e^{-a(x-s)}f(s)\diff s.\] 504 | 可以在空间 $H^0$ 中定义一个变换 $\varphi$, 它把 $f$ 变到 $y$. 试证: $\varphi$ 是有界线性算子. 505 | \end{exercise} 506 | 507 | \begin{proof} 508 | 任取 $H^0$ 中的 Cauchy 序列 $(f_n)_{n\geq 1}$, 则 509 | 对任意 $\epsilon>0$, 存在 $N>0$, 使得当 $m,n>N$ 时有 510 | \[\|f_m-f_n\|<\epsilon,\] 511 | 即 512 | \[\max_{0\leq x\leq 2\pi}|f_m(x)-f_n(x)|<\epsilon.\] 513 | 故对于 $\forall x\in\mathbb{R}$, $\left(f_n(x)\right)_{n\geq 1}$ 是 $\mathbb{R}$ 中的 Cauchy 序列, 514 | 故收敛, 记为$f_n(x)\to f(x)$, 这样就得到了一个函数 $f:\mathbb{R}\to\mathbb{R}$, 515 | 容易验证 $f(x)$ 是 $2\pi$ 周期函数, 且 516 | \[\|f_n-f\|=\max_{0\leq x\leq2\pi}|f_n(x)-f(x)|\to 0\quad (n\to\infty).\] 517 | 故 $f_n\to f$ $(n\to\infty)$, 所以 $H^0$ 是 Banach 空间, 下面证明 $\varphi$ 是有界线性算子: 518 | 线性性显然, 有界性如下 519 | \begin{align*} 520 | \|\varphi(f)\| 521 | & = \max_{0\leq x\leq2\pi}\left|\frac{1}{\e^{2a\pi}-1} 522 | \int_x^{x+2\pi}\e^{-a(x-s)}f(s)\diff s\right| \\ 523 | & \leq \|f\|\cdot\left|\frac{1}{\e^{2a\pi}-1} 524 | \int_x^{x+2\pi}\e^{-a(x-s)}\diff s\right| = \frac{1}{a}\|f\|. 525 | \qedhere 526 | \end{align*} 527 | \end{proof} 528 | 529 | 530 | 531 | \section{初等变换法} 532 | 533 | 534 | 535 | \begin{exercise} 536 | 求解下列微分方程: 537 | \begin{enumerate}[(1)] 538 | \item $\displaystyle y'=\frac{2y-x}{2x-y}$; 539 | \item $\displaystyle y'=\frac{2y-x+5}{2x-y-4}$; 540 | \item $\displaystyle y'=\frac{x+2y+1}{2x+4y-1}$; 541 | \item $\displaystyle y'=x^3y^3-xy$. 542 | \end{enumerate} 543 | \end{exercise} 544 | 545 | \begin{solution} 546 | (1) 令 $u=\frac{y}{x}$, 则 547 | \[\frac{\diff y}{\diff x}=u+x\frac{\diff u}{\diff x}=\frac{2u-1}{2-u}.\] 548 | 当 $u\neq\pm 1$时, 上式化为 549 | \[\frac{2-u}{u^2-1}\diff u = \frac{1}{x}\diff x.\] 550 | 积分得 $y-x=C(x+y)^3$ $(C\neq 0)$, 551 | 当 $u=1$ 时, 特解 $y=x$ 可以令 $C=0$ 合并到通解之中, 当 $u=-1$ 时特解为 $x+y=0$. 552 | 综上, 原方程的通解为 $y-x=C(x+y)^3$ $(C\in\mathbb{R})$, 特解为 $x+y=0$. 553 | 554 | (2) 令 $\begin{cases}2\beta-\alpha+5=0\\2\alpha-\beta-4=0\end{cases}$, 555 | 解得 $\alpha=1$, $\beta=-2$, 556 | 故作变量代换 $\begin{cases}x=\xi+1\\y=\eta-2\end{cases}$, 则原方程化为 557 | \[\frac{\diff\eta}{\diff\xi}=\frac{2\eta-\xi}{2\xi-\eta}.\] 558 | 由 (1) 知上述方程的通解为 $\eta-\xi=C(\xi+\eta)^3$ $(C\in\mathbb{R})$, 特解为 $\xi+\eta=0$, 559 | 因此原方程的通解为 $y-x+3=C(x+y+1)^3$ $(C\in\mathbb{R})$, 特解为 $x+y+1=0$. 560 | 561 | (3) 令 $v=x+2y$, 则原方程化为 562 | \[\frac{\diff v}{\diff x}=1+2\frac{v+1}{2v-1}=\frac{4v+1}{2v-1}.\] 563 | 当 $4v+1\neq 0$ 时, 上述方程等价于 564 | \[\frac{2v-1}{4v+1}\diff v=\diff x.\] 565 | 积分并代回原变量得通解 $8y-4x-3\ln|4x+8y+1|=C$, 当 $4v+1=0$ 时, 得特解 $4x+8y+1=0$. 566 | 567 | (4) 此方程为伯努利方程, 当 $y\neq 0$ 时, 在方程两边同时乘以 $-2y^{-3}$, 得 568 | \[-2y^{-3}y'=-2x^3+2xy^{-2}.\] 569 | 令 $u=y^{-2}$, 则上式化为 570 | \[\frac{\diff u}{\diff x}=-2y^{-3}\frac{\diff y}{\diff x}=2xu-2x^3.\] 571 | 解得 572 | \[u(x)=\e^{\int2x\diff x}\left(C+\int-2x^3\e^{\int-2x\diff x}\diff x\right)=C\e^{x^2}+x^2+1.\] 573 | 代回变量即得原方程的通解为 $y^2=\left(C\e^{x^2}+x^2+1\right)^{-1}$, 另外 $y=0$ 为特解. 574 | \end{solution} 575 | 576 | 577 | 578 | \begin{exercise} 579 | 利用适当的变换,求解下列方程: 580 | \begin{enumerate}[(1)] 581 | \item $\displaystyle y'=\cos(x-y)$; 582 | \item $\displaystyle(3uv+v^2)\diff u+(u^2+uv)\diff v=0$; 583 | \item $\displaystyle(x^2+y^2+3)\frac{\diff y}{\diff x}=2x\left(2y-\frac{x^2}{y}\right)$; 584 | \item $\displaystyle\frac{\diff y}{\diff x}=\frac{2x^3+3xy^2-7x}{3x^2y+2y^3-8y}$. 585 | \end{enumerate} 586 | \end{exercise} 587 | 588 | \begin{solution} 589 | (1) 令 $u=x-y$, 则 590 | \[\frac{\diff u}{\diff x}=1-\cos u.\] 591 | 当 $1-\cos u\neq 0$, 即 $x-y\neq 2k\pi$ $(k\in\mathbb{Z})$ 时, 上述方程化为 592 | \[\frac{\diff u}{1-\cos u}=\diff x.\] 593 | 积分并代回原变量得通解为 $\cot\frac{x-y}{2}+x+C=0$, 当 $1-\cos u=0$ 时, 594 | 有特解 $y=x+2k\pi$ $(k\in\mathbb{Z})$. 595 | 596 | (2) 可以用齐次方程的标准解法求解, 但是也可以用积分因子法, 在方程两边同时乘以 $u$, 得 597 | \[(3u^2v+uv^2)\diff u+(u^3+u^2v)\diff v=0.\] 598 | 分组得 599 | \[(3u^2v\diff u+u^3\diff v)+(uv^2\diff u+u^2v\diff v)=\diff\left(u^3v+\frac{1}{2}u^2v^2\right)=0.\] 600 | 故通解为 $u^3v+\frac{1}{2}u^2v^2=C$. 601 | 602 | (3)原方程等价于 603 | \[\frac{2y\diff y}{2x\diff x} = \frac{4y^2-2x^2}{x^2+y^2+3},\] 604 | 即 605 | \[\frac{\diff y^2}{\diff x^2} = \frac{4y^2-2x^2}{x^2+y^2+3}.\] 606 | 令 $v=x^2,u=y^2$, 则上述方程化为 607 | \[\frac{\diff u}{\diff v}=\frac{4u-2v}{u+v+3}.\] 608 | 令 $\begin{cases}4u-2v=0\\u+v+3=0\end{cases}$, 解得 $u=-1,v=-2$, 故作变换 609 | $\begin{cases}v=\xi-2\\u=\eta-1\end{cases}$, 则方程化为 610 | \[\frac{\diff\eta}{\diff\xi}=\frac{4\eta-2\xi}{\eta+\xi}.\] 611 | 令 $\beta=\frac{\eta}{\xi}$, 则当 $\beta\neq 1$ 且 $\beta\neq 2$ 时, 612 | \[\frac{\diff\eta}{\diff\xi}=\beta+\xi\frac{\diff\beta}{\diff\xi} 613 | = \frac{4\beta-2}{\beta+1} 614 | \Rightarrow\frac{\beta+1}{(\beta-1)(\beta-2)}\diff\beta=-\frac{1}{\xi}\diff\xi.\] 615 | 积分并代回原变量得 $\left(y^2-2x^2-3\right)^3 = C\left(y^2-x^2-1\right)^2$ $(C\neq 0)$. 616 | 当 $\beta=1$ 时, 得特解 $y^2=x^2+1$, 当 $\beta=2$ 时, 617 | 得特解 $y^2-2x^2-3=0$, 显然这个特解可以合并到通解之中. 618 | 综上所述, 原方程的通解为 $\left(y^2-2x^2-3\right)^3=C\left(y^2-x^2-1\right)^2$ $(C\in\mathbb{R})$, 619 | 特解为 $y^2=x^2+1$. 620 | 621 | (4)原方程等价于 622 | \[\frac{\diff y^2}{\diff x^2}=\frac{2x^2+3y^2-7}{3x^2+2y^2-8}.\] 623 | 令 $v=x^2,u=y^2$, 则上述方程化为 624 | \[\frac{\diff u}{\diff v}=\frac{3u+2v-7}{2u+3v-8}.\] 625 | 令 $\begin{cases}v=\xi+2\\u=\eta+1\end{cases}$, 则 626 | \[\frac{\diff\eta}{\diff\xi}=\frac{3\eta+2\xi}{2\eta+3\xi}.\] 627 | 令 $\beta=\frac{\eta}{\xi}$, 则当 $\beta\neq\pm 1$ 时, 628 | \[\beta+\xi\frac{\diff\beta}{\diff\xi} = \frac{3\beta+2}{2\beta+3} 629 | \Rightarrow\frac{2\beta+3}{\beta^2-1}\diff\beta=\frac{-2}{\xi}\diff\xi.\] 630 | 积分并代回原变量得 $(y^2-x^2+1)^5=C(x^2+y^2-3)$ $(C\neq 0)$. 631 | 当 $\beta=1$ 时, 得特解 $y^2-x^2+1=0$, 显然此特解可合并到通解之中, 当 $\beta=-1$ 时得特解 $x^2+y^2-3=0$. 632 | \end{solution} 633 | 634 | 635 | 636 | \begin{exercise} 637 | 求解下列微分方程: 638 | \begin{enumerate}[(1)] 639 | \item $\displaystyle y'=-y^2-\frac{1}{4x^2}$; 640 | \item $\displaystyle x^2y'=x^2y^2+xy+1$. 641 | \end{enumerate} 642 | \end{exercise} 643 | 644 | \begin{solution} 645 | (1)这是 Riccati 方程, 由定理 2.3 中的做法, 令 $z=xy$, 则原方程化为 646 | \[\frac{\diff z}{\diff x}=\frac{-\frac{1}{4}+z-z^2}{x}=\frac{-\left(z-\frac{1}{2}\right)^2}{x}.\] 647 | 当 $z=\frac{1}{2}$ 时得特解 $y=\frac{1}{2x}$, 当 $z\neq\frac{1}{2}$ 时, 上述方程化为 648 | \[\frac{\diff z}{-\left(z-\frac{1}{2}\right)^2}=\frac{\diff x}{x}.\] 649 | 积分得通解为 $y=\frac{1}{2x}+\frac{1}{Cx+x\ln|x|}$. 650 | 651 | (2)这是 Riccati 方程, 容易观察出一个特解为 $y=-\frac{1}{x}$, 652 | 令 $y=u-\frac{1}{x}$, 其中 $u$ 是新的未知函数, 则 653 | \[\frac{\diff y}{\diff x}=\frac{\diff u}{\diff x}+\frac{1}{x^2} 654 | = \left(u-\frac{1}{x}\right)^2+\frac{1}{x}\left(u-\frac{1}{x}\right)+\frac{1}{x^2} 655 | = u^2-\frac{u}{x}+\frac{1}{x^2}.\] 656 | 故 657 | \[\frac{\diff u}{\diff x}+\frac{u}{x} = u^2.\] 658 | 这是伯努利方程, 当 $u=0$ 时, 得特解 $xy+1=0$; 当 $u\neq 0$ 时, 在方程两边同时乘以 $-u^{-2}$, 得 659 | \[-u^{-2}\frac{\diff u}{\diff x}-\frac{u^{-1}}{x}=-1.\] 660 | 令 $z=u^{-1}$, 则上述方程化为一阶线性方程 661 | \[\frac{\diff z}{\diff x}-\frac{z}{x} = -1.\] 662 | 解得 663 | \begin{align*} 664 | z & = \e^{\int\frac{1}{x}\diff x}\left(C+\int-\e^{-\int\frac{1}{x}\diff x}\diff x\right)\\ 665 | & = |x|\left(C+\int\frac{-1}{|x|}\diff x\right)=|x|\left(C-\sgn x\cdot\ln|x|\right)=Cx-x\ln|x|. 666 | \end{align*} 667 | 故原方程的通解为 $y=-\frac{1}{x}+\frac{1}{Cx-x\ln|x|}$. 668 | \end{solution} 669 | 670 | 671 | 672 | \begin{exercise} 673 | 试把二阶微分方程 674 | \[y''+p(x)y'+q(x)y=0\] 675 | 化成一个里卡蒂方程. 676 | \end{exercise} 677 | 678 | \begin{solution} 679 | 令 $y=\e^{\int u\diff x}$, 即得 Riccati 方程 680 | \[u'+u^2+p(x)u+q(x)=0.\qedhere\] 681 | \end{solution} 682 | 683 | 684 | 685 | \begin{exercise} 686 | 求一曲线, 使得过这曲线上任意点的切线与该点向径的夹角等于 $\frac{\pi}{4}$. 687 | \end{exercise} 688 | 689 | \begin{solution} 690 | 设曲线方程为 $y=y(x)$,则 691 | \[\tan\frac{\pi}{4}=\frac{\frac{\diff y}{\diff x}-\frac{y}{x}}{1+\frac{\diff y}{\diff x}\frac{y}{x}} 692 | = 1\Rightarrow\frac{\diff y}{\diff x}=\frac{x+y}{x-y}.\] 693 | 解得 $2\arctan\frac{y}{x}-\ln(x^2+y^2)=C$. 694 | \end{solution} 695 | 696 | 697 | 698 | \begin{exercise} 699 | 探照灯的反光镜 (旋转曲面) 应具有何种形状, 才能使点光源发射的光束反射成平行线束. 700 | \end{exercise} 701 | 702 | \begin{solution} 703 | 设所求曲面由曲线 $y=y(x)$ $(y\geq 0)$ 绕 $x$ 轴旋转而成, 并且不妨将点光源置于原点, 704 | 且平行光线沿 $x$ 轴正方向射出 (所以下面要保证 $\frac{\diff y}{\diff x}>0$), 则由几何关系得 705 | \[\frac{\diff y}{\diff x} 706 | = \frac{\frac{y}{x}-\frac{\diff y}{\diff x}}{1+\frac{y}{x}\frac{\diff y}{\diff x}}.\] 707 | 化简为 708 | \[\frac{y}{x}\left(\frac{\diff y}{\diff x}\right)^2+2\frac{\diff y}{\diff x}-\frac{y}{x}=0.\] 709 | 故 710 | \[\frac{\diff y}{\diff x}=\frac{-1\pm\sqrt{1+\left(\frac{y}{x}\right)^2}}{\frac{y}{x}} 711 | = \frac{-x\pm\sgn x\cdot\sqrt{x^2+y^2}}{y}.\] 712 | 为了使得 $\frac{\diff y}{\diff x}>0$, 取 713 | \[\frac{\diff y}{\diff x}=\frac{-x+\sqrt{x^2+y^2}}{y}.\] 714 | 当 $x>0$ 时, 上述方程等价于 715 | \[\frac{\diff y}{\diff x}=\frac{-1+\sqrt{1+\left(\frac{y}{x}\right)^2}}{\frac{y}{x}}.\] 716 | 令 $u=\frac{y}{x}$, 则 717 | \[u+x\frac{\diff u}{\diff x}=\frac{-1+\sqrt{1+u^2}}{u} 718 | \Rightarrow \frac{u}{1+u^2-\sqrt{1+u^2}}\diff u=-\frac{1}{x}\diff x.\] 719 | 积分并代回原变量得通解 $y^2=C(2x+C)$ $(C>0,x>0)$. 720 | 721 | 当 $x<0$ 时, 上述方程等价于 722 | \[\frac{\diff y}{\diff x}=\frac{-1-\sqrt{1+\left(\frac{y}{x}\right)^2}}{\frac{y}{x}}.\] 723 | 令 $u=\frac{y}{x}$, 则 724 | \[u+x\frac{\diff u}{\diff x}=\frac{-1-\sqrt{1+u^2}}{u} 725 | \Rightarrow \frac{u}{1+u^2+\sqrt{1+u^2}}\diff u=-\frac{1}{x}\diff x.\] 726 | 积分并代回原变量得通解 $y^2=C(2x+C)$ $(C>0,-C/2\leq x<0)$. 727 | 728 | 综上,原方程的通解为 $y^2=C(2x+C)$ $(C>0,x\geq-C/2)$, 729 | 因此旋转曲面的方程为 $y^2+z^2=C(2x+C)$ $(C>0)$, 由此可知该曲面是一个旋转抛物面. 730 | \end{solution} 731 | 732 | 733 | 734 | \section{积分因子法(Integrating Factor)} 735 | 736 | 737 | 738 | \begin{exercise} 739 | 求解下列微分方程: 740 | \begin{enumerate}[(1)] 741 | \item $\displaystyle(3x^2y+2xy+y^3)\diff x+(x^2+y^2)\diff y=0$; 742 | \item $\displaystyle y\diff x+(2xy-\e^{-2y})\diff y=0$; 743 | \item $\displaystyle\left(3x+\frac{6}{y}\right)\diff x+\left(\frac{x^2}{y}+\frac{3y}{x}\right)\diff y=0$; 744 | \item $\displaystyle y\diff x-(x^2+y^2+x)\diff y=0$; 745 | \item $\displaystyle 2xy^3\diff x+(x^2y^2-1)\diff y=0$; 746 | \item $\displaystyle y(1+xy)\diff x-x\diff y=0$; 747 | \item $\displaystyle y^3\diff x+2(x^2-xy^2)\diff y=0$; 748 | \item $\displaystyle\e^x\diff x+(\e^x\cot y+2y\cos y)\diff y=0$. 749 | \end{enumerate} 750 | \end{exercise} 751 | 752 | \begin{solution} 753 | (1)因为 $\frac{1}{Q}\left(\frac{\partial P}{\partial y}-\frac{\partial Q}{\partial x}\right)=3$, 754 | 故取积分因子 $\mu(x)=\e^{3x}$, 得全微分方程: 755 | \[\e^{3x}(3x^2y+2xy+y^3)\diff x+\e^{3x}(x^2+y^2)\diff y=0,\] 756 | 即 757 | \[\diff\left(\e^{3x}x^2y+\frac{1}{3}\e^{3x}y^3\right) = 0.\] 758 | 故通解为 $\e^{3x}(3x^2y+y^3)=C$. 759 | 760 | (2)因为 $\frac{1}{P}\left(\frac{\partial Q}{\partial x}-\frac{\partial P}{\partial y}\right)=2-\frac{1}{y}$, 故取积分因子 $\mu(y)=\frac{\e^{2y}}{y}$, 得全微分方程: 761 | \[\e^{2y}\diff x+\left(2x\e^{2y}-\frac{1}{y}\right)\diff y=0,\] 762 | 即 763 | \[\diff\left(x\e^{2y}-\ln|y|\right)=0.\] 764 | 故通解为 $x\e^{2y}-\ln|y|=C$, 另外特解为 $y=0$. 765 | 766 | (3)在方程两边同时乘以 $xy$, 得 767 | \[3x^2y\diff x+6x\diff x+x^3\diff y+3y^2\diff y=0,\] 768 | 即 769 | \[\diff(x^3y+y^3+3x^2)=0.\] 770 | 故通解为 $x^3y+y^3+3x^2=C$. 771 | 772 | (4)在方程两边同时乘以 $\frac{1}{x^2+y^2}$, 得 773 | \[\frac{y\diff x-x\diff y}{x^2+y^2}-\diff y=\diff\left(-\arctan\frac{y}{x}-y\right)=0.\] 774 | 故通积分为 $\arctan\frac{y}{x}+y=C$ (或者写成 $\arctan\frac{x}{y}-y=C)$, 另有特解 $y=0$. 775 | 776 | (5)因为 $\frac{1}{P}\left(\frac{\partial Q}{\partial x}-\frac{\partial P}{\partial y}\right)=\frac{-2}{y}$, 777 | 故取积分因子 $\mu(y)=\e^{-\int\frac{2}{y}\diff y}=\frac{1}{y^2}$, 得全微分方程: 778 | \[2xy\diff x+x^2\diff y-\frac{1}{y^2}\diff y=\diff\left(x^2y+\frac{1}{y}\right)=0.\] 779 | 故通解为 $x^2y+\frac{1}{y}=C$, 另外有特解 $y=0$. 780 | 781 | (6)因为 $\frac{1}{P}\left(\frac{\partial Q}{\partial x}-\frac{\partial P}{\partial y}\right)=\frac{-2}{y}$, 782 | 故取积分因子 $\mu(y)=\e^{-\int\frac{2}{y}\diff y}=\frac{1}{y^2}$, 得全微分方程: 783 | \[\left(\frac{1}{y}+x\right)\diff x-\frac{x}{y^2}\diff y 784 | = \diff\left(\frac{1}{2}x^2+\frac{x}{y}\right)=0.\] 785 | 故通解为 $\frac{1}{2}x^2+\frac{x}{y}=C$, 另外有特解 $y=0$. 786 | 787 | (7)设方程有积分因子 $\mu=x^my^n$, 则得全微分方程: 788 | \[x^my^{n+3}\diff x+2(x^{m+2}y^n-x^{m+1}y^{n+2})\diff y=0.\] 789 | 于是 790 | \[\frac{\partial\left(x^my^{n+3}\right)}{\partial y} 791 | = \frac{\partial\left(2\left(x^{m+2}y^n-x^{m+1}y^{n+2}\right)\right)}{\partial x}.\] 792 | 即 793 | \[(n+3)x^my^{n+2}=2(m+2)x^{m+1}y^n-2(m+1)x^my^{n+2}.\] 794 | 比较系数得 $2(m+2)=0,n+3=-2(m+1)$, 解得 $m=-2,n=-1$, 795 | 故方程有积分因子 $\mu=\frac{1}{x^2y}$, 因而得全微分方程: 796 | \[\frac{y^2}{x^2}\diff x+2\left(\frac{1}{y}-\frac{y}{x}\right)\diff y 797 | =\diff\left(\ln y^2-\frac{y^2}{x}\right)=0.\] 798 | 故通解为 $\ln y^2-\frac{y^2}{x}=C$,另外有特解$x=0,y=0$. 799 | 800 | 注: 对于 $P(x,y)$ 和 $Q(x,y)$ 都是关于 $x,y$ 的多项式的情形, 使用这种方法比较好. 801 | 802 | (8)因为$\frac{1}{P}\left(\frac{\partial Q}{\partial x}-\frac{\partial P}{\partial y}\right)=\cot y$, 803 | 故取积分因子 $\mu(y)=\sin y$, 得全微分方程: 804 | \[\e^x\sin y\diff x+\left(\e^x\cos y+2y\sin y\cos y\right)\diff y 805 | = \diff\left(\e^x\sin y+\frac{1}{4}\sin2y-\frac{1}{2}y\cos2y\right)=0.\] 806 | 故通解为 $\e^x\sin y+\frac{1}{4}\sin2y-\frac{1}{2}y\cos2y=C$. 807 | \end{solution} 808 | 809 | 810 | 811 | \begin{exercise} 812 | 证明方程$P(x,y)\diff x+Q(x,y)\diff y=0$有形如$\mu=\mu(\varphi(x,y))$的积分因子的充要条件是 813 | \[\frac{\frac{\partial P}{\partial y}-\frac{\partial Q}{\partial x}}{Q\frac{\partial\varphi}{\partial x}-P\frac{\partial\varphi}{\partial y}}=f(\varphi(x,y))\] 814 | 并写出这个积分因子.然后将结果应用到下述各种情形,得出存在每一种类型积分因子的充要条件: 815 | \begin{enumerate}[(1)] 816 | \item $\mu=\mu(x\pm y)$; 817 | \item $\mu=\mu(x^2+y^2)$; 818 | \item $\mu=\mu(xy)$; 819 | \item $\mu=\mu(\frac{y}{x})$; 820 | \item $\mu=\mu(x^{\alpha}y^{\beta})$. 821 | \end{enumerate} 822 | \end{exercise} 823 | 824 | \begin{proof} 825 | \begin{align*} 826 | & \text{有积分因子\ }\mu=\mu(\varphi(x,y))\\ 827 | & \Longleftrightarrow\frac{\partial(\mu P)}{\partial y}=\frac{\partial(\mu Q)}{\partial x}\\ 828 | & \Longleftrightarrow\left(Q\frac{\partial\varphi}{\partial x}-P\frac{\partial\varphi}{\partial y}\right)\frac{\diff\mu}{\diff s}\bigg|_{s=\varphi(x,y)}=\left(\frac{\partial P}{\partial y}-\frac{\partial Q}{\partial x}\right)\mu\\ 829 | & \Longleftrightarrow\frac{\frac{\partial P}{\partial y}-\frac{\partial Q}{\partial x}}{Q\frac{\partial\varphi}{\partial x}-P\frac{\partial\varphi}{\partial y}}=\frac{1}{\mu}\frac{\diff\mu}{\diff s}\bigg|_{s=\varphi(x,y)}=:f(\varphi(x,y)). 830 | \end{align*} 831 | 此时有积分因子 $\mu=\e^{\int f(s)\diff s}|_{s=\varphi(x,y)}$, 于是 832 | 833 | (1) 有积分因子 $\mu=\mu(x\pm y)\Longleftrightarrow\frac{\frac{\partial P}{\partial y}-\frac{\partial Q}{\partial x}}{Q\mp P}=f(x\pm y)$; 834 | 835 | (2) 有积分因子 $\mu=\mu(x^2+y^2)\Longleftrightarrow\frac{\frac{\partial P}{\partial y}-\frac{\partial Q}{\partial x}}{2xQ-2yP}=f(x^2+y^2)$; 836 | 837 | (3) 有积分因子 $\mu=\mu(xy)\Longleftrightarrow\frac{\frac{\partial P}{\partial y}-\frac{\partial Q}{\partial x}}{yQ-xP}=f(xy)$; 838 | 839 | (4) 有积分因子 $\mu=\mu(\frac{y}{x})\Longleftrightarrow\frac{\frac{\partial P}{\partial y}-\frac{\partial Q}{\partial x}}{-\frac{y}{x^2}Q-\frac{1}{x}P}=f(\frac{y}{x})$; 840 | 841 | (5) 有积分因子 $\mu=\mu(x^{\alpha}y^{\beta})\Longleftrightarrow\frac{\frac{\partial P}{\partial y}-\frac{\partial Q}{\partial x}}{\frac{\alpha}{x}Q-\frac{\beta}{y}P}=f(x^{\alpha}y^{\beta})$. 842 | \end{proof} 843 | 844 | 845 | 846 | \begin{exercise} 847 | 证明齐次方程 $P(x,y)\diff x+Q(x,y)\diff y=0$ 有积分因子 $\mu=\frac{1}{xP+yQ}$. 848 | \end{exercise} 849 | 850 | \begin{proof} 851 | (证法 1) 设 $P(x,y)$ 和 $Q(x,y)$ 是 $m$ 次齐次函数, 即 852 | \[P (tx,ty)=t^mP(x,y),Q(tx,ty)=t^mQ(x,y).\] 853 | 两边对 $t$ 求导并取 $t=1$, 得 854 | \[x\frac{\partial P}{\partial x}+y\frac{\partial P}{\partial y} 855 | = mP,x\frac{\partial Q}{\partial x}+y\frac{\partial Q}{\partial y}=mQ.\] 856 | 为了证明 $\frac{1}{xP+yQ}$ 是齐次方程的积分因子, 只需要证明 857 | \[\frac{\partial}{\partial y}\left(\frac{P}{xP+yQ}\right) 858 | = \frac{\partial}{\partial x}\left(\frac{Q}{xP+yQ}\right).\] 859 | 通过计算可得 860 | \[\begin{split} 861 | &\frac{\partial}{\partial y}\left(\frac{P}{xP+yQ}\right)-\frac{\partial}{\partial x}\left(\frac{Q}{xP+yQ}\right)\\ 862 | =&\frac{\frac{\partial P}{\partial y}(xP+yQ)-P\left(x\frac{\partial P}{\partial y}+Q+y\frac{\partial Q}{\partial y}\right)}{(xP+yQ)^2}\\ 863 | &-\frac{\frac{\partial Q}{\partial x}(xP+yQ)-Q\left(P+x\frac{\partial P}{\partial x}+y\frac{\partial Q}{\partial x}\right)}{(xP+yQ)^2}\\ 864 | =&\frac{1}{(xP+yQ)^2}\left[Q\left(x\frac{\partial P}{\partial x}+y\frac{\partial P}{\partial y}\right)-P\left(y\frac{\partial Q}{\partial y}+x\frac{\partial Q}{\partial x}\right)\right]\\ 865 | =&\frac{1}{(xP+yQ)^2}(mQP-mPQ)=0. 866 | \end{split}\] 867 | 故 $\mu=\frac{1}{xP+yQ}$ 是积分因子. 868 | 869 | (证法 2) 设 $P(x,y)$ 和 $Q(x,y)$ 是 $m$ 次齐次函数, 即 870 | \[P(tx,ty)=t^mP(x,y),Q(tx,ty)=t^mQ(x,y).\] 871 | 令 $y=ux$, 则 $\diff y=u\diff x+x\diff u$, 于是方程变为 872 | \[\begin{split} 873 | & P(x,y)\diff x+Q(x,y)\diff y \\ 874 | ={} & P(x,ux)\diff x+Q(x,ux)(u\diff x+x\diff u) \\ 875 | ={} & x^m\left[P(1,u)+uQ(1,u)\right]\diff x+x^{m+1}Q(1,u)\diff u=0. 876 | \end{split}\] 877 | 上式为变量分离的方程, 有积分因子 878 | \[\mu=\frac{1}{x^{m+1}[P(1,u)+uQ(1,u)]}.\] 879 | 将 $u=\frac{y}{x}$ 代回, 即得原方程有积分因子 $\mu=\frac{1}{xP+yQ}$. 880 | \end{proof} 881 | 882 | 883 | 884 | \begin{exercise} 885 | 证明定理 2.6 及其逆定理: 在定理 2.6 的假定下, 886 | 若 $\mu_1$ 是微分方程 $P(x,y)\diff x+Q(x,y)\diff y=0$ 的另一个积分因子, 887 | 则 $\mu_1$ 必可表为 $\mu_1=\mu g(\varPhi)$ 的形式, 其中函数 $g$ 和 $\varPhi$ 的意义与在定理 2.6 中的相同. 888 | \end{exercise} 889 | 890 | \begin{proof} 891 | 由 892 | \[\begin{split} 893 | &\mu(x,y)g(\varPhi(x,y))P(x,y)\diff x+\mu(x,y)g(\varPhi(x,y))Q(x,y)\diff y\\ 894 | =&g(\varPhi(x,y))\diff\varPhi(x,y)=\diff G(\varPhi(x,y)) 895 | \end{split}\] 896 | 即证,其中$G(s)=\int g(s)\diff s$.下面证明其逆定理: 897 | 898 | 设 899 | \[\mu_1P(x,y)\diff x+\mu_1Q(x,y)\diff y=\diff\varPsi.\] 900 | 因为 901 | \[\frac{D[\varPhi,\varPsi]}{D[x,y]}= 902 | \begin{vmatrix}\mu P&\mu Q\\\mu_1P&\mu_1Q\end{vmatrix}=0,\] 903 | 所以 $\varPhi$ 和 $\varPsi$ 函数相关, 故 904 | \[\frac{\mu_1}{\mu}=\frac{\diff\varPsi}{\diff\varPhi}\] 905 | 可以表示为 $\varPhi$ 的函数. 906 | \end{proof} 907 | 908 | 909 | 910 | \begin{exercise} 911 | 设函数 $P(x,y)$, $Q(x,y)$, $\mu_1(x,y)$ 和 $\mu_2(x,y)$ 都是连续可微的, $\mu_1$ 和 $\mu_2$ 是微分方程 912 | \[P(x,y)\diff x+Q(x,y)\diff y=0\] 913 | 的两个积分因子, 而且 $\frac{\mu_1}{\mu_2}$ 不恒为常数. 914 | 试证: $\frac{\mu_1(x,y)}{\mu_2(x,y)}=C$ 是方程 $P(x,y)\diff x+Q(x,y)\diff y\-=0$ 的一个通积分. 915 | \end{exercise} 916 | 917 | \begin{proof} 918 | 先证明一个引理: 设 $P(x,y)\diff x+Q(x,y)\diff y=0$ 是恰当方程, 919 | 且 $\mu(x,y)\neq C$ 为其积分因子, 则 $\mu(x,y)=C$ 是其一个通解. 事实上, 920 | 因为 $\mu(x,y)$ 为其积分因子, 所以 921 | \[\frac{\partial}{\partial y}(\mu P)=\frac{\partial}{\partial x}(\mu Q).\] 922 | 即 923 | \[P\frac{\partial\mu}{\partial y}+\mu\frac{\partial P}{\partial y} 924 | = Q\frac{\partial\mu}{\partial x}+\mu\frac{\partial Q}{\partial x}.\] 925 | 又因为 $\frac{\partial P}{\partial y}=\frac{\partial Q}{\partial x}$, 故 926 | \[P\frac{\partial\mu}{\partial y}=Q\frac{\partial\mu}{\partial x}.\] 927 | 在原方程两边同时乘以 $\frac{\partial\mu}{\partial y}$, 得 928 | \[P\frac{\partial\mu}{\partial y}\diff x+Q\frac{\partial\mu}{\partial y}\diff y 929 | = Q\left(\frac{\partial\mu}{\partial x}\diff x+\frac{\partial\mu}{\partial y}\diff y\right) 930 | = Q\diff\mu=0\Rightarrow\mu(x,y)=C.\] 931 | 故 $\mu(x,y)=C$ 是其一个通解, 引理证毕. 下证本题定理: 932 | 933 | 因为 $\mu_1,\mu_2$ 是 $P(x,y)\diff x+Q(x,y)\diff y=0$ 两个积分因子, 934 | 故 $\mu_1P(x,y)\diff x+\mu_1Q(x,y)\diff y=0$ 为恰当方程, 935 | 且 $\frac{\mu_2}{\mu_1}\neq C$ 为其积分因子, 936 | 故由引理结论知 $\frac{\mu_1(x,y)}{\mu_2(x,y)}=C$ 是方程 $P(x,y)\diff x+Q(x,y)\diff y=0$ 的一个通积分. 937 | \end{proof} 938 | 939 | 940 | 941 | \section{应用举例} 942 | 943 | 944 | 945 | \begin{exercise} 946 | 求下列各曲线族的正交轨线族: 947 | \begin{enumerate}[(1)] 948 | \item $x^2+y^2=Cx$; 949 | \item $xy=C$; 950 | \item $y^2=ax^3$; 951 | \item $x^2+C^2y^2=1$. 952 | \end{enumerate} 953 | \end{exercise} 954 | 955 | \begin{solution} 956 | (1) 联立 $x^2+y^2=Cx$ 与 $(2x-C)\diff x+2y\diff y=0$, 957 | 消去 $C$ 得曲线族满足微分方程 $\frac{\diff y}{\diff x}=\frac{y^2-x^2}{2xy}$, 958 | 故正交曲线族的微分方程为 959 | \[\frac{\diff y}{\diff x}=\frac{2xy}{x^2-y^2}.\] 960 | 解得 $x^2+y^2=Ky$ $(K\neq 0)$. 961 | 962 | (2) 曲线族 $xy=C$ 满足的微分方程为 $y\diff x+x\diff y=0$, 故正交曲线族的微分方程为 963 | \[-x\diff x+y\diff y=0.\] 964 | 解得 $x^2-y^2=K$. 965 | 966 | (3) 联立 $y^2=ax^3$ 与 $2y\diff y=3ax^2\diff x$, 967 | 消去 $a$ 得曲线族满足微分方程 $3y\diff x-2x\diff y=0$, 故正交曲线族的微分方程为 968 | \[2x\diff x+3y\diff y=0.\] 969 | 解得 $x^2+\frac{3}{2}y^2=K$. 970 | 971 | (4) 联立 $x^2+C^2y^2=1$ 与 $2x\diff x+2C^2y\diff y=0$, 972 | 消去 $C^2$ 得曲线族满足微分方程 $xy\diff x-(x^2-1)\diff y=0$, 故正交曲线族的微分方程为 973 | \[(x^2-1)\diff x+xy\diff y=0.\] 974 | 解得 $x^2+y^2-\ln x^2=K$, 特解 $x=0$. 975 | \end{solution} 976 | 977 | 978 | 979 | \begin{exercise} 980 | 求与下列各曲线族相交成$\frac{\pi}{4}$角的曲线族: 981 | \begin{enumerate}[(1)] 982 | \item $x-2y=C$; 983 | \item $xy=C$; 984 | \item $y=x\ln ax$; 985 | \item $y^2=4ax$. 986 | \end{enumerate} 987 | \end{exercise} 988 | 989 | \begin{solution} 990 | (1) 991 | \[\tan\frac{\pi}{4}=\frac{y'-y_1'}{1+y'y_1'}=1\Rightarrow y_1' 992 | = \frac{y'-1}{y'+1}=\frac{1}{2}\Rightarrow y'=3.\] 993 | 积分得等角轨线族为 $y=3x+K$. 994 | 995 | (2) $xy=C$ 满足的微分方程为 $y\diff x+x\diff y=0$, 故 996 | \[y_1'=\frac{y'-1}{y'+1}=-\frac{y}{x}\Rightarrow(x-y)\diff x-(x+y)\diff y=0.\] 997 | 积分得等角轨线族为 $x^2-y^2-2xy=K$. 998 | 999 | (3) 联立 $y=x\ln ax$ 与 $\diff y=(\ln ax+1)\diff x$ 1000 | 消去 $a$ 得曲线族 $y=x\ln ax$ 满足微分方程 $\frac{\diff y}{\diff x}=\frac{y}{x}+1$, 故 1001 | \[y_1'=\frac{y'-1}{y'+1}=\frac{x+y}{x}\Rightarrow\frac{\diff y}{\diff x}=\frac{-2x}{y}-1.\] 1002 | 积分得等角轨线族为 $\ln(y^2+xy+2x^2)-\frac{2}{\sqrt{7}}\arctan\frac{2y+x}{\sqrt{7}x}=K$. 1003 | 1004 | (4) $y^2=4ax$ 满足的微分方程为 $y\diff x-2x\diff y=0$, 故 1005 | \[y_1'=\frac{y'-1}{y'+1}=\frac{y}{2x}\Rightarrow\frac{\diff y}{\diff x}=\frac{2x+y}{2x-y}.\] 1006 | 积分得等角轨线族为 $\ln(y^2-xy+2x^2)-\frac{6}{\sqrt{7}}\arctan\frac{2y-x}{\sqrt{7}x}=K$. 1007 | \end{solution} 1008 | 1009 | 1010 | 1011 | \begin{exercise} 1012 | 给定双曲线族 $x^2-y^2=C$ (其中 $C$ 是任意常数). 设有一个动点 $P$ 在平面 $(x,y)$ 上移动, 1013 | 它的轨迹与和它相交的每条双曲线均成 $\frac{\pi}{6}$ 角, 又设此动点从 $P_0(0,1)$ 出发, 求出动点的轨迹. 1014 | \end{exercise} 1015 | 1016 | \begin{solution} 1017 | 双曲线族 $x^2-y^2=C$ 满足的微分方程为 $x\diff x-y\diff y=0$, 设动点的轨迹方程为 $y=y(x)$, 则 1018 | \[\tan\frac{\pi}{6}=\frac{\sqrt{3}}{3}=\frac{y'-y_1'}{1+y'y_1'}.\] 1019 | 从上式解得 1020 | \[y_1'=\frac{\sqrt{3}y'-1}{y'+\sqrt{3}}=\frac{x}{y} 1021 | \Rightarrow\frac{\diff y}{\diff x}=\frac{\sqrt{3}x+y}{\sqrt{3}y-x}.\] 1022 | 令 $u=\frac{y}{x}$, 则 1023 | \[u+x\frac{\diff u}{\diff x}=\frac{\sqrt{3}+u}{\sqrt{3}u-1} 1024 | \Rightarrow\left(\frac{\frac{1}{2}}{u-\sqrt{3}} 1025 | + \frac{\frac{\sqrt{3}}{2}}{\sqrt{3}u+1}\right)\diff u=-\frac{1}{x}\diff x.\] 1026 | 积分得 $\sqrt{3}y^2-2xy-\sqrt{3}x^2=C\neq 0$, 代入初值 $P_0(0,1)$ 得 $C=\sqrt{3}$, 1027 | 故动点轨迹方程为 $\sqrt{3}(x^2-y^2+1)+2xy=0$. 1028 | \end{solution} 1029 | 1030 | 1031 | 1032 | \begin{exercise}(追线) 1033 | 设在 $Oxy$ 平面上, 有某物 $P$ 从原点 $O$ 出发, 以常速 $a>0$ 沿 $x$ 轴的正方向运动. 1034 | 同时又有某物 $Q$ 以常速 $b$ 从点 $(0,1)$ 出发追赶 $P$. 设 $b>a$, 且 $Q$ 的运动方向永远指向 $P$. 1035 | 试求 $Q$ 的运动轨迹与追上 $P$ 的时间. 1036 | \end{exercise} 1037 | 1038 | \begin{solution} 1039 | 设点 $Q$ 的运动轨迹方程为 $y=y(x)$, 记某时刻 $Q$ 的坐标为 $(x,y(x))$, 1040 | 则相应地点 $P$ 的坐标为 $\left(x-\frac{y(x)}{y'(x)},0\right)$, 由时间关系得 1041 | \[\frac{\int_0^x\sqrt{1+(y'(t))^2}\diff t}{b}=\frac{x-\frac{y(x)}{y'(x)}}{a}.\] 1042 | 将上式对 $x$ 求导得 1043 | \[\frac{1}{b}\sqrt{1+(y'(x))^2}=\frac{1}{a}\left(1-\frac{(y'(x))^2-y(x)y''(x)}{(y'(x))^2}\right) 1044 | = \frac{y(x)y''(x)}{a(y'(x))^2}.\] 1045 | 即 1046 | \[\frac{a}{b}\sqrt{1+(y')^2}=\frac{yy''}{(y')^2}.\] 1047 | 令 $p=y'$, 则 $y''=\frac{\diff p}{\diff x}=\frac{\diff p}{\diff y}\frac{\diff y}{\diff x}=p\frac{\diff p}{\diff y}$, 1048 | 从而上式化为 1049 | \[\frac{a}{b}\sqrt{1+p^2}=\frac{yp\frac{\diff p}{\diff y}}{p^2} 1050 | \Rightarrow\frac{\diff p}{p\sqrt{1+p^2}}=\frac{a}{b}\frac{\diff y}{y}.\] 1051 | 注意到 $p<0$, 故 1052 | \[\frac{a}{b}\frac{\diff y}{y}=\frac{-\frac{1}{p^2}\diff p}{\sqrt{1+\left(\frac{1}{p}\right)^2}} 1053 | = \frac{\diff\left(\frac{1}{p}\right)}{\sqrt{1+\left(\frac{1}{p}\right)^2}}.\] 1054 | 积分得 1055 | \[\ln\left(\frac{1}{p}+\sqrt{1+\left(\frac{1}{p}\right)^2}\right)=\frac{a}{b}(\ln y+\ln C).\] 1056 | 因此 1057 | \[\frac{1}{p}+\sqrt{1+\left(\frac{1}{p}\right)^2}=(Cy)^{\frac{a}{b}}.\] 1058 | 当 $y=1$ 时, $\frac{1}{p}=0$, 故代入初值条件得 $C=1$, 故 1059 | \[\frac{1}{p}+\sqrt{1+\left(\frac{1}{p}\right)^2}=y^{\frac{a}{b}}.\] 1060 | 取倒数得 1061 | \[-\frac{1}{p}+\sqrt{1+\left(\frac{1}{p}\right)^2}=y^{-\frac{a}{b}}.\] 1062 | 两式相减可得变量分离的方程 1063 | \[\diff x=\frac{1}{2}\left(y^{\frac{a}{b}}-y^{-\frac{a}{b}}\right)\diff y.\] 1064 | 积分得 1065 | \[x=\frac{b}{2(a+b)}y^{\frac{a+b}{b}}-\frac{b}{2(b-a)}y^{\frac{b-a}{b}}+C_1.\] 1066 | 代入初值条件 $(x,y)=(0,1)$ 得 $C_1=\frac{ab}{b^2-a^2}$, 故点 $Q$ 的运动轨迹为 1067 | \[x=\frac{b}{2(a+b)}y^{\frac{a+b}{b}}-\frac{b}{2(b-a)}y^{\frac{b-a}{b}}+\frac{ab}{b^2-a^2}.\] 1068 | 当 $Q$ 追上 $P$ 时, 重合点的横坐标为 $x_1=\frac{ab}{b^2-a^2}$, 1069 | 故时间为 $t=\frac{x_1}{a}=\frac{b}{b^2-a^2}$. 1070 | \end{solution} 1071 | 我们可以在本题的基础上讨论更一般的问题: 在正 $n$ 边形的每个顶点上分别有一个物体, 1072 | 按逆时针方向每一个物体都以不变的速度 $v$ 跟踪与它相邻的物体, 那么如何求每个物体的运动轨迹. 1073 | 1074 | 以 $n=4$ 为例, 设 $t=0$ 时四个物体 $A,B,C$ 和 $D$ 分别在二维平面上的 1075 | $(1,1)$, $(-1,1)$, $(-1,-1)$ 和 $(1,-1)$ 处, 1076 | 从此刻开始 $A,B,C,D$ 分别以不变的速度 $v$ 追赶 $B,C,D,A$. 求物体 $A$ 的运动轨迹. 1077 | \begin{solution} 1078 | 设 $A$ 的轨迹方程为 $y=y(x)$, 记某时刻 $A$ 的坐标为 $(x,y(x))$, 1079 | 则此时 $B$ 的坐标为 $(-y(x),x)$, 由于 $A$ 的运动方向指向 $B$, 故 1080 | \[y'(x)=\frac{y(x)-x}{x+y(x)}.\] 1081 | 此为齐次方程容易解得 1082 | \[2\arctan\frac{y}{x}+\ln\left(x^2+y^2\right)=\frac{\pi}{2}+\ln 2.\qedhere\] 1083 | \end{solution} 1084 | 1085 | 1086 | 1087 | \begin{exercise}(逃逸速度) 1088 | 假设地球的半径为 $R=\qty{6437}{\kilo\metre}$, 1089 | 地面上的重力加速度为 $g=\qty[per-mode=symbol]{9.8}{\meter\per\square\second}$, 1090 | 又设质量为 $M$ 的火箭在地面以初速 $v_0$ 垂直上升. 1091 | 假设不计空气阻力和其他任何星球的引力. 试求火箭的逃逸速度, 即: 使火箭一去不复返的最小初速度 $v_0$. 1092 | \end{exercise} 1093 | 1094 | \begin{solution} 1095 | 逃逸速度又称为第二宇宙速度, 取沿地球径向向外为正方向, 记地球质量为 $M_1$, 1096 | 记火箭的速度函数为 $v=v(t)$, 位移函数为 $s=s(t)$, 则由牛顿第二定律知 1097 | \[M\frac{\diff^2s}{\diff t^2}=-\frac{GM_1M}{s^2}.\] 1098 | 故 1099 | \[\frac{\diff^2s}{\diff t^2}=-\frac{GM_1}{s^2}.\] 1100 | 由黄金代换 $GM_1=gR^2$ 得 1101 | \[-\frac{gR^2}{s^2}=\frac{\diff v}{\diff t}=\frac{\diff v}{\diff s}\frac{\diff s}{\diff t} 1102 | =v\frac{\diff v}{\diff s}.\] 1103 | 分离变量得 1104 | \[v\diff v=-gR^2\frac{\diff s}{s^2}.\] 1105 | 积分并代入初值条件得 1106 | \[\frac{1}{2}v^2=\frac{gR^2}{s}+\left(\frac{1}{2}v_0^2-gR\right).\] 1107 | 要使得火箭从地球逃逸, 就必须始终有 $v>0$, 因此 1108 | \[\frac{1}{2}v_0^2-gR\geq 0 1109 | \Longrightarrow v_0\geq\sqrt{2gR}=\qty[per-mode=symbol]{11.2}{\kilo\meter\per\second}.\qedhere\] 1110 | \end{solution} 1111 | 1112 | 1113 | 1114 | \begin{exercise} 1115 | 设某社会的总人数为$N$,当时流行一种传染病,得病人数为$x$.设传染病人数的扩大率是与得病人数和未得病人数的乘积成正比. 1116 | 试讨论传染病人数的发展趋势,并以此解释对传染病人进行隔离的必要性. 1117 | \end{exercise} 1118 | 1119 | \begin{solution} 1120 | 设比例常数为 $k$, 则依题意得 1121 | \[\frac{\diff x}{\diff t}=kx(N-x).\] 1122 | 上式为变量分离的方程, 容易解得 $x(t)=\frac{CN\e^{kNt}}{1+C\e^{kNt}}$, 1123 | 当 $t\to+\infty$ 时, $x(t)\to N$, 因此对传染病人进行隔离是有必要的. 1124 | \end{solution} -------------------------------------------------------------------------------- /chapter06.tex: -------------------------------------------------------------------------------- 1 | \chapter{线性微分方程组} 2 | 3 | 4 | 5 | \section{一般理论} 6 | 7 | 8 | 9 | \subsection{证明与总结} 10 | 11 | 12 | 13 | P159证明: $H(C_1\bm{y}_1^0+C_2\bm{y}_2^0)=C_1H(\bm{y}_1^0)+C_2H(\bm{y}_2^0)$. 14 | \begin{proof} 首先显然$H(C_1\bm{y}_1^0+C_2\bm{y}_2^0)\in\mathcal{S}$, 15 | 又由引理6.1知$C_1H(\bm{y}_1^0)+C_2H(\bm{y}_2^0)\in\mathcal{S}$.然后又因为 16 | \[\begin{split}\left(H(C_1\bm{y}_1^0+C_2\bm{y}_2^0)\right)(x_0)&=C_1\bm{y}_1^0+C_2\bm{y}_2^0\\ 17 | \left(C_1H(\bm{y}_1^0)+C_2H(\bm{y}_2^0)\right)(x_0)&=C_1\bm{y}_1^0+C_2\bm{y}_2^0 18 | \end{split}\] 19 | 由解的唯一性知$H(C_1\bm{y}_1^0+C_2\bm{y}_2^0)=C_1H(\bm{y}_1^0)+C_2H(\bm{y}_2^0)$. 20 | 21 | 注意解矩阵的行列式就是其对应的解组的Wronsky行列式. 22 | \end{proof} 23 | 24 | 推论6.2的证明: 25 | 26 | \begin{proof} 27 | (1) 记 $\bm{C}=(\bm{c}_1,\cdots,\bm{c}_n)$, 28 | 则 $\bm{\varPhi}(x)\bm{C}=(\bm{\varPhi}(x)\bm{c}_1,\cdots,\bm{\varPhi}(x)\bm{c}_n)$, 29 | 由 (6.15) 式知 $\forall 1\leq i\leq n$, $\bm{\varPhi}(x)\bm{c}_i$ 是方程(6.2)的解, 30 | 也即 $\{\bm{\varPhi}(x)\bm{c}_i|1\leq i\leq n\}$ 为(6.2)的解组. 31 | 记 $\bmitPhi(x)$ 对应的基本解组的 Wronsky 行列式为 $W(x)$, 32 | 则 $\bmitPhi(x)\bm{C}$ 对应的解组的 Wronsky 行列式为: 33 | \[W_1(x) = |\bmitPhi(x)\bm{c}_1\cdots\bmitPhi(x)\bm{c}_n| 34 | = |\bmitPhi(x)|\cdot|\bm{C}|=W(x)|\bm{C}|\neq 0.\] 35 | 故 $\bmitPhi(x)\bm{C}$也是基解矩阵. 36 | 37 | (2) $\bmitPhi(x)$ 和 $\bmitPsi(x)$ 都是基解矩阵, 设 38 | \[\bmitPhi(x)=(\bm{y}_1(x),\cdots,\bm{y}_n(x)),\] 39 | \[\bmitPsi(x)=(\bm{y}_1^*(x),\cdots,\bm{y}_n^*(x)).\] 40 | 因为 $\bmitPhi(x)$ 是基解矩阵, 所以 $\bm{y}_1(x),\cdots,\bm{y}_n(x)$ 是 (6.2) 的基本解组, 41 | 故存在 $\{c_{ij}\mid 1\leq i,j\leq n\}$ 使得 42 | \[\bm{y}_i^*(x)=\sum_{j=1}^nc_{ji}\bm{y}_j(x),1\leq i\leq n.\] 43 | 即 44 | \[(\bm{y}_1^*(x),\cdots,\bm{y}_n^*(x)) = (\bm{y}_1(x),\cdots,\bm{y}_n(x)) 45 | \begin{pmatrix} 46 | c_{11}&c_{12}&\cdots&c_{1n}\\ 47 | c_{21}&c_{22}&\cdots&c_{2n}\\ 48 | \vdots&\vdots&&\vdots\\ 49 | c_{n1}&c_{n2}&\cdots&c_{nn} 50 | \end{pmatrix}\] 51 | 也即 52 | \[\bmitPsi(x)=\bmitPhi(x)\bm{C},\] 53 | 其中 $\bm{C}=(c_{ij})_{n\times n}$, 在上述等式两边同时取行列式得$|\bmitPsi|=|\bmitPhi|\cdot|\bm{C}|$, 54 | 由 $|\bmitPsi|\neq 0$, $|\bmitPhi|\neq 0$ 知 $|\bm{C}|\neq 0$. 55 | \end{proof} 56 | 57 | 58 | 59 | \subsection{习题} 60 | 61 | 62 | 63 | \begin{exercise} 64 | 求出齐次线性微分方程组 65 | \[\frac{\diff\bm{y}}{\diff x}=\bm{A}(t)\bm{y}\] 66 | 的通解, 其中 $\bm{A}(t)$ 分别为: 67 | \begin{enumerate}[(1)] 68 | \item $\displaystyle\bm{A}(t)=\begin{pmatrix}\frac{1}{t}&0\\0&\frac{1}{t}\end{pmatrix},t\neq0$; 69 | \item $\displaystyle\bm{A}(t)=\begin{pmatrix}1&1\\0&1\end{pmatrix}$; 70 | \item $\displaystyle\bm{A}(t)=\begin{pmatrix}0&1\\-1&0\end{pmatrix}$; 71 | \item $\displaystyle\bm{A}(t)=\begin{pmatrix}0&0&1\\0&1&0\\1&0&0\end{pmatrix}$. 72 | \end{enumerate} 73 | \end{exercise} 74 | 75 | \begin{solution} 76 | (1) $\displaystyle\frac{\diff}{\diff t}\begin{pmatrix}y_1\\y_2\end{pmatrix}=\begin{pmatrix}\frac{1}{t}&0\\0&\frac{1}{t}\end{pmatrix}\begin{pmatrix}y_1\\y_2\end{pmatrix}$, 77 | 分量形式为 $\displaystyle\frac{\diff y_1}{\diff t}=\frac{y_1}{t},\frac{\diff y_2}{\diff t}=\frac{y_2}{t}$, 78 | 解得 $y_1=kt,y_2=kt$, 不妨取基解矩阵为 $\begin{pmatrix}0&t\\t&0\end{pmatrix}$, 则通解为 79 | \[\bm{y}(t)=C_1\begin{pmatrix}0\\t\end{pmatrix}+C_2\begin{pmatrix}t\\0\end{pmatrix}.\] 80 | 81 | (2) $\displaystyle\frac{\diff}{\diff t}\begin{pmatrix}y_1\\y_2\end{pmatrix}=\begin{pmatrix}1&1\\0&1\end{pmatrix}\begin{pmatrix}y_1\\y_2\end{pmatrix}$, 82 | 分量形式为$\displaystyle\frac{\diff y_1}{\diff t}=y_1+y_2,\frac{\diff y_2}{\diff t}=y_2$, 83 | 由$\displaystyle\frac{\diff y_2}{\diff t}=y_2$解得$y_2=C\e^t$, 先取$y_2=0$得$y_1=C\e^t$, 84 | 再取$y_2=\e^t$得$y_1=C\e^t+t\e^t$, 不妨取基解矩阵为$\begin{pmatrix}\e^t&t\e^t\\0&\e^t\end{pmatrix}$, 则通解为 85 | \[\bm{y}(t)=C_1\begin{pmatrix}\e^t\\0\end{pmatrix}+C_2\begin{pmatrix}t\e^t\\\e^t\end{pmatrix}\] 86 | 87 | (3) $\displaystyle\frac{\diff}{\diff t}\begin{pmatrix}y_1\\y_2\end{pmatrix}=\begin{pmatrix}0&1\\-1&0\end{pmatrix}\begin{pmatrix}y_1\\y_2\end{pmatrix}$, 88 | 分量形式为 $\displaystyle\frac{\diff y_1}{\diff t}=y_2,\frac{\diff y_2}{\diff t}=-y_1$, 89 | 故 $\displaystyle\frac{\diff^2y_1}{\diff t}=\frac{\diff y_2}{\diff t}=-y_1\Rightarrow y_1=C_1\sin t+C_2\cos t,y_2=C_1\cos t-C_2\sin t$, 90 | 不妨取基解矩阵为 $\begin{pmatrix}\sin t&\cos t\\\cos t&-\sin t\end{pmatrix}$, 则通解为 91 | \[\bm{y}(t)=C_1\begin{pmatrix}\sin t\\\cos t\end{pmatrix}+C_2\begin{pmatrix}\cos t\\-\sin t\end{pmatrix}.\] 92 | 93 | (4) $\displaystyle\frac{\diff}{\diff t}\begin{pmatrix}y_1\\y_2\\y_3\end{pmatrix}=\begin{pmatrix}0&0&1\\0&1&0\\1&0&0\end{pmatrix}\begin{pmatrix}y_1\\y_2\\y_3\end{pmatrix}$, 94 | 分量形式为 $\displaystyle\frac{\diff y_1}{\diff t}=y_3,\frac{\diff y_2}{\diff t}=y_2,\frac{\diff y_3}{\diff t}=y_1$, 95 | 由 $\displaystyle\frac{\diff y_2}{\diff t}=y_2$ 96 | 得 $y_2=C\e^t$, 又$\displaystyle\frac{\diff^2y_1}{\diff t^2}=\frac{\diff y_3}{\diff t}=y_1\Rightarrow y_1=C_1\e^t+C_2\e^{-t}$, 97 | 取 $y_1=\e^t$, 得 $y_3=\e^t$, 取 $y_1=\e^{-t}$, 得 $y_3=-\e^{-t}$, 98 | 不妨取基解矩阵为 $\begin{pmatrix}\e^t&0&\e^{-t}\\0&\e^t&0\\\e^t&0&-\e^{-t}\end{pmatrix}$, 则通解为 99 | \[\bm{y}(t)=C_1\begin{pmatrix}\e^t\\0\\\e^t\end{pmatrix}+C_2\begin{pmatrix}0\\\e^t\\0\end{pmatrix}+C_3\begin{pmatrix}\e^{-t}\\0\\-\e^{-t}\end{pmatrix}.\qedhere\] 100 | \end{solution} 101 | 102 | 103 | 104 | \begin{exercise} 105 | 求解非齐次线性微分方程组的初值问题: 106 | 107 | (1) $\begin{cases} 108 | \frac{\diff x}{\diff t} = 1-\frac{2}{t}x,\quad\frac{\diff y}{\diff t}=x+y-1+\frac{2}{t}x\quad(t>0),\\ 109 | x(1) = \frac{1}{3},\quad y(1)=-\frac{1}{3};\end{cases}$ 110 | 111 | (2) $\begin{cases} 112 | \frac{\diff x}{\diff t}=\frac{2t}{1+t^2}x,\quad\frac{\diff y}{\diff t}=-\frac{1}{t}y+x+t\quad(t>0),\\ 113 | x(1)=0,\quad y(1)=\frac{4}{3}. 114 | \end{cases}$. 115 | \end{exercise} 116 | 117 | \begin{solution} 118 | (1) 由 $\frac{\diff x}{\diff t}=1-\frac{2}{t}x$ 得 119 | $x=\e^{-\int\frac{2}{t}\diff t}\left(C+\int\e^{\int\frac{2}{t}\diff t}\diff t\right)=\frac{t}{3}+\frac{C}{t^2}$, 120 | 代入初值条件 $x(1)=\frac{1}{3}$ 得 $x(t)=\frac{t}{3}$. 121 | 又 $\frac{\diff}{\diff t}(x+y)=x+y$ 且 $(x+y)(1)=0$, 122 | 故 $x+y\equiv 0$, 故 $y(t)=-\frac{t}{3}$. 123 | 124 | (2) 由 $\frac{\diff x}{\diff t}=\frac{2t}{1+t^2}x$ 得 $x(t)=C(1+t^2)$, 125 | 代入初值条件 $x(1)=0$ 得 $x(t)\equiv 0$, 126 | 故 $\frac{\diff y}{\diff t}=-\frac{y}{t}+t$, 127 | 解得 $y(t)=\frac{C}{t}+\frac{t^2}{3}$, 128 | 代入初值条件 $y(1)=\frac{4}{3}$ 得 $y(t)=\frac{1}{t}+\frac{t^2}{3}$. 129 | \end{solution} 130 | 131 | 132 | 133 | \begin{exercise} 134 | 试证向量函数组 135 | \[\begin{pmatrix}1\\0\\0\end{pmatrix},\begin{pmatrix}x\\0\\0\end{pmatrix},\begin{pmatrix}x^2\\0\\0\end{pmatrix}\] 136 | 在任意区间 $a0,\exists N>0,\forall m,n>N$, 有 248 | \[\|\bm{A}_m-\bm{A}_n\| = \sum_{i,j=1}^n \bigl|a_{ij}^{(m)} - a_{ij}^{(n)}\bigr|<\epsilon.\] 249 | 故对任意给定的 $i,j$, 序列 $\bigl(a_{ij}^{(n)}\bigr)_{n\geq 1}$ 250 | 是 $\mathbb{R}$ 中的 Cauchy 序列, 必收敛, 记之为 $a_{ij}^{(n)}\to a_{ij}$ $(n\to\infty)$, 251 | 则 $\bm{A}_n\to \bm{A}=(a_{ij})_{n\times n}$, 因此 $(\mathcal{M},\|\cdot\|)$ 是 Banach 空间. 252 | \end{proof} 253 | 254 | 255 | 256 | \begin{proposition}[教材 Page 171] 257 | 矩阵 $\bm{A}$ 的幂级数 258 | \[\bm{E} + \bm{A} + \frac{1}{2!}\bm{A}^2 + \cdots + 259 | \frac{1}{k!}\bm{A}^k + \cdots\] 260 | 是绝对收敛的. 261 | \end{proposition} 262 | 263 | \begin{proof} 264 | 因为 265 | \begin{align*} 266 | & \|\bm{E}\|+\|\bm{A}\|+\|\frac{1}{2!}\bm{A}^2\|+\cdots+\|\frac{1}{k!}\bm{A}^k\|+\cdots\\ 267 | \leq{} & \|\bm{E}\|+\|\bm{A}\|+\frac{1}{2!}\|\bm{A}\|^2+\cdots+\frac{1}{k!}\|\bm{A}\|^k+\cdots\\ 268 | ={} & n+\e^{\|\bm{A}\|}-1<\infty, 269 | \end{align*} 270 | 故矩阵 $\bm{A}$ 的幂级数 271 | $\bm{E}+\bm{A}+\frac{1}{2!}\bm{A}^2+\cdots+\frac{1}{k!}\bm{A}^k+\cdots$ 绝对收敛. 272 | 273 | 结合泛函分析中定理: 赋范空间完备当且仅当绝对收敛级数必收敛. 274 | 故幂级数 $\bm{E}+\bm{A}+\frac{1}{2!}\bm{A}^2+\cdots+\frac{1}{k!}\bm{A}^k+\cdots$收敛, 275 | 将其和记为 $\e^{\bm{A}}$. 276 | \end{proof} 277 | 278 | 279 | 280 | \begin{proposition} 281 | 矩阵指数函数有下面的性质: 282 | \begin{enumerate}[1)] 283 | \item 若矩阵 $\bm{A}$ 和 $\bm{B}$ 是可交换的 (即 $\bm{AB}=\bm{BA}$), 则 284 | \[\e^{\bm{A}+\bm{B}} = \e^{\bm{A}}\e^{\bm{B}};\] 285 | \item 对任何矩阵 $\bm{A}$, 指数函数 $\e^{\bm{A}}$ 是可逆的, 且 286 | \[\bigl(\e^{\bm{A}}\bigr)^{-1} = \e^{-\bm{A}};\] 287 | \item 若 $\bm{P}$ 是一个非奇异的 $n$ 阶矩阵, 则 288 | \[\e^{\bm{P}\bm{A}\bm{P}^{-1}} = \bm{P}\e^{\bm{A}}\bm{P}^{-1}.\] 289 | \end{enumerate} 290 | \end{proposition} 291 | 292 | \begin{proof} 293 | (1) 294 | \[\begin{split} 295 | \e^{\bm{A}}\e^{\bm{B}} 296 | & = \left(\sum_{k=0}^{\infty}\frac{\bm{A}^k}{k!}\right)\left(\sum_{k=0}^{\infty}\frac{\bm{B}^k}{k!}\right) 297 | = \sum_{k=0}^{\infty}\sum_{i+j=k}\frac{\bm{A}^i}{i!}\frac{\bm{B}^j}{j!} 298 | = \sum_{k=0}^{\infty}\sum_{i=0}^k\frac{\bm{A}^i\bm{B}^{k-i}}{i!(k-i)!} \\ 299 | & = \sum_{k=0}^{\infty}\sum_{i=0}^k\frac{1}{k!}C_k^i\bm{A}^i\bm{B}^{k-i}=\sum_{k=0}^{\infty}\frac{1}{k!}(\bm{A}+\bm{B})^k=\e^{\bm{A}+\bm{B}}. 300 | \end{split}\] 301 | 302 | (2) 由于 $\bm{A}$ 与 $-\bm{A}$ 可交换, 故 303 | \[\e^{\bm{A}}\e^{-\bm{A}}=\e^{\bm{0}} = \bm{E}.\] 304 | 所以 $\left(\e^{\bm{A}}\right)^{-1}=\e^{-\bm{A}}$. 305 | 306 | (3) 307 | \[\e^{\bm{P}\bm{A}\bm{P}^{-1}}=\sum_{k=0}^{\infty}\frac{\left(\bm{P}\bm{A}\bm{P}^{-1}\right)^k}{k!}=\sum_{k=0}^{\infty}\frac{\bm{P}\bm{A}^k\bm{P}^{-1}}{k!}=\bm{P}\e^{\bm{A}}\bm{P}^{-1}.\qedhere\] 308 | \end{proof} 309 | 310 | 311 | 本节引进新的概念: 矩阵指数函数 $\e^{\bm{A}}$, 312 | 容易证明 $\e^{x\bm{A}}$ 是常系数齐次线性微分方程组 313 | $\displaystyle\frac{\diff\bm{y}}{\diff x}=\bm{A}\bm{y}$ 的标准基解矩阵, 314 | 然后利用若尔当标准型找到实际计算基解矩阵 $\e^{x\bm{A}}$ 的一个方法: 315 | \[\e^{x\bm{A}}=\e^{x\bm{P}\bm{J}\bm{P}^{-1}}=\bm{P}\e^{x\bm{J}}\bm{P}^{-1}.\] 316 | 从而 317 | \[\e^{x\bm{A}}\bm{P}=\bm{P}\e^{x\bm{J}} 318 | =\bm{P}\begin{pmatrix}\e^{x\bm{J}_1}&&&\\&\e^{x\bm{J}_2}&&\\&&\ddots&\\&&&\e^{x\bm{J}_m} 319 | \end{pmatrix}\] 320 | 也是常系数齐次线性微分方程组的一个基解矩阵, 但是上述结果计算量较大, 将之再细致分析, 得到下面求基解矩阵的具体计算方法: 321 | 322 | 由 $|\bm{A}-\lambda\bm{E}|=0$ 求出特征值 $\lambda$, 分两种情况: 323 | 324 | (i)若全为单根 $\lambda_1,\cdots,\lambda_n$, 325 | 则基解矩阵为 $\bmitPhi(x)=\left(\e^{\lambda_1x}\bm{r}_1,\e^{\lambda_2x}\bm{r}_2,\cdots,\e^{\lambda_nx}\bm{r}_n\right)$, 326 | 其中 $\bm{r}_i$ 是与 $\lambda_i$ 对应的特征向量. 327 | 328 | (ii)若有重根, 设 $\lambda_1,\cdots,\lambda_s$ 相应的重数分别为 $n_1,\cdots,n_s$, 329 | 则由 $(\bm{A}-\lambda_i\bm{E})^{n_i}\bm{r}=0$ 330 | 算出 $n_i$ 个线性无关的特征向量 $\bm{r}_{10}^{(i)},\cdots,\bm{r}_{n_i0}^{(i)}$, 再由 331 | \[\bm{r}_{jk}^{(i)}=(\bm{A}-\lambda_i\bm{E})\bm{r}_{j,k-1}^{(i)} 332 | \quad (k=1,2,\cdots,n_i-1;j=1,2,\cdots,n_i;i=1,2,\cdots,s)\] 333 | 算出其它所需向量, 则基解矩阵为 334 | \[\left(\e^{\lambda_1x}P_1^{(1)}(x),\cdots,\e^{\lambda_1x}P_{n_1}^{(1)}(x);\cdots; 335 | \e^{\lambda_sx}P_1^{(s)}(x),\cdots,\e^{\lambda_sx}P_{n_s}^{(s)}(x)\right),\] 336 | 其中 $\displaystyle P_j^{(i)}(x)=\bm{r}_{j0}^{(i)}+\frac{x}{1!}\bm{r}_{j1}^{(i)}+\frac{x^2}{2!}\bm{r}_{j2}^{(i)}+\cdots+\frac{x^{n_i-1}}{(n_i-1)!}\bm{r}_{j,n_i-1}^{(i)}$. 337 | 338 | 339 | 340 | \subsection{习题} 341 | 342 | 343 | 344 | \begin{exercise} 345 | 求出常系数齐次线性微分方程组(6.25)的通解, 其中的矩阵 $\bm{A}$ 分别为: 346 | \begin{enumerate} 347 | \item $\displaystyle\begin{pmatrix}3&4\\5&2\end{pmatrix}$; 348 | \item $\displaystyle\begin{pmatrix}0&a\\-a&0\end{pmatrix}$; 349 | \item $\displaystyle\begin{pmatrix}-1&1&0\\0&-1&0\\1&0&-4\end{pmatrix}$; 350 | \item $\displaystyle\begin{pmatrix}-5&-10&-20\\5&5&10\\2&4&9\end{pmatrix}$; 351 | \item $\displaystyle\begin{pmatrix}1&\frac{2}{3}&-\frac{2}{3}\\0&\frac{2}{3}&\frac{1}{3}\\0&-\frac{1}{3}&\frac{4}{3}\end{pmatrix}$; 352 | \item $\displaystyle\begin{pmatrix}1&1&1&1\\1&1&-1&-1\\1&-1&1&-1\\1&-1&-1&1\end{pmatrix}$. 353 | \end{enumerate} 354 | \end{exercise} 355 | 356 | \begin{solution} 357 | (1) $|\bm{A}-\lambda\bm{E}|=(\lambda-7)(\lambda+2)=0\Rightarrow\lambda=7$ 或 $\lambda=-2$. 358 | 359 | 当 $\lambda=7$ 时, $\begin{pmatrix}-4&4\\5&-5\end{pmatrix}\to\begin{pmatrix}1&-1\\0&0\end{pmatrix}$, 360 | 取特征向量为 $\bm{r}=\begin{pmatrix}1\\1\end{pmatrix}$. 361 | 362 | 当 $\lambda=-2$ 时, $\begin{pmatrix}5&4\\5&4\end{pmatrix}\to\begin{pmatrix}5&4\\0&0\end{pmatrix}$, 363 | 取特征向量为 $\bm{r}=\begin{pmatrix}4\\-5\end{pmatrix}$. 364 | 365 | 故基解矩阵为 366 | \[\bmitPhi(x)=\begin{pmatrix}\e^{7x}&4\e^{-2x}\\\e^{7x}&-5\e^{-2x}\end{pmatrix}.\] 367 | 368 | 故通解为 369 | \[\bm{y}=C_1\begin{pmatrix}\e^{7x}\\\e^{7x}\end{pmatrix} 370 | + C_2\begin{pmatrix}4\e^{-2x}\\-5\e^{-2x}\end{pmatrix}.\] 371 | 372 | (2) $|\bm{A}-\lambda\bm{E}|=\lambda^2+a^2=0\Rightarrow\lambda=\pm a\upi$. 373 | 374 | 当 $\lambda=a\upi$ 时, $\begin{pmatrix}-a\upi&a\\-a&-a\upi\end{pmatrix}\to\begin{pmatrix}-\upi&1\\0&0\end{pmatrix}$, 375 | 取特征向量为 $\bm{r}=\begin{pmatrix}1\\\upi\end{pmatrix}$. 376 | 377 | 当 $\lambda=-a\upi$ 时, $\begin{pmatrix}a\upi&a\\-a&a\upi\end{pmatrix}\to\begin{pmatrix}\upi&1\\0&0\end{pmatrix}$, 378 | 取特征向量为 $\bm{r}=\begin{pmatrix}\upi\\1\end{pmatrix}$. 379 | 380 | 故基解矩阵为 381 | \[\bmitPhi(x)=\begin{pmatrix}\e^{\upi ax}&\upi\e^{-\upi ax}\\\upi\e^{\upi ax}&\e^{-\upi ax}\end{pmatrix} 382 | \Rightarrow\widetilde{\bmitPhi}(x)=\begin{pmatrix}\cos ax&\sin ax\\-\sin ax&\cos ax\end{pmatrix}.\] 383 | 384 | 从而通解为 385 | \[\bm{y}=C_1\begin{pmatrix}\cos ax\\-\sin ax\end{pmatrix} 386 | + C_2\begin{pmatrix}\sin ax\\\cos ax\end{pmatrix}.\] 387 | 388 | (3) 由$\displaystyle\frac{\diff y_2}{\diff x}=-y_2$, 得 $y_2=C_1\e^{-x}$; 389 | 再由 $\displaystyle\frac{\diff y_1}{\diff x}=-y_1+y_2=-y_1+C_1\e^{-x}$, 得 390 | \[\displaystyle y_1=\e^{-\int\diff x}\left(C_2+\int C_1\e^{-x}\e^{\int\diff x}\diff x\right) 391 | = C_1x\e^{-x}+C_2\e^{-x}.\] 392 | 再由 $\displaystyle\frac{\diff y_3}{\diff x}=y_1-4y_3=-4y_3+C_1x\e^{-x}+C_2\e^{-x}$, 得 393 | \[\displaystyle y_3=\e^{-\int4\diff x}\left(C_3+\int\left(C_1x\e^{-x}+C_2\e^{-x}\right) 394 | \e^{\int4\diff x}\diff x\right)=C_1\left(\frac{x}{3}-\frac{1}{9}\right)\e^{-x} 395 | +\frac{1}{3}C_2\e^{-x}+C_3\e^{-4x}.\] 396 | 故通解为 397 | \[\bm{y}=C_1\begin{pmatrix}x\\1\\\frac{x}{3}-\frac{1}{9}\end{pmatrix}\e^{-x}+C_2 398 | \begin{pmatrix}1\\0\\\frac{1}{3}\end{pmatrix}\e^{-x} 399 | +C_3\begin{pmatrix}0\\0\\1\end{pmatrix}\e^{-4x}.\] 400 | 401 | (4) $\displaystyle|\bm{A}-\lambda\bm{E}|=\begin{vmatrix}-5-\lambda&-10&-20\\5&5-\lambda&10\\2&4&9-\lambda\end{vmatrix}=-(\lambda-5)(\lambda^2-4\lambda+5)=0\Rightarrow\lambda=5,2\pm\upi$. 402 | 403 | 当 $\lambda=5$ 时, 404 | $\begin{pmatrix}-10&-10&-20\\5&0&10\\2&4&4\end{pmatrix}\to\begin{pmatrix}1&0&2\\0&1&0\\0&0&0\end{pmatrix}$, 405 | 取特征向量为 $\bm{r}=\begin{pmatrix}2\\0\\-1\end{pmatrix}$. 406 | 407 | 当 $\lambda=2+\upi$ 时, 408 | $\begin{pmatrix}-7-\upi&-10&-20\\5&3-\upi&10\\2&4&7-\upi\end{pmatrix}\to\begin{pmatrix}1&0&\frac{1}{2}(3+\upi)\\0&1&\frac{1}{2}(2-\upi)\\0&0&0\end{pmatrix}$ 409 | (这里的矩阵行变换也不是很复杂, 主要就是凑出 $1$), 410 | 取特征向量为 $\bm{r}=\begin{pmatrix}3+\upi\\2-\upi\\-2\end{pmatrix}$, 411 | 故对于特征值 $\lambda=2-\upi$ 可以取到特征向量 $\bm{r}=\begin{pmatrix}3-\upi\\2+\upi\\-2\end{pmatrix}$. 412 | 413 | 故基解矩阵为 414 | \[\begin{split} 415 | &\bmitPhi(x)=\begin{pmatrix}2\e^{5x}&(3+\upi)\e^{(2+\upi)x}&(3-\upi)\e^{(2-\upi)x}\\0&(2-\upi)\e^{(2+\upi)x}&(2+\upi)\e^{(2-\upi)x}\\-\e^{5x}&-2\e^{(2+\upi)x}&-2\e^{(2-\upi)x}\end{pmatrix}\\ 416 | \Rightarrow&\widetilde{\bmitPhi}(x)\begin{pmatrix}2\e^{5x}&(3\cos x-\sin x)\e^{2x}&(\cos x+3\sin x)\e^{2x}\\0&(2\cos x+\sin x)\e^{2x}&(-\cos x+2\sin x)\e^{2x}\\\e^{5x}&-2\cos x\e^{2x}&-2\sin x\e^{2x}\end{pmatrix}. 417 | \end{split}\] 418 | 从而通解为 419 | \[\bm{y} = C_1\begin{pmatrix}2\\0\\-1\end{pmatrix}\e^{5x} 420 | + C_2\begin{pmatrix}3\cos x-\sin x\\2\cos x+\sin x\\-2\cos x\end{pmatrix}\e^{2x} 421 | + C_3\begin{pmatrix}\cos x+3\sin x\\-\cos x+2\sin x\\-2\sin x\end{pmatrix}\e^{2x}.\] 422 | 423 | (5) $|\bm{A}-\lambda\bm{E}|=\begin{vmatrix}1-\lambda&\frac{2}{3}&-\frac{2}{3}\\0&\frac{2}{3}-\lambda&\frac{1}{3}\\0&-\frac{1}{3}&\frac{4}{3}-\lambda\end{vmatrix}=-(\lambda-1)^3=0\Rightarrow\lambda_{1,2,3}=1$, 424 | 由于 $(\bm{A}-\bm{E})^3=\bm{0}$, 425 | 故由 $(\bm{A}-\bm{E})^3\bm{r}=\bm{0}$ 得到三个线性无关的特征向量 426 | $\bm{r}_{10}=\begin{pmatrix}1\\0\\0\end{pmatrix},\bm{r}_{20}=\begin{pmatrix}0\\1\\0\end{pmatrix},\bm{r}_{30}=\begin{pmatrix}0\\0\\1\end{pmatrix}$, 427 | 且 $\bm{r}_{11}=\bm{r}_{12}=\bm{0}$, $\bm{r}_{21}=\begin{pmatrix}\frac{2}{3}\\-\frac{1}{3}\\-\frac{1}{3}\end{pmatrix}$, 428 | $\bm{r}_{22}=\bm{0}$, $\bm{r}_{31}=\begin{pmatrix}-\frac{2}{3}\\\frac{1}{3}\\\frac{1}{3}\end{pmatrix}$, 429 | $\bm{r}_{32}=\bm{0}$. 430 | 431 | 故基解矩阵为 432 | \[\bmitPhi(x)= 433 | \begin{pmatrix} 434 | \e^x &\frac{2}{3}x\e^x & -\frac{2}{3}x\e^x \\ 435 | 0 &\left(1-\frac{1}{3}x\right)\e^x & \frac{1}{3}x\e^x \\ 436 | 0 &-\frac{1}{3}x\e^x & \left(1+\frac{1}{3}x\right)\e^x 437 | \end{pmatrix}.\] 438 | 439 | 故通解为 440 | \[\bm{y} = C_1\begin{pmatrix}1\\0\\0\end{pmatrix}\e^x 441 | + C_2\begin{pmatrix}\frac{2}{3}x\\1-\frac{1}{3}x\\-\frac{1}{3}x\end{pmatrix}\e^x 442 | + C_3\begin{pmatrix}-\frac{2}{3}x\\\frac{1}{3}x\\1+\frac{1}{3}x\end{pmatrix}\e^x.\] 443 | 444 | (6) $|\bm{A}-\lambda\bm{E}|=\begin{vmatrix}1-\lambda&1&1&1\\1&1-\lambda&-1&-1\\1&-1&1-\lambda&-1\\1&-1&-1&1-\lambda\end{vmatrix}=(\lambda+2)(\lambda-2)^3=0\Rightarrow\lambda_{1,2,3}=2,\lambda_4=-2$. 445 | 446 | 当 $\lambda=2$时, $(\bm{A}-2\bm{E})^3=\begin{pmatrix}-16&16&16&16\\16&-16&-16&-16\\16&-16&-16&-16\\16&-16&-16&-16\end{pmatrix}\to\begin{pmatrix}1&-1&-1&-1\\0&0&0&0\\0&0&0&0\\0&0&0&0\end{pmatrix}$, 由$(\bm{A}-2\bm{E})^3\bm{r}=0$ 447 | 得到三个线性无关的特征向量 $\bm{r}_{10}=\begin{pmatrix}1\\1\\0\\0\end{pmatrix}$, 448 | $\bm{r}_{20}=\begin{pmatrix}1\\0\\1\\0\end{pmatrix}$, 449 | $\bm{r}_{30}=\begin{pmatrix}1\\0\\0\\1\end{pmatrix}$, 450 | 且 $\bm{r}_{ij}=\bm{0}(i=1,2,3;j=1,2)$. 451 | 452 | 当 $\lambda=-2$时, $\bm{A}+2\bm{E}=\begin{pmatrix}3&1&1&1\\1&3&-1&-1\\1&-1&3&-1\\1&-1&-1&3\end{pmatrix}\to\begin{pmatrix}1&0&0&1\\0&1&0&-1\\0&0&1&-1\\0&0&0&0\end{pmatrix}$, 453 | 由 $(\bm{A}+2\bm{E})\bm{r}=\bm{0}$得特征向量$\bm{r}=\begin{pmatrix}-1\\1\\1\\1\end{pmatrix}$. 454 | 455 | 故基解矩阵为 456 | \[\bmitPhi(x) = 457 | \begin{pmatrix} 458 | \e^{2x}&\e^{2x}&\e^{2x}&-\e^{-2x} \\ 459 | \e^{2x}&0&0&\e^{-2x} \\ 460 | 0&\e^{2x}&0&\e^{-2x} \\ 461 | 0&0&\e^{2x}&\e^{-2x} 462 | \end{pmatrix}.\] 463 | 464 | 故通解为 465 | 466 | \[\bm{y} = 467 | C_1\begin{pmatrix}1\\1\\0\\0\end{pmatrix}\e^{2x} 468 | + C_2\begin{pmatrix}1\\0\\1\\0\end{pmatrix}\e^{2x}+C_3\begin{pmatrix}1\\0\\0\\1\end{pmatrix}\e^{2x} 469 | + C_4\begin{pmatrix}-1\\1\\1\\1\end{pmatrix}\e^{-2x}.\qedhere\] 470 | \end{solution} 471 | 472 | 473 | 474 | \begin{exercise} 475 | 求出常系数非齐次线性微分方程组 (6.24) 的通解, 其中: 476 | \begin{enumerate}[(1)] 477 | \item $\bm{A}=\begin{pmatrix}2&1\\0&2\end{pmatrix}, 478 | \quad\bm{f}(x)=\begin{pmatrix}1\\0\end{pmatrix}$; 479 | \item $\bm{A}=\begin{pmatrix}0&-n^2\\-n^2&0\end{pmatrix}, 480 | \quad\bm{f}(x)=\begin{pmatrix}\cos nx\\\sin nx\end{pmatrix}$; 481 | \item $\bm{A}=\begin{pmatrix}2&-1\\1&0\end{pmatrix}, 482 | \quad\bm{f}(x)=\begin{pmatrix}0\\2\e^x\end{pmatrix}$; 483 | \item $\bm{A}=\begin{pmatrix}2&1&-2\\-1&0&0\\1&1&-1\end{pmatrix}, 484 | \quad\bm{f}(x)=\begin{pmatrix}2-x\\0\\1-x\end{pmatrix}$; 485 | \item $\bm{A}=\begin{pmatrix}-1&-1&0\\0&-1&-1\\0&0&-1\end{pmatrix}, 486 | \quad\bm{f}(x)=\begin{pmatrix}x^2\\2x\\x\end{pmatrix}$. 487 | \end{enumerate} 488 | \end{exercise} 489 | 490 | \begin{solution} 491 | (1) $|\bm{A}-\lambda\bm{E}|=(\lambda-2)^2=0\Rightarrow\lambda_{1,2}=2$. 492 | 由 $(\bm{A}-2\bm{E})^2\bm{r}=0$ 得到两个线性无关的特征向量 493 | $\bm{r}_{10}=\begin{pmatrix}1\\0\end{pmatrix}$, 494 | $\bm{r}_{20}=\begin{pmatrix}0\\1\end{pmatrix}$, 495 | 且 $\bm{r}_{11}=\begin{pmatrix}0\\0\end{pmatrix},\bm{r}_{21}=\begin{pmatrix}1\\0\end{pmatrix}$, 496 | 故基解矩阵为 497 | \[\bmitPhi(x)=\begin{pmatrix}\e^{2x}&x\e^{2x}\\0&\e^{2x}\end{pmatrix}.\] 498 | 499 | 通解为 500 | \begin{align*} 501 | \bm{y} 502 | & = \bmitPhi(x)\bm{c}+\bmitPhi(x)\int\bmitPhi^{-1}(x)\bm{f}(x)\diff x\\ 503 | & = C_2\begin{pmatrix}1\\0\end{pmatrix}\e^{2x} 504 | + C_2\begin{pmatrix}x\\1\end{pmatrix}\e^{2x}-\begin{pmatrix}\frac{1}{2}\\0\end{pmatrix}. 505 | \end{align*} 506 | 507 | (2) $|\bm{A}-\lambda\bm{E}|=\lambda^2-n^4=0\Rightarrow\lambda=\pm n^2$. 508 | 509 | 当 $\lambda=n^2$ 时, $\begin{pmatrix}-n^2&-n^2\\-n^2&-n^2\end{pmatrix}\to\begin{pmatrix}1&1\\0&0\end{pmatrix}$, 510 | 取特征向量为 $\bm{r}=\begin{pmatrix}1\\-1\end{pmatrix}$. 511 | 512 | 当 $\lambda=-n^2$ 时, $\begin{pmatrix}n^2&-n^2\\-n^2&n^2\end{pmatrix}\to\begin{pmatrix}1&-1\\0&0\end{pmatrix}$, 513 | 取特征向量为 $\bm{r}=\begin{pmatrix}1\\1\end{pmatrix}$. 514 | 515 | 故基解矩阵为 516 | \[\bmitPhi(x)=\begin{pmatrix}\e^{n^2x}&\e^{-n^2x}\\-\e^{n^2x}&\e^{-n^2x}\end{pmatrix}.\] 517 | 518 | 其逆矩阵为 519 | \[\bmitPhi^{-1}(x) 520 | = \frac{1}{2}\begin{pmatrix}\e^{-n^2x}&-\e^{-n^2x}\\\e^{n^2x}&\e^{n^2x}\end{pmatrix}.\] 521 | 故 522 | \[\bmitPhi(x)^{-1}\bm{f}(x) 523 | = \frac{1}{2}\begin{pmatrix}\e^{-n^2x}\cos nx-\e^{-n^2x}\sin nx\\\e^{n^2x}\cos nx+\e^{n^2x}\sin nx\end{pmatrix}.\] 524 | 利用不定积分公式 525 | \[\begin{cases}\displaystyle 526 | \int\e^{\alpha x}\sin\beta x\diff x=\frac{\alpha}{\alpha^2+\beta^2}\e^{\alpha x}\sin\beta x-\frac{\beta}{\alpha^2+\beta^2}\e^{\alpha x}\cos\beta x+C, \\ 527 | \displaystyle\int\e^{\alpha x}\cos\beta x\diff x=\frac{\alpha}{\alpha^2+\beta^2}\e^{\alpha x}\cos\beta x+\frac{\beta}{\alpha^2+\beta^2}\e^{\alpha x}\sin\beta x+C 528 | \end{cases}\] 529 | 得 530 | \[\int\bmitPhi(x)^{-1}\bm{f}(x)\diff x=\frac{1}{2}\begin{pmatrix}\frac{n+1}{n^3+n}\e^{-n^2x}\sin nx+\frac{-n+1}{n^3+n}\e^{-n^2x}\cos nx\\ 531 | \frac{n+1}{n^3+n}\e^{n^2x}\sin nx+\frac{n-1}{n^3+n}\e^{n^2x}\cos nx 532 | \end{pmatrix}.\] 533 | 534 | 故 535 | \[\bmitPhi(x)\int\bmitPhi(x)^{-1}\bm{f}(x)\diff x 536 | =\begin{pmatrix} 537 | \frac{n+1}{n^3+n}\sin nx \\ 538 | \frac{n-1}{n^3+n}\cos nx 539 | \end{pmatrix}.\] 540 | 541 | 因此通解为 542 | \[\bm{y} = C_1\begin{pmatrix}1\\-1\end{pmatrix}\e^{n^2x} 543 | + C_2\begin{pmatrix}1\\1\end{pmatrix}\e^{-n^2x} 544 | + \begin{pmatrix} 545 | \frac{n+1}{n^3+n}\sin nx\\ 546 | \frac{n-1}{n^3+n}\cos nx 547 | \end{pmatrix}.\] 548 | 549 | (3) $|\bm{A}-\lambda\bm{E}|=\begin{vmatrix}2-\lambda&-1\\1&-\lambda\end{vmatrix}=(\lambda-1)^2=0\Rightarrow\lambda=1$, 550 | 由 $(\bm{A}-\bm{E})^2\bm{r}=0$ 得到两个线性无关得特征向量 551 | $\bm{r}_{10}=\begin{pmatrix}1\\0\end{pmatrix},\bm{r}_{20}=\begin{pmatrix}0\\1\end{pmatrix}$, 552 | 且 $\bm{r}_{11}=\begin{pmatrix}1\\1\end{pmatrix},\bm{r}_{21}=\begin{pmatrix}-1\\-1\end{pmatrix}$. 553 | 故基解矩阵为 554 | \[\bmitPhi(x)=\begin{pmatrix}(x+1)\e^x&-x\e^x\\x\e^x&(1-x)\e^x\end{pmatrix}.\] 555 | 故 556 | \[\bmitPhi(x)\int\bmitPhi(x)^{-1}\bm{f}(x)\diff x 557 | = \begin{pmatrix}-x^2\e^x\\(-x^2+2x)\e^x\end{pmatrix}.\] 558 | 故通解为 559 | \[\bm{y} = C_1\begin{pmatrix}x+1\\x\end{pmatrix}\e^x 560 | + C_2\begin{pmatrix}-x\\-x+1\end{pmatrix}\e^x 561 | + \begin{pmatrix}-x^2\e^x\\(-x^2+2x)\e^x\end{pmatrix}.\] 562 | 563 | (4) $|\bm{A}-\lambda\bm{E}|=\begin{vmatrix}2-\lambda&1&-2\\-1&-\lambda&0\\1&1&-1-\lambda\end{vmatrix}=-(\lambda-1)(\lambda^2+1)=0\Rightarrow\lambda=1,\pm\upi$. 564 | 565 | 当 $\lambda=1$ 时, $\begin{pmatrix}1&1&-2\\-1&-1&0\\1&1&-2\end{pmatrix}\to\begin{pmatrix}1&1&0\\0&0&1\\0&0&0\end{pmatrix}$, 566 | 取特征向量为 $\bm{r}=\begin{pmatrix}1\\-1\\0\end{pmatrix}$. 567 | 568 | 当 $\lambda=\upi$ 时, $\begin{pmatrix}2-\upi&1&-2\\-1&-\upi&0\\1&1&-1-\upi\end{pmatrix}\to\begin{pmatrix}1&0&-1\\0&1&\upi\\0&0&0\end{pmatrix}$, 569 | 取特征向量为 $\bm{r}=\begin{pmatrix}1\\\upi\\1\end{pmatrix}$. 570 | 571 | 当 $\lambda=-\upi$ 时, 可取特征向量为 $\bm{r}=\begin{pmatrix}\upi\\1\\\upi\end{pmatrix}$ 572 | (注意共轭特征向量为共轭特征值的特征向量). 573 | 574 | 故基解矩阵为 575 | \[\widetilde{\bmitPhi}(x) = 576 | \begin{pmatrix} 577 | \e^x&\e^{\upi x}&\upi\e^{-\upi x} \\ 578 | -\e^x&\upi\e^{\upi x}&\e^{-\upi x} \\ 579 | 0&\e^{\upi x}&\upi\e^{-\upi x} 580 | \end{pmatrix}\Rightarrow 581 | \bmitPhi(x) = 582 | \begin{pmatrix} 583 | \e^x&\cos x&\sin x \\ 584 | -\e^x&-\sin x&\cos x \\ 585 | 0&\cos x&\sin x 586 | \end{pmatrix}.\] 587 | 故 588 | \[\bmitPhi^{-1}(x) = 589 | \begin{pmatrix} 590 | \e^{-x}&0&-\e^{-x} \\ 591 | -\sin x&-\sin x&\sin x+\cos x \\ 592 | \cos x&\cos x&\sin x-\cos x 593 | \end{pmatrix}.\] 594 | 故 595 | \[\bmitPhi^{-1}(x)\bm{f}(x) = 596 | \begin{pmatrix} 597 | \e^{-x} \\ 598 | (1-x)\cos x-\sin x \\ 599 | (1-x)\sin x+\cos x 600 | \end{pmatrix}\Rightarrow 601 | \int\bmitPhi^{-1}(x)\bm{f}(x)\diff x = 602 | \begin{pmatrix} 603 | -\e^{-x} \\ 604 | (1-x)\sin x \\ 605 | (x-1)\cos x 606 | \end{pmatrix}.\] 607 | 故 608 | \[\bmitPhi(x)\int\bmitPhi^{-1}(x)\bm{f}(x)\diff x 609 | = \begin{pmatrix}-1\\x\\0\end{pmatrix}.\] 610 | 故通解为 611 | \[\bm{y} = C_1\begin{pmatrix}1\\-1\\0\end{pmatrix}\e^x 612 | + C_2\begin{pmatrix}\cos x\\-\sin x\\\cos x\end{pmatrix} 613 | + C_3\begin{pmatrix}\sin x\\\cos x\\\sin x\end{pmatrix} 614 | + \begin{pmatrix}-1\\x\\0\end{pmatrix}.\] 615 | 616 | (5) $|\bm{A}-\lambda\bm{E}|=-(\lambda+1)^3=0\Rightarrow\lambda_{1,2,3}=-1$. 617 | 618 | 由 $(\bm{A}+\bm{E})^3\bm{r}=0$ 得到三个线性无关的特征向量 619 | $\bm{r}_{10}=\begin{pmatrix}1\\0\\0\end{pmatrix}$, 620 | $\bm{r}_{20}=\begin{pmatrix}0\\1\\0\end{pmatrix}$, 621 | $\bm{r}_{30}=\begin{pmatrix}0\\0\\1\end{pmatrix}$, 622 | 且 $\bm{r}_{11}=\bm{r}_{12}=\bm{r}_{22}=\bm{0}$, 623 | $\bm{r}_{21}=\begin{pmatrix}-1\\0\\0\end{pmatrix}$, 624 | $\bm{r}_{31}=\begin{pmatrix}0\\-1\\0\end{pmatrix}$, 625 | $\bm{r}_{32}=\begin{pmatrix}1\\0\\0\end{pmatrix}$. 626 | 627 | 故基解矩阵为 628 | \[\bmitPhi(x) = 629 | \begin{pmatrix} 630 | \e^{-x}&-x\e^{-x}&\frac{1}{2}x^2\e^{-x} \\ 631 | 0&\e^{-x}&-x\e^{-x} \\ 632 | 0&0&\e^{-x} 633 | \end{pmatrix}.\] 634 | 故 635 | \[\bmitPhi^{-1}(x) = 636 | \begin{pmatrix} 637 | \e^x&x\e^x&\frac{1}{2}x^2\e^x \\ 638 | 0&\e^x&x\e^x \\ 639 | 0&0&\e^x 640 | \end{pmatrix}.\] 641 | 故 642 | \[\bmitPhi^{-1}(x)\bm{f}(x) = 643 | \begin{pmatrix} 644 | 3x^2\e^x+\frac{1}{2}x^3\e^x \\ 645 | 2x\e^x+x^2\e^x \\ 646 | x\e^x 647 | \end{pmatrix}\Rightarrow 648 | \int\bmitPhi^{-1}(x)\bm{f}(x)\diff x = 649 | \begin{pmatrix} 650 | \left(\frac{1}{2}x^3+\frac{3}{2}x^2-3x+3\right)\e^x \\ 651 | x^2\e^x \\ 652 | (x-1)\e^x 653 | \end{pmatrix}.\] 654 | 故 655 | \[\bmitPhi(x)\int\bmitPhi^{-1}(x)\bm{f}(x)\diff x=\begin{pmatrix}x^2-3x+3\\x\\x-1\end{pmatrix}.\] 656 | 657 | 故通解为 658 | \[\bm{y} = C_1\begin{pmatrix}1\\0\\0\end{pmatrix}\e^{-x} 659 | + C_2\begin{pmatrix}-x\\1\\0\end{pmatrix}\e^{-x} 660 | + C_3\begin{pmatrix}x^2\\-2x\\2\end{pmatrix}\e^{-x} 661 | + \begin{pmatrix}x^2-3x+3\\x\\x-1\end{pmatrix}.\qedhere\] 662 | \end{solution} 663 | 664 | 665 | 666 | \begin{exercise} 667 | 求出微分方程组 (6.24) 满足初值条件 $\bm{y}(0)=\bm{\eta}$ 的解, 其中: 668 | \begin{enumerate}[(1)] 669 | \item $\bm{A}=\begin{pmatrix}-5&-1\\1&-3\end{pmatrix}, 670 | \quad\bm{f}(x)=\begin{pmatrix}\e^x\\\e^{2x}\end{pmatrix}, 671 | \quad\bm{\eta}=\begin{pmatrix}1\\0\end{pmatrix}$; 672 | \item $\bm{A}=\begin{pmatrix}0&-2\\2&0\end{pmatrix}, 673 | \quad\bm{f}(x)=\begin{pmatrix}3x\\4\end{pmatrix}, 674 | \quad\bm{\eta}=\begin{pmatrix}2\\3\end{pmatrix}$; 675 | \item $\bm{A}=\begin{pmatrix}4&-3\\2&-1\end{pmatrix}, 676 | \quad\bm{f}(x)=\begin{pmatrix}\sin x\\-2\cos x\end{pmatrix}, 677 | \quad\bm{\eta}=\begin{pmatrix}0\\0\end{pmatrix}$; 678 | \item $\bm{A}=\begin{pmatrix}16&14&38\\-9&-7&-18\\-4&-4&-11\end{pmatrix}, 679 | \quad\bm{f}(x)=\begin{pmatrix}-2\e^{-x}\\-3\e^{-x}\\2\e^{-x}\end{pmatrix}, 680 | \quad\bm{\eta}=\begin{pmatrix}0\\0\\0\end{pmatrix}$. 681 | \end{enumerate} 682 | \end{exercise} 683 | 684 | \begin{solution} 685 | (1) $|\bm{A}-\lambda\bm{E}|=(\lambda+4)^2=0\Rightarrow\lambda_{1,2}=-4$. 686 | 687 | 由 $(\bm{A}+4\bm{E})^2\bm{r}=0$ 得两个线性无关的特征向量 688 | $\bm{r}_{10}=\begin{pmatrix}1\\0\end{pmatrix}$, 689 | $\bm{r}_{20}=\begin{pmatrix}0\\1\end{pmatrix}$, 690 | 且 $\bm{r}_{11}=\begin{pmatrix}-1\\1\end{pmatrix},\bm{r}_{21}=\begin{pmatrix}-1\\1\end{pmatrix}$. 691 | 692 | 故基解矩阵为 693 | \[\bmitPhi(x)=\begin{pmatrix}(1-x)\e^{-4x}&-x\e^{-4x}\\x\e^{-4x}&(1+x)\e^{-4x}\end{pmatrix}.\] 694 | 故 695 | \[\bmitPhi^{-1}(x)=\begin{pmatrix}(1+x)\e^{4x}&x\e^{4x}\\-x\e^{4x}&(1-x)\e^{4x}\end{pmatrix}.\] 696 | 故 697 | \[\bmitPhi^{-1}(x)\bm{f}(x)=\begin{pmatrix}(1+x)\e^{5x}+x\e^{6x}\\-x\e^{5x}+(1-x)\e^{6x}\end{pmatrix}\Rightarrow\int\bmitPhi^{-1}(x)\bm{f}(x)\diff x= 698 | \renewcommand\arraystretch{1.2}\begin{pmatrix}\frac{4}{25}\e^{5x}+\frac{1}{5}x\e^{5x}-\frac{1}{36}\e^{6x}+\frac{1}{6}x\e^{6x}\\-\frac{1}{5}x\e^{5x}+\frac{1}{25}\e^{5x}+\frac{7}{36}\e^{6x}-\frac{1}{6}x\e^{6x}\end{pmatrix}.\] 699 | 故 700 | \[\bmitPhi(x)\int\bmitPhi^{-1}(x)\bm{f}(x)\diff x= 701 | \renewcommand\arraystretch{1.2}\begin{pmatrix}\frac{4}{25}\e^x-\frac{1}{36}\e^{2x}\\\frac{1}{25}\e^x+\frac{7}{36}\e^{2x}\end{pmatrix}.\] 702 | 故通解为 703 | \[\bm{y}=C_1\begin{pmatrix}(1-x)\e^{-4x}\\x\e^{-4x}\end{pmatrix}+C_2\begin{pmatrix}-x\e^{-4x}\\(1+x)\e^{-4x}\end{pmatrix}+\renewcommand\arraystretch{1.2}\begin{pmatrix}\frac{4}{25}\e^x-\frac{1}{36}\e^{2x}\\\frac{1}{25}\e^x+\frac{7}{36}\e^{2x}\end{pmatrix}.\] 704 | 结合初值条件 705 | \[C_1\begin{pmatrix}1\\0\end{pmatrix}+C_2\begin{pmatrix}0\\1\end{pmatrix}+\begin{pmatrix}\frac{4}{25}-\frac{1}{36}\\\frac{1}{25}+\frac{7}{36}\end{pmatrix}=\begin{pmatrix}1\\0\end{pmatrix}\] 706 | 得 $C_1=\frac{781}{900},C_2=\frac{-211}{900}$, 故初值问题的解为 707 | \[\bm{y} = 708 | \frac{781}{900}\begin{pmatrix}1-x\\x\end{pmatrix}\e^{-4x} 709 | + \frac{-211}{900}\begin{pmatrix}-x\\1+x\end{pmatrix}\e^{-4x} 710 | + \begin{pmatrix} 711 | \frac{4}{25}\e^x-\frac{1}{36}\e^{2x} \\ 712 | \frac{1}{25}\e^x+\frac{7}{36}\e^{2x} 713 | \end{pmatrix}.\] 714 | 715 | (2) $|\bm{A}-\lambda\bm{E}|=\lambda^2+4=0\Rightarrow\lambda=\pm2\upi$. 716 | 717 | 当 $\lambda=2\upi$ 时, $\begin{pmatrix}-2\upi&-2\\2&-2\upi\end{pmatrix}\to\begin{pmatrix}1&-\upi\\0&0\end{pmatrix}$, 718 | 取特征向量为 $\bm{r}=\begin{pmatrix}\upi\\1\end{pmatrix}$. 719 | 720 | 当 $\lambda=-2\upi$ 时, 取特征向量为 $\bm{r}=\begin{pmatrix}1\\\upi\end{pmatrix}$. 721 | 722 | 故基解矩阵为 723 | \[\widetilde{\bmitPhi}(x) = 724 | \begin{pmatrix} 725 | \upi\e^{2\upi x}&\e^{-2\upi x} \\ 726 | \e^{2\upi x}&\upi\e^{-2\upi x} 727 | \end{pmatrix}\Rightarrow\bmitPhi(x) = 728 | \begin{pmatrix}\cos2x&-\sin2x\\\sin2x&\cos2x\end{pmatrix}.\] 729 | 故 730 | \[\bmitPhi^{-1}(x)=\begin{pmatrix}\cos2x&\sin2x\\-\sin2x&\cos2x\end{pmatrix}.\] 731 | 故 732 | \[\bmitPhi^{-1}(x)\bm{f}(x) = 733 | \begin{pmatrix}3x\cos2x+4\sin2x\\-3x\sin2x+4\cos2x\end{pmatrix} 734 | \Rightarrow 735 | \int\bmitPhi^{-1}(x)\bm{f}(x)\diff x = 736 | \begin{pmatrix} 737 | \frac{3}{2}x\sin2x-\frac{5}{4}\cos2x \\ 738 | \frac{3}{2}x\cos2x+\frac{5}{4}\sin2x 739 | \end{pmatrix}.\] 740 | 故 741 | \[\bmitPhi(x)\int\bmitPhi^{-1}(x)\bm{f}(x)\diff x = 742 | \begin{pmatrix}-\frac{5}{4}\\\frac{3}{2}x\end{pmatrix}.\] 743 | 故通解为 744 | \[\bm{y} = C_1\begin{pmatrix}\cos2x\\\sin2x\end{pmatrix} 745 | + C_2\begin{pmatrix}-\sin2x\\\cos2x\end{pmatrix} 746 | + \begin{pmatrix}-\frac{5}{4}\\\frac{3}{2}x\end{pmatrix}.\] 747 | 结合初值条件 748 | \[C_1\begin{pmatrix}1\\0\end{pmatrix} 749 | + C_2\begin{pmatrix}0\\1\end{pmatrix} 750 | + \begin{pmatrix}-\frac{5}{4}\\0\end{pmatrix} 751 | = \begin{pmatrix}2\\3\end{pmatrix},\] 752 | 得 $C_1=\frac{13}{4},C_2=3$, 故初值问题的解为 753 | \[\bm{y} = 754 | \frac{13}{4}\begin{pmatrix}\cos2x\\\sin2x\end{pmatrix} 755 | + 3\begin{pmatrix}-\sin2x\\\cos2x\end{pmatrix} 756 | + \begin{pmatrix}-\frac{5}{4}\\\frac{3}{2}x\end{pmatrix}.\] 757 | 758 | (3) $|\bm{A}-\lambda\bm{E}|=(\lambda-1)(\lambda-2)=0\Rightarrow\lambda_1=1,\lambda_2=2$. 759 | 760 | 当 $\lambda=1$ 时, $\begin{pmatrix}3&-3\\2&-2\end{pmatrix}\to\begin{pmatrix}1&-1\\0&0\end{pmatrix}$, 761 | 取特征向量为 $\bm{r}=\begin{pmatrix}1\\1\end{pmatrix}$. 762 | 763 | 当 $\lambda=2$ 时, $\begin{pmatrix}2&-3\\2&-3\end{pmatrix}\to\begin{pmatrix}2&-3\\0&0\end{pmatrix}$, 764 | 取特征向量为 $\bm{r}=\begin{pmatrix}3\\2\end{pmatrix}$. 765 | 766 | 故基解矩阵为 767 | \[\bmitPhi(x)=\begin{pmatrix}\e^x&3\e^{2x}\\\e^x&2\e^{2x}\end{pmatrix}.\] 768 | 故 769 | \[\bmitPhi^{-1}(x)=\begin{pmatrix}-2\e^{-x}&3\e^{-x}\\\e^{-2x}&-\e^{-2x}\end{pmatrix}.\] 770 | 故 771 | \[\bmitPhi^{-1}(x)\bm{f}(x) = 772 | \begin{pmatrix}-2\e^{-x}\sin x-6\e^{-x}\cos x\\\e^{-2x}\sin x+2\e^{-2x}\cos x\end{pmatrix}\] 773 | 积分得 774 | \[\int\bmitPhi^{-1}(x)\bm{f}(x)\diff x = 775 | \begin{pmatrix}-2\e^{-x}\sin x+4\e^{-x}\cos x\\-2\e^{-2x}\cos x\end{pmatrix}.\] 776 | 故 777 | \[\bmitPhi(x)\int\bmitPhi^{-1}(x)\bm{f}(x)\diff x = 778 | \begin{pmatrix}-2\sin x+\cos x\\-2\sin x+2\cos x\end{pmatrix}.\] 779 | 故通解为 780 | \[\bm{y} = C_1\begin{pmatrix}1\\1\end{pmatrix}\e^x 781 | + C_2\begin{pmatrix}3\\2\end{pmatrix}\e^{2x} 782 | + \begin{pmatrix}-2\sin x+\cos x\\-2\sin x+2\cos x\end{pmatrix}.\] 783 | 结合初值条件 784 | \[C_1\begin{pmatrix}1\\1\end{pmatrix} 785 | + C_2\begin{pmatrix}3\\2\end{pmatrix} 786 | + \begin{pmatrix}1\\2\end{pmatrix} 787 | = \begin{pmatrix}0\\0\end{pmatrix},\] 788 | 得 $C_1=-4$, $C_2=1$, 故初值问题的解为 789 | \[\bm{y} = 790 | - 4\begin{pmatrix}1\\1\end{pmatrix}\e^x 791 | + \begin{pmatrix}3\\2\end{pmatrix}\e^{2x} 792 | + \begin{pmatrix}-2\sin x+\cos x\\-2\sin x+2\cos x\end{pmatrix}.\] 793 | (4) $(-2x,-3x,2x)\e^{-x}$. 794 | \end{solution} 795 | 796 | 797 | 798 | \begin{exercise} 799 | 求解微分方程组 800 | \[\frac{\diff}{\diff t}\begin{pmatrix}x\\y\end{pmatrix} 801 | = \begin{pmatrix}a&-b\\b&a\end{pmatrix}\begin{pmatrix}x\\y\end{pmatrix},\] 802 | 其中 $a$ 和 $b$ 为实常数, 而且 $b\neq 0$. 803 | \end{exercise} 804 | 805 | \begin{solution} 806 | $\begin{vmatrix}a-\lambda&-b\\b&a-\lambda\end{vmatrix}=(a-\lambda)^2+b^2=0\Rightarrow\lambda=a\pm b\upi$. 807 | 808 | 当 $\lambda=a+b\upi$ 时, 809 | $\begin{pmatrix}-b\upi&-b\\b&-b\upi\end{pmatrix}\to\begin{pmatrix}\upi&1\\0&0\end{pmatrix}$, 810 | 取特征向量为 $\bm{r}=\begin{pmatrix}\upi\\1\end{pmatrix}$. 811 | 812 | 当 $\lambda=a-b\upi$ 时, 取特征向量为 $\bm{r}=\begin{pmatrix}1\\\upi\end{pmatrix}$. 813 | 814 | 故基解矩阵为 815 | \[\widetilde{\bmitPhi}(t) = 816 | \begin{pmatrix}\upi\e^{(a+b\upi)t}&\e^{(a-b\upi)t}\\\e^{(a+b\upi)t}&\upi\e^{(a-b\upi)t}\end{pmatrix} 817 | \Rightarrow\bmitPhi(t) = 818 | \begin{pmatrix}\e^{at}\cos bt&-\e^{at}\sin bt\\ \e^{at}\sin bt&\e^{at}\cos bt\end{pmatrix}.\] 819 | 故通解为 820 | \[\begin{pmatrix}x\\y\end{pmatrix} = 821 | C_1\begin{pmatrix}\e^{at}\cos bt\\\e^{at}\sin bt\end{pmatrix} 822 | + C_2\begin{pmatrix}-\e^{at}\sin bt\\\e^{at}\cos bt\end{pmatrix}.\qedhere\] 823 | \end{solution} 824 | 825 | 826 | 827 | \begin{exercise} 828 | 证明: 常系数齐次线性微分方程组 $\displaystyle\frac{\diff\bm{y}}{\diff x}=\bm{A}\bm{y}$ 829 | 的任何解当 $x\to+\infty$ 时都趋于零, 当且仅当它的系数矩阵 $\bm{A}$ 的所有特征根都具有负的实部. 830 | \end{exercise} 831 | 832 | \begin{proof} 833 | 方程的基解矩阵为 834 | \[\left(\e^{\lambda_1x}P_1^{(1)}(x),\cdots,\e^{\lambda_1x}P_{n_1}^{(1)}(x);\cdots; 835 | \e^{\lambda_sx}P_1^{(s)}(x),\cdots,\e^{\lambda_sx}P_{n_s}^{(s)}(x)\right).\] 836 | 故 837 | \[\begin{split} 838 | & \text{当\ }x\to+\infty\text{\ 时任何解都趋于零} \\ 839 | \iff & \e^{\lambda_ix}\to0\;(x\to+\infty)\quad (i=1,2,\cdots,s)\\ 840 | \iff & \Re(\lambda_i)<0\quad (i=1,2,\cdots,s).\qedhere 841 | \end{split}\] 842 | \end{proof} 843 | 844 | 845 | 846 | \section{高阶线性微分方程} 847 | 848 | 849 | 850 | \subsection{证明与总结} 851 | 852 | 853 | 854 | 对于高阶线性微分方程 855 | \[y^{(n)}+a_1(x)y^{(n-1)}+\cdots+a_{n-1}(x)y'+a_n(x)y=f(x),\] 856 | 令 $y_1=y,y_2=y',\cdots,y_n=y^{(n-1)}$, 记 $\bm{y}=(y_1,\cdots,y_n)^{\T}$ 则 857 | \[\frac{\diff\bm{y}}{\diff x} 858 | = \bm{A}(x)\bm{y}+\bm{f}(x) 859 | = \begin{pmatrix} 860 | 0&1&0&\cdots&0 \\ 861 | 0&0&1&\cdots&0 \\ 862 | \vdots&\vdots&\vdots&&\vdots \\ 863 | 0&0&0&\cdots&1 \\ 864 | -a_n(x)&-a_{n-1}(x)&-a_{n-2}(x)&\cdots&-a_1(x) 865 | \end{pmatrix} 866 | \begin{pmatrix} 867 | y_1\\y_2\\\vdots\\y_n 868 | \end{pmatrix}+\begin{pmatrix}0\\0\\\vdots\\f(x)\end{pmatrix}.\] 869 | 870 | (I)齐次方程: 有 $n$ 个线性无关的解 $\varphi_1(x),\cdots,\varphi_n(x)$, 解组的朗斯基行列式 871 | \[W(x)=W(x_0)\e^{-\int_{x_0}^xa_1(s)\diff s}.\] 872 | (一个运用: 二阶齐次线性微分方程组 $y''+p(x)y'+q(x)y=0$ 若知道一个特解可以求出通解). 873 | 874 | (II)非齐次方程: 通解为 875 | \[y=C_1\varphi_1(x)+\cdots+C_n\varphi_n(x)+\varphi^*(x),\] 876 | 其中特解 877 | \[\varphi^*(x)=\sum_{k=1}^n\varphi_k(x)\int_{x_0}^x\frac{W_k(s)}{W(s)}f(s)\diff s.\] 878 | 这里 $W(x)$ 是 $\varphi_1(x),\cdots,\varphi_n(x)$ 879 | 的 Wronsky 行列式, 而 $W_k(x)$ 是 $W(x)$ 中第 $n$ 行第 $k$ 列元素的代数余子式. 880 | 此公式既可以用前面的公式 881 | \[\bm{y}=\bmitPhi(x)\left(\bm{c}+\int_{x_0}^x\bmitPhi^{-1}(s)\bm{f}(s)\diff s\right)\] 882 | 取第一个分量导出, 也可以用常数变易法导出 (见习题 6). 883 | 884 | esp: 常系数高阶线性微分方程, 其系数矩阵为 885 | \[\bm{A} = 886 | \begin{pmatrix} 887 | 0 & 1 & 0 & \cdots & 0 \\ 888 | 0 & 0 & 1 & \cdots & 0 \\ 889 | \vdots&\vdots&\vdots&&\vdots \\ 890 | 0 & 0 & 0 & \cdots & 1 \\ 891 | -a_n&-a_{n-1}&-a_{n-2}&\cdots&-a_1 892 | \end{pmatrix}.\] 893 | 由 $|\bm{A}-\lambda\bm{E}|=\lambda^n+a_1\lambda^{n-1}+\cdots+a_{n-1}\lambda+a_n=0$ 894 | 算出特征值 $\lambda_1,\lambda_2,\cdots,\lambda_s$, 其重数分别为$n_1,n_2,\cdots,n_s$ 895 | $(n_1+n_2+\cdots+n_s=n)$, 则齐次方程基本解组为: 896 | \[\begin{cases} 897 | \e^{\lambda_1x},x\e^{\lambda_1x},\cdots,x^{n_1-1}\e^{\lambda_1x};\\ 898 | \cdots\cdots\cdots\\ 899 | \e^{\lambda_sx},x\e^{\lambda_sx},\cdots,x^{n_s-1}\e^{\lambda_sx}. 900 | \end{cases}\] 901 | 902 | 对于非齐次方程, 还需要求出特解, 一般用上述特解求解公式, 在 $f(x)$ 形式特殊时, 可以用待定系数法: 903 | 904 | 当 $f(x)=P_m(x)\e^{\mu x}$ 时, 取 905 | \[\varphi^*(x)=x^kQ_m(x)\e^{\mu x},\] 906 | 其中 $\mu$ 为 $k$ 重特征根. 907 | 908 | 当 $f(x)=[A_m(x)\cos(\beta x)+B_l(x)\sin(\beta x)]\e^{\alpha x}$ 时, 取 909 | \[\varphi^*(x)=x^k[C_n(x)\cos(\beta x)+D_n(x)\sin(\beta x)]\e^{\alpha x},\] 910 | 其中 $\alpha\pm\upi\beta$ 为 $k$ 重特征根, $n=\max\{m,l\}$. 911 | 912 | 913 | 914 | \subsection{习题} 915 | 916 | 917 | 918 | \begin{exercise} 919 | 证明函数组 920 | \[\varphi_1(x) = 921 | \begin{cases} 922 | x^2, & \text{当\ } x\geq 0, \\ 923 | 0, & \text{当\ } x<0; 924 | \end{cases}\quad 925 | \varphi_2(x)= 926 | \begin{cases} 927 | 0, & \text{当\ } x\geq 0, \\ 928 | x^2, & \text{当\ } x<0 929 | \end{cases}\] 930 | 在区间 $(-\infty,+\infty)$ 上线性无关, 但它们的朗斯基行列式恒等于零. 931 | 这与本节的定理 $6.2^*$ 是否矛盾? 如果并不矛盾, 那么它说明了什么? 932 | \end{exercise} 933 | 934 | \begin{proof} 935 | 设 $k_1\varphi_1(x)+k_2\varphi_2(x)=0, \forall x\in (-\infty,+\infty)$, 936 | 当 $x\geq 0$时, $k_1x^2=0\Rightarrow k_1=0$, 937 | 当 $x<0$ 时, $k_2x^2=0\Rightarrow k_2=0$, 938 | 故 $\varphi_1(x)$ 与 $\varphi_2(x)$ 在 $(-\infty,+\infty)$ 上线性无关. 939 | 这与定理 $6.2^*$ 不矛盾, 并说明不存在二阶齐次线性微分方程使得它以 $\varphi_1(x)$ 与 $\varphi_2(x)$ 为解组. 940 | \end{proof} 941 | 942 | 943 | 944 | \begin{exercise} 945 | 证明命题5. 946 | \end{exercise} 947 | 948 | \begin{proof} 949 | ($\Rightarrow$)显然 950 | 951 | ($\Leftarrow$) 设 $k_1\varphi_1(x)+\cdots+k_n\varphi_n(x)=0$, 则 952 | \[k_1\varphi_1'(x)+\cdots+k_n\varphi_n'(x)=0,\] 953 | \[\cdots\] 954 | \[k_1\varphi_1^{(n-1)}(x)+\cdots+k_n\varphi_n^{(n-1)}(x)=0.\] 955 | 由向量函数组线性无关即得 $k_i=0$ $(i=1,2,\cdots,n)$, 故 $\varphi_1(x),\cdots,\varphi_n(x)$ 线性无关. 956 | \end{proof} 957 | 958 | 959 | 960 | \begin{exercise} 961 | 考虑微分方程: $y''+q(x)y=0$. 962 | \begin{enumerate}[(1)] 963 | \item 设 $y=\varphi(x)$ 与 $y=\psi(x)$ 是它的两个解, 964 | 试证 $\varphi(x)$ 与 $\psi(x)$ 的朗斯基行列式恒等于一个常数. 965 | \item 设已知方程有一个特解为 $y=\e^x$, 试求这方程的通解, 并确定 $q(x)=$ ? 966 | \end{enumerate} 967 | \end{exercise} 968 | 969 | \begin{solution} 970 | (1) $W(x)=W(x_0)\e^{-\int_{x_0}^x0\diff s}=W(x_0)$. 971 | 972 | (2) 将 $y=\e^x$ 代入原方程得 $q(x)=-1$, 即原方程为 $y''-y=0$, 解得通解为 $y=C_1\e^x+C_2\e^{-x}$. 973 | \end{solution} 974 | 975 | 976 | 977 | \begin{exercise} 978 | 考虑微分方程 979 | \begin{equation} 980 | y''+p(x)y'+q(x)=0, \tag{$\star$} 981 | \end{equation} 982 | 其中 $p(x)$ 和 $q(x)$ 是区间 $I:a0$. 试利用适当的变换把它化成常系数的齐次线性微分方程. 1109 | \end{exercise} 1110 | 1111 | \begin{solution} 1112 | 令 $x=\e^t$, 则 1113 | \begin{align*} 1114 | & \frac{\diff y}{\diff x}=\frac{\diff y}{\diff t}\cdot\frac{\diff t}{\diff x}=\e^{-t}\frac{\diff y}{\diff t}, \\ 1115 | & \frac{\diff^2y}{\diff x^2}=\frac{\diff}{\diff t}\left(\frac{\diff y}{\diff x}\right)\cdot\frac{\diff t}{\diff x}=\e^{-2t}\left(\frac{\diff^2y}{\diff t^2}-\frac{\diff y}{\diff t}\right). 1116 | \end{align*} 1117 | 用归纳法可以证明 1118 | \begin{equation} 1119 | \frac{\diff^ky}{\diff x^k} = \e^{-kt}\left(\frac{\diff^ky}{\diff t^k} 1120 | + \beta_1\frac{\diff^{k-1}y}{\diff t^{k-1}} 1121 | + \cdots+\beta_{k-1}\frac{\diff y}{\diff t}\right). \tag{$\star$} 1122 | \end{equation} 1123 | 其中 $\beta_1,\beta_2,\beta_{k-1}$ 都是常数. 将其代入原方程就得到常系数齐次线性微分方程 1124 | \[\frac{\diff^ny}{\diff t^n}+b_1\frac{\diff^{n-1}y}{\diff t^{n-1}}+\cdots+b_{n-1}\frac{\diff y}{\diff t}+b_ny=0,\] 1125 | 其中 $b_1,b_2,\cdots,b_n$ 是常数. 求解之, 再代回原变量, 便可得原方程通解. 1126 | \end{solution} 1127 | 1128 | \begin{remark} 1129 | $(\star)$ 式其实不太精细, 事实上, 利用归纳法容易证明下列关系式 1130 | \[x^k\frac{\diff^ky}{\diff x^k} 1131 | = \frac{\diff}{\diff t}\left(\frac{\diff}{\diff t}-1\right) 1132 | \cdots\left(\frac{\diff}{\diff t}-k+1\right)y.\] 1133 | \end{remark} 1134 | 1135 | 1136 | \begin{exercise} 1137 | 求解有阻尼的弹簧振动方程 1138 | \[m\frac{\diff^2x}{\diff t^2}+r\frac{\diff x}{\diff t}+kx=0,\] 1139 | 其中 $m,r,k$ 都是正的常数. 并就 $\Delta=r^2-4mk$ 大于, 等于和小于零的不同情况, 说明相应解的物理意义. 1140 | \end{exercise} 1141 | 1142 | \begin{solution} 1143 | 特征方程为 1144 | \[m\lambda^2+r\lambda+k=0.\] 1145 | 特征根为 1146 | \[\lambda_1=\frac{-r+\sqrt{r^2-4mk}}{2m},\quad\lambda_2=\frac{-r-\sqrt{r^2-4mk}}{2m}.\] 1147 | \begin{enumerate}[(i)] 1148 | \item $\Delta>0$ 即大阻尼情形, $\lambda_2<\lambda_1<0$, 通解为 1149 | \[x(t)=C_1\e^{\lambda_1t}+C_2\e^{\lambda_2t},\] 1150 | 其中 $C_1,C_2$ 为任意常数. 此时 $\lim_{t\to\infty}x(t)=0$, 并且有 1151 | \begin{enumerate}[(a)] 1152 | \item 当常数 $C_1$ 和 $C_2$ 全为零时, 则 $x(t)\equiv 0$, 即弹簧静止; 1153 | \item 当常数 $C_1$ 和 $C_2$ 有且只有一个为零时, 则 $x(t)$ 保持定号, 即弹簧不能振动; 1154 | \item 当常数 $C_1$ 和 $C_2$ 都不为零时, 此时弹簧最多只能经过一次静止点, 亦即 1155 | \[x(t_0)=C_1\e^{\lambda_1t_0}+C_2\e^{\lambda_2t_0}=0\] 1156 | 当且仅当 $-1<\frac{C_1}{C_2}<0$ 异号, 1157 | 而且 $t_0=\frac{1}{\lambda_2-\lambda_1}\ln\left(-\frac{C_1}{C_2}\right)$. 1158 | \end{enumerate} 1159 | \item $\Delta<0$ 即小阻尼情形, 此时 $\lambda_1=\alpha+\upi\beta,\lambda_2=\alpha-\upi\beta$, 1160 | 其中 $\alpha=-\frac{r}{2m}<0,\beta=\frac{\sqrt{-\Delta}}{2m}>0$, 通解为 1161 | \[x(t)=\e^{\alpha t}(C_1\cos\beta t+C_2\sin\beta t)=A\e^{\alpha t}\cos(\beta t-\theta_0).\] 1162 | 故 $\lim_{t\to\infty}x(t)=0$, 且 1163 | \begin{enumerate}[(a)] 1164 | \item 当 $A=0$ 时, 弹簧静止; 1165 | \item 当 $A>0$ 时, 弹簧振动. 1166 | \end{enumerate} 1167 | \item $\Delta=0$ 即临界阻尼情形, 有两个相等的特征根 $\lambda_1=\lambda_2=-\frac{r}{2m}$, 通解为 1168 | \[x(t)=\e^{-\frac{r}{2m}t}(C_1+C_2t).\] 1169 | 此时 $\lim_{t\to\infty}x(t)=0$ 且 $x(t)$ 至多有一个零点, 故弹簧不振动.\qedhere 1170 | \end{enumerate} 1171 | \end{solution} 1172 | 1173 | 1174 | 1175 | \begin{exercise} 1176 | 求解弹簧振子在无阻尼下的强迫振动方程 1177 | \[m\frac{\diff^2x}{\diff t^2}+kx=p\cos\omega t,\] 1178 | 其中 $m,k,p$ 和 $\omega$ 都是正的常数. 1179 | 并对外加频率 $\omega\neq\omega_0$ 和 $\omega=\omega_0$ 两种不同的情况, 1180 | 说明解的物理意义, 这里 $\omega_0=\sqrt{\frac{k}{m}}$是弹簧振子的固有频率. 1181 | \end{exercise} 1182 | 1183 | \begin{solution} 1184 | 特征方程为 1185 | \[m\lambda^2+k=0.\] 1186 | 解得特征根为 $\lambda=\pm\sqrt{\frac{k}{m}}\upi=\pm\omega_0\upi$, 故相应齐次线性微分方程的解为 1187 | \[x(t)=C_1\cos\omega_0t+C_2\sin\omega_0t.\] 1188 | 当 $\omega\neq\omega_0$ 时, 方程有特解 $x(t)=A\cos\omega t+B\sin\omega t$, 1189 | 代入原方程得 $A=\frac{p}{k-m\omega^2}$, $B=0$, 故原方程通解为 1190 | \[x(t)=C_1\cos\omega_0t+C_2\sin\omega_0t+\frac{p}{k-m\omega^2}\cos\omega t.\] 1191 | 当 $\omega=\omega_0$时, 方程有特解 $x(t)=t(A\cos\omega_0t+B\sin\omega_0t)$, 1192 | 代入原方程得 $A=0,B=\frac{p}{2m\omega_0}$, 故原方程通解为 1193 | \[x(t)=C_1\cos\omega_0t+C_2\sin\omega_0t+\frac{p}{2m\omega_0}t\sin\omega_0 t.\] 1194 | 此时发生了共振. 1195 | \end{solution} 1196 | 1197 | 1198 | 1199 | \begin{exercise} 1200 | 求解下列常系数线性微分方程: 1201 | \begin{enumerate}[(1)] 1202 | \item $y''+y'-2y=2x,y(0)=0,y'(0)=1$; 1203 | \item $2y''-4y'-6y=3\e^{2x}$; 1204 | \item $y''+2y'=3+4\sin 2x$; 1205 | \item $y'''+3y'-4y=0$; 1206 | \item $y'''-2y''-3y'+10y=0$; 1207 | \item $y'''-3ay''+3a^2y'-a^3y=0$; 1208 | \item $y^{(4)}-4y'''+8y''-8y'+3y=0$; 1209 | \item $y^{(5)}+2y'''+y'=0$; 1210 | \item $y^{(4)}+2y''+y=\sin x,y(0)=1,y'(0)=-2,y''(0)=3,y'''(0)=0$; 1211 | \item $y^{(4)}+y=2\e^x,y(0)=y'(0)=y''(0)=y'''(0)=1$; 1212 | \item $y''-2y'+2y=4\e^x\cos x$; 1213 | \item $y''-5y'+6y=(12x-7)\e^{-x}$; 1214 | \item $x^2y''+5xy'+13y=0(x>0)$; 1215 | \item $(2x+1)^2y''-4(2x+1)y'+8y=0$. 1216 | \end{enumerate} 1217 | \end{exercise} 1218 | 1219 | \begin{solution} 1220 | (1) $\lambda^2+\lambda-2=0\Rightarrow\lambda_1=-2,\lambda_2=1$, 1221 | 设方程的特解为 $y=ax+b$, 代入原方程得 $a=-1,b=-\frac{1}{2}$, 故原方程的通解为 1222 | \[y=C_1\e^{-2x}+C_2\e^x-x-\frac{1}{2}.\] 1223 | 代入初值条件得 $C_1=-\frac{1}{2},C_2=1$, 故原方程的解为 1224 | \[y=-\frac{1}{2}\e^{-2x}+\e^x-x-\frac{1}{2}.\] 1225 | 1226 | (2) $2\lambda^2-4\lambda-6=0\Rightarrow\lambda_1=3,\lambda_2=-1$, 1227 | 设方程的特解为 $y=a\e^{2x}$, 代入原方程得 $a=-\frac{1}{2}$, 故原方程的通解为 1228 | \[y=C_1\e^{3x}+C_2\e^{-x}-\frac{1}{2}\e^{2x}.\] 1229 | 1230 | (3) $\lambda^2+2\lambda=0\Rightarrow\lambda_1=0,\lambda_2=-2$, 1231 | 设特解为 $y=Ax+B\cos 2x+C\sin2x$, 代入原方程得 $A=\frac{3}{2},B=C=-\frac{1}{2}$, 故原方程通解为 1232 | \[y=C_1+C_2\e^{-2c}+\frac{3}{2}x-\frac{1}{2}(\sin2x+\cos2x).\] 1233 | 1234 | (4) $\lambda^3+3\lambda-4=0\Rightarrow\lambda=1,\frac{-1\pm\sqrt{15}\upi}{2}$, 1235 | 故方程的实基本解组为 1236 | $\e^x$, $\e^{-\frac{1}{2}x}\cos\frac{\sqrt{15}}{2}x$, 1237 | $\e^{-\frac{1}{2}x}\sin\frac{\sqrt{15}}{2}x$, 故通解为 1238 | \[y=C_1\e^x+\left(C_2\cos\frac{\sqrt{15}}{2}x+C_3\sin\frac{\sqrt{15}}{2}x\right)\e^{-\frac{1}{2}x}.\] 1239 | 1240 | (5) $\lambda^3-2\lambda^2-3\lambda+10=0\Rightarrow\lambda=-2,2\pm\upi$, 1241 | 故实基本解组为 $\e^{-2x}$, $\e^{2x}\cos x$, $\e^{2x}\sin x$, 故通解为 1242 | \[y=C_1\e^{-2x}+(C_2\cos x+C_3\sin x)\e^{2x}.\] 1243 | 1244 | (6) $\lambda^3-3a\lambda^2+3a^2\lambda-a^3=(\lambda-a)^3=0\Rightarrow\lambda_{1,2,3}=a$, 故通解为 1245 | \[y=\left(C_1+C_2x+C_3x^2\right)\e^{ax}.\] 1246 | 1247 | (7) $\lambda^4-4\lambda^3+8\lambda^2-8\lambda+3=(\lambda-1)^2(\lambda^2-2\lambda+3)=0\Rightarrow\lambda_{1,2}=1,\lambda_3=1+\sqrt{2}\upi,\lambda_4=1-\sqrt{2}\upi$, 故通解为 1248 | \[y=(C_1+C_2x)\e^x+(C_3\cos\sqrt{2}x+C_4\sin\sqrt{2}x)\e^x.\] 1249 | 1250 | (8) $\lambda^5+2\lambda^3+\lambda=\lambda(\lambda^2+1)^2=0\Rightarrow\lambda_1=0,\lambda_{2,3}=\upi,\lambda_{4,5}=-\upi$, 1251 | 故复基本解组为 $1$, $\e^{\upi x}$, $x\e^{\upi x}$, $\e^{-\upi x}$, $x\e^{-\upi x}$, 1252 | 相应的实基本解组为 $1$, $\cos x$, $\sin x$, $x\cos x$, $x\sin x$, 故通解为 1253 | \[y=C_1+(C_2+C_3x)\cos x+(C_4+C_5x)\sin x.\] 1254 | 1255 | (9) $\lambda^4+2\lambda^2+1=(\lambda^2+1)^2=0\Rightarrow\lambda_{1,2}=\upi,\lambda_{3,4}=-\upi$, 1256 | 故对应齐次方程的通解为 1257 | \[\varphi(x)=C_1\sin x+C_2\cos x+C_3x\sin x+C_4x\cos x.\] 1258 | 设特解为 $\varphi^*(x)=x^2(A\cos x+B\sin x)$, 则 1259 | \[\begin{split} 1260 | \left(\varphi^*(x)\right)'&=2x(A\cos x+B\sin x)+x^2(-A\sin x+B\cos x), \\ 1261 | \left(\varphi^*(x)\right)''&=(2-x^2)(A\cos x+B\sin x)+4x(-A\sin x+B\cos x), \\ 1262 | \left(\varphi^*(x)\right)'''&=-6x(A\cos x+B\sin x)+(6-x^2)(-A\sin x+B\cos x), \\ 1263 | \left(\varphi^*(x)\right)^{(4)}&=(x^2-12)(A\cos x+B\sin x)-8x(-A\sin x+B\cos x). 1264 | \end{split}\] 1265 | 故 $(x^2-12+4-2x^2+x^2)(A\cos x+B\sin x)+(-8x+8x)(-A\sin x+B\cos x)=-8(A\cos x+B\sin x)=\sin x\Rightarrow A=0,B=-\frac{1}{8}$, 1266 | 故 $\varphi^*(x)=-\frac{1}{8}x^2\sin x$, 故原方程的通解为 1267 | \[y=C_1\sin x+C_2\cos x+C_3x\sin x+C_4x\cos x-\frac{1}{8}x^2\sin x.\] 1268 | 再代入初值条件 $y(0)=C_2=1$, $y'(0)=C_1+C_4=-2$, $y''(0)=-C_2+2C_3=3$, 1269 | $y'''(0)=-C_1-3C_4-\frac{3}{4}=0$, 1270 | 解得 $C_1=-\frac{21}{8},C_2=1,C_3=2,C_4=\frac{5}{8}$, 故原方程的解为 1271 | \[y=\left(-\frac{1}{8}x^2+2x-\frac{21}{8}\right)\sin x+\left(\frac{5}{8}x+1\right)\cos x.\] 1272 | 1273 | (10) $\lambda^4+1=0\Rightarrow\lambda=\e^{\upi\left(\frac{\pi}{4}+\frac{1}{2}k\pi\right)}$ 1274 | $(k=0,1,2,3)$, 1275 | 也即 1276 | $\lambda_1=\frac{\sqrt{2}}{2}+\frac{\sqrt{2}}{2}\upi$, 1277 | $\lambda_2=-\frac{\sqrt{2}}{2}+\frac{\sqrt{2}}{2}\upi$, 1278 | $\lambda_3=-\frac{\sqrt{2}}{2}-\frac{\sqrt{2}}{2}\upi$, 1279 | $\lambda_4=\frac{\sqrt{2}}{2}-\frac{\sqrt{2}}{2}\upi$, 1280 | 故相应的齐次线性微分方程的解为 1281 | \[\varphi(x) = 1282 | \left(C_1\cos\frac{\sqrt{2}}{2}x+C_2\sin\frac{\sqrt{2}}{2}x\right)\e^{\frac{\sqrt{2}}{2}x} 1283 | + \left(C_3\cos\frac{\sqrt{2}}{2}x+C_4\sin\frac{\sqrt{2}}{2}x\right)\e^{-\frac{\sqrt{2}}{2}x}.\] 1284 | 设原方程的特解为 $\varphi^*(x)=A\e^x$, 代入原方程得 $A=1$, 故特解为 $\varphi^*(x)=\e^x$, 因此原方程的通解为 1285 | \[y = \left(C_1\cos\frac{\sqrt{2}}{2}x+C_2\sin\frac{\sqrt{2}}{2}x\right)\e^{\frac{\sqrt{2}}{2}x} 1286 | + \left(C_3\cos\frac{\sqrt{2}}{2}x+C_4\sin\frac{\sqrt{2}}{2}x\right)\e^{-\frac{\sqrt{2}}{2}x}+\e^x.\] 1287 | 结合初值条件知 $C_i=0$ $(i=1,2,3,4)$, 故原方程满足初值条件的解为 $y=\e^x$. 1288 | 1289 | (11) $\lambda^2-2\lambda+2=0\Rightarrow\lambda=1\pm\upi$, 故相应齐次线性微分方程的通解为 1290 | \[\varphi(x)=(C_1\cos x+C_2\sin x)\e^x.\] 1291 | 设原方程的特解为 $\varphi^*(x)=x(A\cos x+B\sin x)\e^x$, 代入原方程得 $A=0,B=2$, 1292 | 故特解为 $\varphi^*(x)=2x\e^x\sin x$, 故原方程的通解为 1293 | \[y=(C_1\cos x+C_2\sin x)\e^x+2x\e^x\sin x.\] 1294 | 1295 | (12) $\lambda^2-5\lambda+6=0\Rightarrow\lambda_1=2,\lambda_2=3$, 故相应齐次线性微分方程的通解为 1296 | \[\varphi(x)=C_1\e^{2x}+C_2\e^{3x}.\] 1297 | 设原方程的特解为 $\varphi^*(x)=(Ax+B)\e^{-x}$, 代入原方程得 $A=1,B=0$, 1298 | 故特解为 $\varphi^*(x)=x\e^{-x}$, 因此原方程的通解为 1299 | \[y=C_1\e^{2x}+C_2\e^{3x}+x\e^{-x}.\] 1300 | 1301 | (13) 令 $x=\e^t$, 则原方程化为 1302 | \[\frac{\diff}{\diff t}\left(\frac{\diff}{\diff t}-1\right)y+5\frac{\diff}{\diff t}y+13y 1303 | = \frac{\diff^2y}{\diff t^2}+4\frac{\diff y}{\diff t}+13y=0.\] 1304 | 特征方程为 $\lambda^2+4\lambda+13=0\Rightarrow\lambda=-2\pm3\upi$, 1305 | 故实基本解组为 $\e^{-2t}\cos3t,\e^{-2t}\sin3t$, 1306 | 代回原变量即得基本解组为 $\frac{1}{x^2}\cos(3\ln x),\frac{1}{x^2}\sin(3\ln x)$, 故通解为 1307 | \[y=\frac{1}{x^2}(C_1\cos(3\ln x)+C_2\sin(3\ln x)).\] 1308 | 1309 | (14) 令$u=2x+1$, 则 1310 | \[\frac{\diff y}{\diff x}=\frac{\diff y}{\diff u}\frac{\diff u}{\diff x}=2\frac{\diff y}{\diff u},\] 1311 | \[\frac{\diff^2y}{\diff x^2}=\frac{\diff}{\diff u}\left(2\frac{\diff y}{\diff u}\right)\frac{\diff u}{\diff x}=4\frac{\diff^2y}{\diff u^2}.\] 1312 | 故原方程化为 1313 | \[u^2\cdot4\frac{\diff^2y}{\diff u^2}-4u\cdot2\frac{\diff y}{\diff u}+8y=0 1314 | \Rightarrow u^2\frac{\diff^2y}{\diff u^2}-2u\frac{\diff y}{\diff u}+2y=0.\] 1315 | 令 $u=\e^t$, 则上述方程化为 1316 | \[\frac{\diff}{\diff t}\left(\frac{\diff}{\diff t}-1\right)y-2\frac{\diff}{\diff t}y+2y 1317 | = \frac{\diff^2y}{\diff t^2}-3\frac{\diff y}{\diff t}+2y=0.\] 1318 | 特征方程为 $\lambda^2-3\lambda+2=0\Rightarrow\lambda_1=1,\lambda_2=2$, 故通解为 1319 | \[y=C_1\e^t+C_2\e^{2t}=C_1u+C_2u^2=C_1(2x+1)+C_2(2x+1)^2.\qedhere\] 1320 | \end{solution} --------------------------------------------------------------------------------